Len's Practice Questions 2

¡Supera tus tareas y exámenes ahora con Quizwiz!

In August 1953, Oscar, who was the owner of 1500 acres of undeveloped timberland, mortgaged his land to Jackson by a mortgage deed, which was not recorded in the Tract Index until January 1954. The mortgage was given to secure a note for $50,000, repayable over a twenty-year period. In September 1953, Oscar executed a warranty deed purport¬ing to convey the same land to Babson in fee simple. Babson recorded immediately in the Grantor-Grantee Index. Then in April 1954, Babson conveyed the same tract to Callahan in fee simple by warranty deed. Callahan paid full market value and recorded the deed at once in the Grantor-Grantee Index. The land in question had never been occupied, fenced or cleared except that between the years 1950-1974, Groves Mining Co., owner of an adjacent tract, regularly drove trucks over a cleared path pursuant to a 1950 agreement with Oscar. The agreement, which was duly recorded, provided that "the parties expressly agree and Oscar promises that Oscar and his successors shall refrain from obstructing the said described path-way across Oscar's land which the Groves Mining Co. and its successors may perpetually use as a road, in consideration of which Groves, and its successors, will pay the sum of $700 per annum." During the period between 1954 through 1975, Callahan collected the yearly $700 fees from Groves and also paid all property taxes. In November 1977, Callahan entered into a contract to sell the land to Daniels. The contract called for a "good and marketable title" with an express exception for rights under the 1950 Oscar Groves agreement. However, Daniels learned about the 1953 mortgage in the course of his title search and refused to honor the contract for sale of the property. Assume this jurisdiction follows a lien theory for mortgages and has a pure-notice type recording statute. 91. Asserting that he had marketable title, Callahan instituted suit against Daniels for specific performance. If Callahan prevails, it will be because (A) Callahan's grantor, Babson, recorded before Jackson (B) Jackson's prior recorded mortgage is deemed to be out¬side Callahan's chain of title (C) Callahan's grantor, Babson, had no notice of Jackson's mortgage interest (D) as between two warranty deeds, the latter one is controlling

(B) A prospective purchaser of real estate or of an interest in real estate is chargeable with knowledge of what appears in the Grantor-Grantee Index, the legal record required to be main¬tained by the Recorder; generally, he is not chargeable with notice of that which appears in other records which may be kept as a convenience, such as a Tract Index. Thus, if Callahan pre¬vails, it will be because the record of the mortgage in the Tract Index was outside this chain of title, and thus not constructive notice to third persons.

100. If the property in question was located in a state having a notice-type statute, which of the following parties would ultimately prevail? (A) Dave (B) Betty (C) Clyde (D) none of the above

100. (A) Under a notice type statute, the subsequent bona fide purchaser without notice prevails over the prior interest whether the subsequent purchaser records or not.

43. For the purposes of this question only, assume that the orig¬inal leasehold agreement provided for the rental of the three story house in addition to a Tk acre tract upon which the house was located. During Moore's tenancy, the State Highway Authority filed proceedings to condemn two acres of the tract for the purpose of highway construction. As a result, Moore contacted Davis and informed him that he should be entitled to an apportioned reduction in the rental. After Davis refused to reduce the rent, Moore brings an action to have his rent apportioned pro fanto. Judgment for: (A) Davis, although Moore would be entitled to terminate the lease (B) Davis, since Moore would be held to the original leasehold contract (C) Moore, since the value of his leasehold interest was reduced pro tanto (D) Moore, since eminent domain operates as an appor¬tionment of rent

43. (B) The question of law presented here, namely, when there is a partial taking of a leasehold estate by condemnation and the remaining portion is susceptible for occupation, shall the con¬tractual monthly rental be abated pro tanto for the part taken during the remainder of the time of the lease? The majority rule is that rentals are not abated in this situation, but rather the ten¬ant is obligated to continue the payment of the rentals provided in the lease contract and must look to an apportionment of the damages assessed against the condemning authority based on the reduced value of his lease.

72. Suppose that Gamma did, in fact, deliver to Delta a quit¬claim deed which made no reference to the mortgage. If Gamma thereafter defaulted on the mortgage and Epsilon brought an in personam action against Delta to recover the amount due on the mortgage debt, the mortgagee will probably (A) succeed, because Epsilon is a third party beneficiary of the agreement between Gamma and Delta (B) succeed, because there was an implied delegation of duties to Delta (C) not succeed, because Delta did not promise to pay the mortgage debt (D) not succeed, because the law does not permit the mortgagor to delegate duties under this type of a property transfer

72. (C) In this case, Epsilon, the mortgagee, is bringing a claim against Delta, the grantee. Since the facts state that the deed "made no reference to the mortgage," it is presumed that Delta took "subject to" Gamma's mortgage. Therefore, Delta will not be personally liable, because she did not promise to pay the mortgage debt. Choice (C) is correct. Note: Only Gamma as mortgagor would be liable to Epsilon under these facts.

9. Assume for the purposes of this question only that Wanda and Hugo are married and own Blackacre as tenants by the entirety. If Wanda transfers her interest in Blackacre by quit¬claim deed to Louis, without Hugo's knowledge, what inter¬est, if any, does Louis have? (A) no interest (B) an undivided one-half interest with right of survivorship (C) an undivided one-half interest without right of survivorship (D) a lien against the entire property

9. (A) A tenancy by the entirety, unlike a joint tenancy, does not allow either party to convey away his or her interest in the property without the other's consent. As a result, Louis would not have any interest in Blackacre.

Questions 96-98 are based on the following fact situation. Scenicacre is a twenty acre tract of open meadowland with numerous streams running through. In 1965 Ohner had good record title to Scenicacre in fee simple absolute. In 1966 Ohner delivered to Son, with intent to make a gift of Scenicacre to Son, a deed signed by Ohner, naming Son and his heirs grantee and appearing valid on its face. Son neglected to record the deed. In 1970, Buyer, aware of the existence of the Ohner-to-Son deed, sought out Ohner and asked to buy for $10,000 a deed of Scenicacre from Ohner to Buyer and his heirs. Ohner executed such a deed and Buyer promptly recorded it. Buyer's intent was to acquire color of title and obtain ownership of Scenicacre by adverse possession, In 1970, Buyer constructed a fence around Scenicacre. In 1971, Son presented his deed of Scenicacre to Purchaser and, for $1 5,000 paid by Purchaser, signed and delivered a deed of Scenicacre in favor of Purchaser and his heirs. After receiving the deed, Purchaser made no effort to search the title, to exam¬ine the property or to record the deed. In 1975, Vendee paid Buyer $20,000 and Buyer delivered to Vendee a deed of Scenicacre in favor of Vendee and his heirs. Vendee had examined the property, had searched the title, and had no knowledge of Buyer's awareness of the prior Ohner-to-Son instrument. Although Vendee did not reside on the prop¬erty, he regularly visited Scenicacre twice a week. Vendee recorded his deed. In 1979 for $25,000 paid by Oscar, Purchaser signed and delivered a deed of Scenicacre naming Oscar and his heirs as grantees. Oscar obtained from Purchaser the Son-to-Purchaser deed, and from Son the Ohner-to-Son deed and took all three documents to the Recorder's Office to be recorded. At Oscar's request, all three instruments were recorded in the following order: (1) Ohner-to-Son; (2) Son-to-Purchaser; (3) Purchaser-to-Oscar. Before Oscar had paid Purchaser and taken his deed, Oscar visited Scenicacre and observed the fence. However, Vendee was not present when Oscar visited the property and nothing suggested who, if anyone, was using it. In any case, Oscar did not attempt to search the title before making his purchase. NOTE: This jurisdiction uses Grantor-Grantee Indices and has no Tract Index. 96. In 1980, what is the present state of title to Scenicacre if the jurisdiction's recording act provides: "Every conveyance of real property shall be invalid as against any person, except the grantor, his heirs, and assigns, having actual notice thereof, unless it is recorded as provided by Statute?" (A) In a notice jurisdiction, Oscar, as a subsequent bona fide purchaser, is only chargeable with notice of what appears in his chain of title and therefore would acquire record title to Scenicacre. (B) In a race-notice jurisdiction, Buyer would acquire equitable title to Scenicacre, since he erected the fence and failed to inform Vendee of the prior Ohner-to-Son instrument. (C) In a race-notice jurisdiction, Vendee, as subsequent bona fide purchaser without notice of prior Ohner-to-Son instrument, would acquire record title to Scenicacre. (D) In a notice jurisdiction, purchaser would acquire record title to Scenicacre even though he failed to record his deed.

96. (A) First of all, students should note that the aforementioned recording act is a pure notice type statute. Under this type of statute the subsequent bona fide purchaser prevails over any prior interests whether the subsequent purchaser records or not. The subsequent purchaser's priority is determined by his status at the time he acquires his deed; there is no premium on his race to the recorder's office. Secondly, in referring to our factual situation, Oscar has a superior right in the property. At the time Oscar purchased Scenicacre, Purchaser had not recorded. Consequently, Oscar as a subsequent bona fide purchaser for value, had established his priority as soon as he purchased the property. Moreover, note that a subsequent bona fide purchaser in a notice jurisdiction is charged with notice of every recorded instrument of conveyance which is a link in his chain of title.

171. If Pieper had chosen to evict Marino immediately upon the end of the three-year period and before he had accepted the next rental check, what, if proven, is Marino's strongest argument to prevent eviction? (A) The leasehold contract was void ab initio. (B) Pieper had not given Marino requisite notice for terminating a periodic tenancy. (C) Pieper had not given Marino requisite notice for terminating a term of years. (D) The terms of the leasehold contract were binding on both parties.

(B) Marino's strongest argument is that Pieper had not given him notice for terminating a periodic tenancy. Notice of termination is an integral element of a periodic tenancy. Remember a periodic tenancy continues indefinitely in the absence of either party's giving notice of termination.

Aimee Aldretti was a real estate investor who specialized in developing suburban shopping malls throughout the southeast. She constructed a mall outside of Atlanta and began renting commercial store space. The complex, which was called Bull¬dog Junction, contained space for ten stores. One of the ten¬ants renting space in Bulldog Junction was Yuki Yamagato, who planned to open a sushi restaurant. Yuki's lease with Aimee contained a covenant not to compete in which Aimee agreed not to rent to any other tenant(s) in a five mile radius who would be operating a restaurant. The lease agreement ex¬tended from January 1, 1998 to December 31, 2000. In February, 1999 Aimee purchased the Yellow Jacket Mall, a shopping complex, located on a parcel of land adjacent to Bulldog Junction. There were ten stores located within the Yellow Jacket Mall. When Aimee acquired Yellow Jacket, existing leases were in effect for each of the stores. One of the businesses was a sushi restaurant operated by Benny Hana. When Yuki learned that Benny was now one of Aimee's ten¬ants, he informed her that she was in violation of the non¬compete clause in their lease. Aimee informed Yuki that Benny had entered into his lease before she acquired the property. Since she merely assumed the lease, there was nothing she could do. 177. If Yuki sues Aimee for breach of covenant not to com¬pete, which of the following would provide Aimee with her STRONGEST defense? (A) The negative covenant does not extend to the Yellow Jacket Mall property. (B) Benny's lease was already in effect when Aimee acquired the Yellow Jacket Mall. (C) The covenant is unenforceable as against public policy. (D) Yuki lacks privity of estate to enforce such a covenant against Benny.

(B) Once again, always remember the best way to handle multiple-choice or MBE questions is by process of elimination. Choice (A) is wrong because the negative covenant contained in Yuki's lease with Aimee does extend to the Yellow Jacket Mall which is within the five mile radius area. Choice (C) is incorrect because covenants not to compete are valid as long they are reasonable and do not restrain competition indefinitely. In this instance, the covenant is reasonable because it only restrains competition within a five mile radius. Choice (D) is wrong because Yuki is not enforcing the covenant against Benny but rather is suing Aimee for breach of covenant. Clearly, there is privity of contract between Aimee and Yuki based on the lease agreement. Therefore, choice (B) must be correct.

In 1986, Scully purchased a one-acre lot in the Pacific Highlands section of Los Angeles. The next year, he built a truly elegant 10,000 square foot Mediterranean house with an ocean view on the property. The house, which conformed to all city building code regulations, featured six bedrooms, six baths, two family rooms, a black granite kitchen, a two-story walnut library and a sixty-foot mosaic tile pool. The house and the pool were heated entirely by solar energy. Scully was advised that in order for this method of heating to be effective, it was essential that unrestricted sunlight fall upon the home without any blockage. In 1990, Vincent has just bought an unimproved lot adjacent to the Scully tract. Vincent has applied for a building permit to construct a three-story house ten feet from Scully's property line. The plans and specifications of Vincent's pro¬posed construction meet with city code and have been approved by the building inspector. However, if Vincent's house is built as planned, Scully's home will lose about 60% of the sunlight it currently receives. Consequently, Scully has filed an action seeking an order that would require Vincent to construct her house in the center of her lot, so as to minimize the loss of sunlight to Scully's house. 179. In the event that Vincent moves to dismiss for failure to state a claim on which relief may be granted, which of the following will be the critical question? (A) Will Vincent's house, if built as planned, substantially interfere with Scully's use and enjoyment of his property? (B) Can Vincent's house be built in the center of her lot without unreasonable interference to Vincent? (C) Does the court recognize access to sunlight as a right entitled to legal protection? (D) Will Vincent's house, if built as planned, require Scully to install an entirely new heating system to compensate for the loss of solar energy?

(C) For Scully to prevail in his action seeking a court order to have Vincent construct her house in the center of her lot, rather than ten feet from Scully's property line, a negative easement by implication would be required. In most jurisdictions negative easements for light, air, and view have been repudiated, absent an express grant between the parties, since such use restrictions would seriously hamper land development. Burby, Real Property, p. 75. Therefore, the critical issue in Vincent's motion to dismiss is whether the court recognizes access to sunlight as a right entitled to legal protection. Choice (C) is correct. Since Vincent would not be creating a nuisance by building a home on her own property unless a negative easement existed, Scully would have no tort action against her. Therefore, choice (A) is incorrect because the easement issue must be addressed first. Choice (B) is incorrect because where no legally protected right exists, the other party need not seek an alternate remedy. Choice (D) is incorrect because absent a recognized right to restrict Vincent's activities on the servient estate, the issue of damages to the dominant tenement owner, Scully, is moot.

In 1958, Ohner was the undisputed, record owner of Grass-acre, a tract of farmland in Landenburg flanked on its western border by the Red River. In that year Ohner leased Grassacre to Tenant for a term of 7 years. In 1959 Ohner died a spinster, leaving a will by which she gave her entire estate to the Girl Scouts' of America Association. Grassacre was not specifically men¬tioned in the will. Prior to 1958, the southern half of Grassacre had been cleared and placed in cultivation, while the northern half remained wooded and unused except for an occasional hike or a gather¬ing of timber for use as domestic fuel or building material. Tenant continued with this established pattern of use. Now (in January, 1982) he still is using the southern half of Grassacre as a farm and as his home, and the northern half in the manner just indicated. At no time has he had communication of any sort from the Girl Scouts' organization. Since hearing of Ohner's death in 1959, Tenant has never tendered any rent to anyone; nor has he ever paid taxes on Grassacre. In 1960, Neighbor, the owner of the farm located just to the east of Grassacre's northern half, began, with no permission from anyone, to make regular use of a trail across Grassacre's wooded area whenever he or his cattle needed to reach the Red River. This activity has never inconvenienced Tenant, and neither he nor anyone else has ever objected to it. Neighbor and his cattle continue to traverse Grassacre's woods when they want to reach the river. In the course of a recent inventory of testamentary gifts to the Girl Scouts', lawyers for the Association have stumbled upon the Girl Scouts' claim to Grassacre. The Girl Scouts would like to have Tenant evicted, to clear the northern half, and to fence the whole tract for a summer camp thereby excluding Neighbor and his stock. The statutory period for adverse possession in this jurisdiction is 20 years. 158. In an ejection action by the Girl Scouts' Association against Tenant, the former will prevail (A) because Tenant discontinued paying rent following Ohner's death (B) if Tenant leased Grassacre to Jones for 3 years while he (Tenant) was called into military service (C) If Tenant believed that his lease with Ohner was still in effect after the latter died (D) because Tenant never paid taxes on Grassacre

(C) It is important to remember that adverse possession must be (a) actual and exclusive, (b) hostile, (c) peaceable, (d) open, (e) notorious, (f) continuous, (g) adverse, and (h) for the statutory period. Be aware that the intention of the adverse possessor is all important. Here, if the adverse possessor believed that his leasehold contract was still in effect, then his possession would not be hostile. Moreover, no one claiming less than a freehold estate in land can get title by adverse possession.

161. Assume for the purposes of this question only that the Rule Against Perpetuities is in effect in this jurisdiction. Which of the following statements is/are correct regarding Doris' interest under her father's will? I. It is a contingent remainder. II. It is an executory interest of a shifting type. III. It is void under the Rule Against Perpetuities. (A) I only (B) II only (C) I and III (D) II and III

. (D) Executory interests and devises are subject to the Rule Against Perpetuities. In the present case, the Rule is violated since Doris' interest will neither vest nor fail within the lifetime of Monte. This is so because the horses may fail to receive proper care after Monte's death. Had Doris' interest been limited to take effect "if the horses fail to receive proper care during Monte's lifetime" (instead of "if ever the horses fail to receive proper care"), then the interest would not have been void under the Rule Against Perpetuities.

101. Suppose that the jurisdiction in which the property is located has a pure race statute as their deed recordation law. Under these circumstances, which of the aforementioned parties would ultimately prevail? (A) Dave (B) Betty (C) Clyde (D) none of the above

101. (A) In a pure-race jurisdiction, the first to record wins and a subsequent purchaser need not be bona fide and without notice since he will prevail if he records first.

Questions 106-108 are based on the following fact situation. Wolverine Land Development Company (hereafter referred to as WLDC) was the owner of a 400 acre tract of land in northern Michigan. Over the course of time, WLDC developed two residential subdivisions of the land, called East Lake Shore and West Lake Shore, each of which contained 150 acres. These subdivisions were created by separate plats which made no reference to each other. The restrictions in the plats were, however, substantially identical. The plats and each deed provided that "the use of the land herein conveyed is restricted to single family dwellings only and this covenant is intended to apply to each and every lot in the subdivision and runs with the land, binding every lot owner, his heirs and assigns." After all but four lots in each subdivision had been sold by WLDC, it sold 50 acres of the remaining 100 acres of land to the Burning Tree Country Club (hereafter referred to as BTCC) by a deed containing the following provisions: "This deed is executed and accepted with the understanding that the property above described is hereby restricted so that same may only be used for the following purposes: (a) As a country club, with a golf course, pool, tennis courts, club house, eating facilities and other improvements appropriate to a country club; (b) Said property may also be subdivided and plat¬ted as a residential subdivision similar to East Lake Shore and the property shall thereafter be used in accordance with and conveyed subject to residential restrictions which shall conform with those restrictions in force against East Lake Shore; (c) The restrictions herein contained shall be deemed covenants running with the land, and for breach of any covenant herein, grantor-Wolverine Land Development Company, its successors and assigns may, at its option, reenter and terminate the estate conveyed hereby." At the time of this conveyance, WLDC retained title to the remaining 50 acres in the original 400 acre tract. Thereafter, WLDC developed an exclusive shopping center on 25 acres of the retained land. Gale Galleria holds a 30 year lease on a building in the shopping center in which he operates a high quality department store. Another tenant in the shopping center is Skate-Around-Town, Inc. which has a month-to-month lease and sells roller skates, skateboards and skating accessories. In February 1980, WLDC sold the remaining eight residential lots in East Lake Shore and West Lake Shore. The next month, WLDC executed the following instrument to BTCC: "Wolverine Land Development Company, for itself, its successors and assigns, does hereby release, surrender and quitclaim all rights, title or other property interest in that certain acres owned by the Burning Tree Country Club." At the time this instrument was executed, BTCC had built a club house, golf course and tennis courts on a portion of its land, and it had 25 acres of vacant land upon which it wished to build a complex containing a hotel, a shopping mall surrounded by high-rise buildings and luxury apartments. 106. Which of the following would best describe BTCC's interest in the 50 acre tract which it purchased from WLDC (i.e., BTCC's interest immediately after the purchase but prior to the March, 1980 instrument)? (A) fee simple determinable (B) fee simple subject to condition subsequent (C) determinable fee subject to an executory interest (D) easement appurtenant

106. (B) A fee simple subject to condition subsequent means a fee simple subject to being terminated by exercise of the power of termination or right of re-entry for condition broken. Students should note that the important characteristic which distinguishes this type of estate from a fee simple determinable, is that the estate will continue in the grantee, or his successors, unless and until the power of termination is exercised. The estate in fee simple subject to a condition sub¬sequent does not end, ipso facto, upon the happening of the named event. The basic difference, therefore, is that the fee simple determinable automatically expires by force of the special limitation contained in the instrument creating the estate, when the stated contingency occurs, whereas the fee simple on condition subsequent continues despite the breach of the specified condition until it is divested or cut short by the exercise by the grantor of his power to terminate.

108. With respect to the 50 acre tract which BTCC purchased from WLDC, which of the following most accurately describes the deed restrictions (a) and (b)? (A) affirmative covenant(s) (B) equitable easement(s) (C) express easement(s) appurtenant (D) equitable servitude(s)

108. (D) An equitable servitude is a restriction on the use of land enforceable in a court of equity. An equitable servitude is more than "a covenant running with the land in equity" because it is an interest in land. The term, equitable servitude, is broader than "equitable easement" because it applies not only to land but also to chattel property such as a business. Note that choice (A) is incorrect because the deed restrictions were negative, or restrictive, covenants rather than affirmative covenants.

110. Oliver's proposed designation of the ten-acre tract for construction of a school building would best be described as a (an) (A) equitable servitude (B) restrictive covenant (C) unenforceable restriction (D) easement for public use

110. (D) The public dedication of the 10 acre tract for the erection of a school would be construed as an easement for public use. Students should note that although ordinarily dedication arises out of the creation of a public use in private land, a municipality or other subdivision may likewise acquire title and then create a public use by dedication.

In 1965, Collins owned Devon Farms, a fifty acre tract located near Kennett Square in West Chester County. Devon Farms included the old family mansion Collinwood, with its private tennis courts, stables, and a small cottage which had been unoccupied for several years. In 1966, Collins sold two-acre lots each to Jones, Smith, Harris and Perkins in the northeastern sector of the Devon Farms tract. Early in 1970, Collins built a small grocery store which, with its parking lot, occupied one-half acre of the northeastern corner of the tract. In August 1970, Collins rented the store and parking lot to Tenant for a ten year period. In 1972, Collins had sold thirty-four one-acre lots in the northwestern and southern sectors of the tract, using two and a half acres for streets. He retained (in addition to the store) only a five-acre tract which included Collinwood, its stables, tennis courts and the cottage. The first four lots which Collins sold in 1966 did not include any deed restrictions. The next thirty-four lots, sold between 1967 and 1972, provided in each deed the following provision: "It is an express covenant and condition that the property hereby conveyed shall not be used for other than single family residences." In February 1972, Collins recorded a declaration reciting the aforementioned restriction with the West Chester County Recorder's Office. In March 1973, Volley, a tennis pro, purchased the cottage and the tennis courts from Collins. Because of the proximity of the tennis courts to the mansion, Collins' deed to Volley provided: "The grantee hereby covenants for himself, his heirs, successors and assigns that so much of the property hereby conveyed as is now occupied by the tennis courts shall not be used for construction of any building or structure whatsoever." In 1974, Collins sold the mansion and his remaining land, except for the grocery store and parking lot, to Boris. No restrictions were contained in this deed. Boris immediately began to renovate the mansion as a resort hotel. In January 1975, West Chester County enacted a zoning ordinance; the relevant provision applicable to the former Devon Farms tract permitted only single family residences, "except to whatever extent any prior nonconforming uses may not be subjected to such restrictions." On September 1, 1975, the State brought suit to have the property on which the parking lot was located, condemned in an eminent domain proceeding for high¬way construction purposes. 116. If Jones brought suit in 1970 to enjoin Collins from constructing the grocery store, judgment would have been for (A) Jones, since the construction of the grocery store would constitute a nonconforming use (B) Jones, since a common development scheme had been established for the entire Devon Farms tract (C) Collins, since there was no enforceable restriction to prevent the construction of the grocery store (D) Collins, unless Jones was joined by Smith, Harris and Perkins as party-complainants

116. (C) In determining whether Jones should succeed in his action to enjoin Collins from constructing the grocery store in 1970, it is necessary to ask ourselves the following question: was there any restriction (i.e., restrictive covenant) on the northeastern sector in effect in 1970 prohibiting the construction of a grocery store? The answer is no. In fact, we are certain that Collins' intent was not to restrict the development of the northeastern sector for residential purposes since none of the lots in that section contained deed restrictions (including Jones' property).

121. What is the probable legal effect of the West Chester County zoning ordinance on the Devon Farms tract? (A) Jones, Smith, Harris and Perkins would be restricted to maintain only single family residences on their two-acre lots. (B) The zoning ordinance would require the immediate cessation of the grocery store business as a non¬conforming use. (C) Boris would be precluded from continuing to use the mansion as a resort hotel. (D) All prior uses of the Devon Farms tract would be permitted to continue in accordance with the zoning ordinance.

121. (D) All prior use of the Devon Farms tract would be permitted under the zoning ordinance which, although restricting future development of the said tract to single family residences, provided that "any prior non-conforming uses" would be permitted.

134. Suppose Johnson by the excessive excavating conducted in his sand removal business causes the collapse of a large storage building on Dave's property. If Dave brings an action to recover damages for the collapse of his storage building, Dave will probably (A) be successful if he can prove that Johnson was negligent in his excavations (B) be successful unless Johnson exercised due care in his excavations (C) not be successful, because Johnson could not foresee that the adjacent storage building would collapse (D) not be successful, because an adjacent land owner is under no affirmative duty to laterally support his neighbor's land

134. (A) Choice (A) is correct because an adjacent landowner is liable for damage to buildings on neighboring property resulting from his negligent excavation. Moreover, a landowner is absolutely liable for the collapse and or damage to neighboring structures (even if the excavation is done without negligence) so long as it can be shown that the land would have collapsed in its natural state.

140. Assume for the purposes of this question only that the deed for the remaining parcel contained the identical restrictions of the original three parcels. Sid Slick, ignoring these restrictions, was able to use his political influence to persuade the County Zoning Board to re-zone the said parcel for commercial use. The residents of Tower Hills seek to enjoin the construction of the shopping center. The residents will have: (A) little chance of preventing the projected commercial development (B) no standing to enjoin the construction of the shopping center (C) no chance of succeeding because the zoning ordinance takes precedence over the covenant running with the land (D) succeed in their cause of action in accordance with the common development scheme

140. (D) (D) is the correct choice since a subsequent zoning ordinance will not destroy the restrictive covenant as long as there has not been such a change in the neighborhood as to make the physical use and enjoyment of the restriction meaningless. Quite so, the restrictive covenants in the deeds of the owners of Tower Hills will be enforceable in accordance with the common development scheme preserving the residential character of the property.

142. Suppose that during Pittsburgh Coal Co.'s strip mining operations, Archie returns to the farm and demands a proportionate share of the royalties paid to his brother Adam. Which of the following is the LEAST accurate statement regarding Archie's right to share in the royalties? (A) As a joint tenant, Archie would be entitled to contribution for a proportionate share of the royalties. (B) It would be inequitable for one concurrent owner to receive an unapportionate share of the royalties. (C) Archie's redomiciling constituted a severance of the joint tenancy, which resulted in a destruction of his interest in the mining royalties. (D) Since Archie's redomiciling did not effectuate an ouster, he retained his right to contribution as a joint tenant.

142. (C) Choice (C) is the least accurate statement because the redomiciling (of a joint tenant) does not effectuate a severance. Generally, a severance of the joint tenancy can be made by (1) conveyance inter vivos or (2) by partition only. Be aware that a severance can never be made by will because survivorship is prior to and defeats the effect of the will.

146. Assume for the purposes of this question only that H, W, and B die in a common disaster and as a consequence, the order of their deaths cannot be established by proof. H's will devises whatever interest in the land in question that is subject to disposition by his will to his son S. Thus, S owns (A) an undivided one-third interest in the land (B) an undivided one-quarter interest in the land (C) an undivided one-half interest in the land (D) an undivided three-quarter interest in the land

146. (D) A tenancy by the entirety is destroyed by the death of both spouses, since the entirety interest only applies to a husband and wife during marriage. Death, just as divorce, eliminates the unity of person, and thus the need for the attribute of survivorship is also eliminated. Disposition of property upon the death of both spouses would then follow just as if the tenancy by the entirety were severed into a tenancy in common. In other words, the heirs would inherit the undivided interest of the deceased cotenant. Similarly, the disposition of a joint tenancy would follow as if a tenancy in common had existed. Therefore, even though neither a tenancy by the entirety nor a joint tenancy can be disposed of by will, a tenancy in common can be so devised. Thus, S would obtain H and W's one-half interest in the land under the 1989 deed as well as H's one-quarter interest under the 1990 deed. Alternative (D) is therefore the correct answer. See Smith and Boyer, Law of Property, pp. 61-65.

164. Dickers, the owner of lot 29 in the Brunswick development sells his land to Youngblood with a deed containing no restrictions. The Brunswick Association, pursuant to Article 1 of the Declaration, sues Youngblood to collect an assessment levied for financing of the community nursery school. Which of the following is the best argument for Youngblood? (A) There is not privity of contract between Candlestick and The Brunswick Association. (B) Since the charge constitutes a lien, there is no personal obligation on Youngblood's part. (C) There is no privity of contract between Dickers and Youngblood. (D) There is no privity of estate between Dickers and Youngblood.

164. (B) The provision in Article 1 relating to the creation of the "Brunswick Fund" is enforceable as a covenant running with the land. The effect of such a covenant is that the benefit, burden or both pass to succeeding holders of the estate of the original covenanting parties. Therefore, the assessment may be enforceable against Youngblood since he is "succeeding to the estate" of Dickers. However, Article 1, Section I stipulates that "the assessment or charge when due shall become a lien upon the parcel of the property." Consequently, since the assessment involves an in rem interest, choice (B) is the best answer. Note that choices (C) and (D) are incor¬rect because the conveyance of the property from Dickers to Youngblood created both privity of estate and privity of contract between the parties.

On May 1, 1981, Mayfair purchased a large tract of river¬front property (hereafter referred to as Riverview) in downtown St. Louis for $750,000. According to the terms of the real estate sales agreement, Mayfair was required to make a 20% down payment of $150,000. To finance the balance of the purchase price, Mayfair borrowed $600,000 from the Center City Bank with the loan being secured by a first mortgage. The mortgage was for a thirty-year term with a fixed 7 1/2% interest rate. The mortgage instrument contained a standard "due on sale" clause that provided: "If all or any part of Riverview or any interest in it is sold or transferred...said Mortgagee may, at its option, require immediate payment in full of all sums secured." In addition, the mortgage contained a prepayment penalty clause that provided that in the event the mortgagor (or his successors in interest) should sell or transfer the property be¬fore the due date, the mortgagee would be entitled to increase the interest rate and collect 20% interest on the unpaid principal balance. After purchasing the property, Mayfair developed Riverview with upscale shops and restaurants. The commercial development was a huge success and in 1997 Mayfair decided to sell a 50% interest in Riverfront to Langford, a real estate investor, for $2,000,000. After Mayfair and Langford entered into their real estate sales agreement, Mayfair contacted Center City Bank to make arrangements to pay off the unpaid balance of the mortgage. Center City Bank informed Mayfair that he owed $500,000 in mortgage principal and $100,000 as a prepayment interest penalty. 184. Assume that Mayfair contests the validity of the prepayment penalty clause. May Center City Bank enforce the prepayment charge before Mayfair pays off the mortgage? (A) No, because the prepayment penalty charge "clogs the equity of redemption". (B) No, because the prepayment penalty clause constitutes an unlawful restraint against alienation. (C) Yes, the prepayment penalty clause is valid because the mortgagee will receive diminished interest since the principal is being prepaid before maturity. (D) Yes, because the due on sale clause allows the mortgagee to unilaterally accelerate the prepayment charge.

184. (C) Once again, mortgages is the most heavily tested area in Property on the Multistate. Last summer, bar candidates were required to know that a prepayment penalty clause provision contained in a mortgage instrument is enforceable. The rationale is that a lender will lose anticipated interest in the event that the mortgage principal is prepaid before the due date. As such, choice (A) is wrong. Answer (B) is wrong because a prepayment penalty clause does not restrict the free alienability of land. Choice (D) is not the best answer because the question concerns the validity of the prepayment penalty clause, not the due on sale clause.

2. In an appropriate action to construe the agreement between Wilkes and the Lovette Preservation League, the court will determine provision (b) to a (an) (A) easement (B) profit (C) covenant running with the land (D) equitable servitude

2. (A) Section (b) of the Wilkes-Lovette agreement would be construed as an easement. An ease¬ment is the right of one person to go onto the land in possession of another and make a limited use thereof. Students are advised to refer to CAVEAT 44 for a more complete explanation on the distinction(s) between easements, profits and licenses.

22. During Burkhart's lifetime, Drew's interest may best be described as a (an) (A) estate pur autre vie (B) contingent remainder pur autre vie (C) vested remainder pur autre vie (D) shifting executory interest pur autre vie

22. (D) Alternative (D) is correct since Drew's interest would be a shifting executory interest pur autre vie. An executory interest is an interest that divests the interest of another transferee (shift¬ing executory interest) or that "follows a gap" or divests the interest of the transferor (springing executory interest). Furthermore, his shifting executory interest would be pur autre vie, since it would be an estate for the life of another (Carr).

Carson is the owner of a parcel of land known as Tall Acres which is situated upon the top of Candy Rock Mountain. Located below Tall Acres is Grasslands, a forty acre hillside estate which is owned by DuVall. Crystal Stream is a non-navigable watercourse that originates at the top of Candy Rock Mountain and runs all the way down into the San Vicente Valley. Both Tall Acres and Grasslands are within the watershed of Crystal Stream. When DuVall purchased Grasslands in 1956, he started taking water from Crystal Stream and used it to irrigate the southern half of his property which he has used as a farm. Prior to 1956, the southern half of Grasslands had been cleared and placed in cultivation, while the northern half remained wooded and unused except for an occasional hike or gathering of timber for use as domestic fuel. DuVall continued this established pattern of use. Now (January, 1988), he is still taking water from Crystal Stream and using it to irrigate the southern half of Grasslands. In 1986, Carson built a home on Tall Acres and started taking water from Crystal Stream for domestic purposes. During that year there was heavy rainfall, and this caused Crystal Stream to run down the mountain at a high water level. The next year, however, there was a drought. As a result, Crystal Stream flowed at a very low level. Consequently, there was only enough water to irrigate DuVall's farmland or, in the alternative, to supply all of Carson's domestic water needs and one-quarter of DuVall's irrigation requirements. Candy Rock Mountain is located in a jurisdiction where the period of prescription is fifteen years. 77. Inasmuch as Crystal Stream is still flowing at a very low level and Carson is continuing to take water for his personal needs, there is insufficient water to irrigate Grasslands. As a consequence, DuVall brings an appropriate action to declare that his water rights to the stream are superior to those of Carson. In addition, DuVall moves to have the full flow of Crystal Stream passed to him, notwithstanding the effect it might have on Carson. If this state follows the common law of riparian rights but does not follow the doctrine of prior appropriation, judgment for whom? (A) Carson, because as an upstream landowner, he would have superior rights to the water than a downstream owner. (B) Carson, because domestic use is superior to and is pro¬tected against an agricultural use. (C) DuVall, because he has obtained an easement by prescription to remove as much water as he may need. (D) DuVall, because he has put the water to a beneficial use prior to Carson's use and has continuously used the water.

77. (B) Under the doctrine of riparian rights there are two theories, the "natural flow" theory and the "reasonable use" theory. The former stresses the right of each owner to have the natural state of the stream or lake undiminished in both quality and quantity. The latter stresses maximum use by each owner provided it does not interfere with like use by other owners. Also, under the riparian doctrine, use of water for natural purposes—domestic use, watering of stock—is superior to use of water for artificial purposes—irrigation, mining, industry. Smith and Boyer, Survey of the Law of Property, pp. 186-189. Therefore, based on these rules, Carson will prevail over DuVall in accordance with answer choice (B). Choice (A) is incorrect because the theory of riparian rights allows a lower riparian (DuVall) to sue an upper riparian (Carson) where the latter's use materially affects the quantity or quality of water—the natural flow—or unreasonably causes damages—reasonable use. Choice (C) is incorrect because DuVall's use was only "adverse" since 1986, not for the statutory 15 years. Choice (D) is incorrect since it states the rule under the prior appropriation theory.

was located in the township of Deer Valley. In 1954, Erickson conveyed Hillacre to The Downhill Development Association "on condition that a ski lodge and resort area be built thereon and the land be used solely for skiing purposes and, in the event that said property is not used as a ski resort, the property shall revert to myself, the grantor, my heirs or assigns." Subsequently, a ski lodge was built and the land was continuously used as a ski resort for over twenty years. In 1959, Stein, owner of Grassacre, a property abutting the northeast corner of Hillacre, began to use a portion of Hillacre in order to gain access to Grassacre for ingress and egress. He used this access road openly, visibly and notoriously until 1980. The period of prescription in this particular jurisdiction was 20 years. In March, 1980, he decided to sell Grassacre to Grich for the purchase price of $500,000. Thereupon, Stein and Grich entered into a written real estate sales contract with closing being set for May 1, 1980. On April 1, 1980, Downer, the president of The Downhill Development Association, was approached by Grich with the request that written easement be given over the access road and that an appropriate instrument be delivered to Grich concurrently with Grich's closing with Stein. Following Downer's meeting with Grich, The Downhill Development Association held a duly announced meeting and voted unanimously to authorize its trustees to grant such an easement. On April 15, 1980, Erickson died. Erickson's sole descendant and successor in interest was his wife, Erika. Upon her husband's death, she immediately notified The Downhill Development Association that if it granted the easement to Grich, Hillacre would revert to her. Grich then brought an appropriate action, joining Stein, Erika and The Downhill Development Association, seeking a declaratory judgment that Stein has a perpetual easement appurtenant to Hillacre. 83. In this action, judgment should be in favor of which of the following parties? (A) Erika, because Grich could not obtain rights against The Downhill Development Association by prescription. (B) Erika, because the easement would violate the negative restriction that Erickson originally sought to impose. (C) Grich, because Stein had already obtained an ease¬ment by prescription. (D) Grich, because the language of the conveyance to The Downhill Development Association was too vague to support any claim by Erika.

83. (C) An easement by prescription arises by adverse use of the servient tenement by the dominant tenant for the period of the statute of limitations. To mature such an easement against a landowner the use must be (a) adverse as distinct from permissive; in derogation of right rather than in subordination to the rights of the landowner; and (b) open and notorious, and (c) continuous and without interruption, and (d) for the period of prescription. See Smith and Boyer, Law of Property, p. 387. Since Stein openly and notoriously used the northeast portion of Hillacre to gain access to Grassacre for the 20-year period, as required by statute, he obtained an easement by prescription. This easement passed on to Grich, who was in privity with Stein by means of the 1980 sales contract. Therefore, Grich's rights are enforceable against the whole world and he will prevail against Erika. Choice (C) is the correct answer.

85. What is the probable legal effect of Warner's mortgage (agreement) in 1918 with the Baden National Bank? (A) The mortgage would be invalid, since Warner conveyed his interest in the property to Tidwell in 1915. (B) The mortgage would be invalid, since the first mortgage in 1898 would take precedence over the second mortgage. (C) The mortgage would be invalid, unless the mortgagee Baden National Bank had knowledge of the prior conveyance to Tidwell. (D) The subsequent mortgagee, Baden National Bank, would prevail as against the prior conveyee-Tidwell, who failed to record before the mortgage was effectuated.

85. (D) Under notice-type recording statutes, an unrecorded conveyance or other instrument is invalid as against a subsequent bona fide purchaser (creditor or mortgagee) for value and with¬out notice. Under this type of statute the subsequent bona fide purchaser prevails over the prior interest whether the subsequent purchaser records or not. Insofar as the subsequent purchaser is concerned, there is no premium on his race to the recorder's office; his priority is determined upon his status at the time he acquires his deed or mortgage.

132. Wilson's strongest argument in an action against Johnson for removal of the barriers which are obstructing his access to the property would be (A) an easement appurtenant is alienable in that any conveyance transferring possession of the dominant tenement also passes the easement privilege (B) Wilson as a bona fide purchaser is protected from Johnson's obstruction even though the easement was unrecorded (C) since the easement was created by implication, Dave's prior and continuous use gave rise to a prescriptive easement (D) Dave and Wilson being in privity of estate could "tackon" their successive use periods

(A) (A) is the correct choice since today the statutory presumption is in favor of creating easements appurtenant for perpetual duration unless the grant specifically limits the interest. On the other hand, one should note that an easement in gross is generally inalienable.

In 1991 Bradley Beachbaum purchased Seaside Heights, a twenty-acre parcel of coastal land, from Maxwell Margate for $175,000. Seaside Heights was undeveloped beachfront property located along the Atlantic coastline in the state of Old Jersey. To finance the purchase, Bradley borrowed $125,000 from the Monmouth Trust Company Bank and paid the balance in cash. As per the terms of the loan agreement, Bradley executed and delivered a negotiable note and a mortgage securing the note to Monmouth, which was designated as payee of the note and the mortgage. The mortgage instrument contained a "due on sale" clause. After acquiring title to the land, Bradley built a luxurious beachfront home overlooking the Atlantic. He lived there until May 2000 when he decided to sell the property and the house to Sidney Springsteen. The deed of conveyance from Beachbaum to Springsteen provided that the buyer acquired Seaside Heights "subject to the mortgage" with Monmouth Bank. Springsteen paid Beachbaum the agreed purchase price and took possession of the property. Following the sale of the property to Springsteen, Beachbaum did not pay off the mortgage principal to Monmouth but continued to make his monthly interest payments to the mortgagee. Six months later, Beachbaum defaulted on the mortgage. Monmouth has recently instituted a foreclosure action on Seaside and joined Springsteen as a party-defendant. 176. Springsteen seeks your legal advice on how best to proceed. You should advise him that (A) Monmouth may foreclose on Seaside Heights but may not sue him in personam because he took the property "subject to the mortgage." (B) Monmouth may sue Beachbaum in personam because he breached the "due on sale" provision of the deed but the bank may not foreclose on the property. (C) Monmouth may either foreclose on Seaside Heights or sue Springsteen in personam because Springsteen is in privity of estate with Beachbaum. (D) Since Beachbaum's conveyance to Springsteen violated the "due on sale" clause, Monmouth may fore¬close on Seaside Heights; and if the proceeds from the foreclosure sale are insufficient to pay off the mortgage debt, Monmouth may sue insufficient to pay off the mortgage debt, Monmouth may sue either Springsteen or Beachbaum for the deficiency.

(A) As noted previously, mortgages remain one of the "hottest" areas tested on the MBE. This is a rather simple mortgage question because the facts indicate that Springsteen acquired the property "subject to the mortgage." Consequently, choices (C) and (D) are wrong because Springsteen is not personally liable for the mortgage debt. Choice (B) is likewise wrong because the bank may foreclose on the property after the mortgagor has defaulted on his/her mortgage payments. By process of elimination, choice (A) is correct. In analyzing this question, there are two "red her¬rings" intended to distract students. First, the mortgage contained a "due on sale" clause which is an irrelevant side issue since the bank evidently waived it by continuing to accept Beachbaum's monthly payments after the property was sold to Springsteen. Second, Beachbaum is continuing to make mortgage payments to the bank even after the conveyance. Although this is atypical, keep in mind the original mortgagor remains obligated to the bank for the mortgage until it is paid off.

Question 194 is based on the following fact situation. O'Malley was the undisputed owner of Blueacre, a tract of farmland located in Chavez Ravine and flanked on its eastern border by the Reese River. In 1990, O'Malley died a widow¬er and by his will, which was duly admitted to probate, specifically devised Blueacre as follows: "To my son, Tommy, his heirs and assigns, but if Tommy dies survived by a wife and a child or children, then to Tommy's wife during her life¬time with remainder to Tommy's children, their heirs and as¬signs." Another provision of the will specifically provided that "in the event that Tommy dies survived by a wife and no child, then Blueacre is to be devised to my nephew, Gibson, his heirs and assigns." While O'Malley's will was still in probate, Gibson quit¬claimed all his interest in Blueacre to Tommy's wife, Linquinia. On January 14, 1994, Tommy died from botulism and was survived by Linquinia but no children. Tommy left a will in which he devised his interest in Blueacre to his wife, Linquinia. Note: this jurisdiction has an applicable modern law statute which provides that any interest in land is freely alienable. 194. Gibson instituted an appropriate action against Linquinia to establish title to Blueacre. Judgment should be for (A) Linquinia, because Gibson had effectively conveyed his interest in Blueacre to Linquinia (B) Linquinia, because the doctrine of estoppel by deed applies to a devise by will (C) Gibson, because his quitclaim deed did not transfer his after-acquired title (D) Gibson, because Linquinia took nothing under O'Malley's will

(A) Based on O'Malley's devise, Gibson has a contingent remainder interest. A contingent remainder is a remainder which is (a) created in favor of an unborn or unascertained person or (b) otherwise subject to a condition precedent. According to the given facts, Gibson quitclaimed his interest in Blueacre to Linquinia. The issue presented is whether the contingent remainder interest is transferable. The facts state that this jurisdiction has an applicable statute which provides that any interest in land is freely alienable. Therefore, choice (A) is correct because based on the relevant statute Gibson's contingent remainder interest was transferable.

. Assume for the purposes of this question only that Shelley's Rule has been abolished by statute in this jurisdiction. Thus, John's prayer for specific performance would now be (A) denied, since the Rule would not be triggered, thus creating only a life estate in John (B) granted, since the remainder in his heirs would become vested into a full fee in those heirs (C) granted, since John's heirs receive a vested indefeasible interest in Blackacre (D) denied, since under the Doctrine of Worthier Title, at the termination of John's life estate, the grantor has a reversionary interest

(A) Choice (A) is correct because since the Rule is abolished, John would only acquire a life estate. As a result, in order to convey marketable title John's heirs must join in the conveyance of Blackacre. The remainder to John's heirs is treated as a vested remainder which does not merge with John's life estate.

Penn is the owner of Sylvaniacre in fee simple absolute (the record title is also in Penn). Sylvaniacre is and has been unpossessed timberland. In 1960, Penn conveyed the property by general warranty deed as follows: "the grantor hereby conveys Sylvaniacre to my brother, William, for his life, then to the chidren of my sister, Sylvia, who reach the age of 25." At the time of the conveyance, Sylvia was single. Thereafter, Sylvia married and had two children, George and Scott. Subsequently, Scott died in an automobile accident. Three years after Scott's death, Sylvia had a third child Loretta. 156. In an appropriate action to determine the ownership rights to Sylvaniacre with all minors represented, title is in (A) William only (B) William, George and Loretta (C) William, George, Loretta and any additional children of Sylvia born during her lifetime (D) William, George, Loretta and any additional children of Sylvia born within 21 years after the death of Scott

(A) In brief, the Rule Against Perpetuities provides that "no interest (usually meaning, contingent remainders or executory interests) is good unless it must vest, if at all, not later than twenty-one years after some life in being at the creation for the interest." Remember that the rule is measured from the time the instrument takes effect, and the time for vesting is calculated accordingly. In the case of a deed, the period is measured from the date of delivery. In our case since Sylvia did not have any children at the time of the conveyance, the (contingent) remain¬der in favor of her children who reach the age of 25 violates the rule (because it might not vest within twenty-one years of Sylvia's death). To illustrate, assume that one year after the conveyance Sylvia gave birth to George. Sylvia then died two years later. Now twenty-three years after Sylvia's death, George reaches the age of twenty-five. Since this is more than twenty-one years after Sylvia's death, it violates the rule (which requires the vesting not later than twenty-one years after "some life in being"). Hence, no other lives (of Sylvia's children) can vest; there¬fore, choice (A) is correct.

33. After Simon failed to make his rental payments for the first two months of his lease, Harry brought suit against Tony to recover for the unpaid rent. Judgment for (A) Harry, since Tony's contractual obligation under the lease survived the assignments (B) Harry, since he did not object to the assignments (C) Tony, since Simon, as assignee, would only be held liable (D) Tony, since his assignment to Rick constituted a nova¬tion, thereby extinguishing his obligation to pay rent

(A) In reiteration, where a tenant-assignor transfers all of his leasehold interest to an assignee, such assignment does not release the tenant from his contract obligations to the landlord under the terms of the lease. This is true even when the assignee, thereafter, assigns/transfers his lease¬hold interest to a sub-assignee.

41. After McQuirk's assignment to Moore, Donaldson sues McQuirk to recover the rent in arrears. Donaldson will most likely: (A) succeed, because Donaldson and McQuirk stood in privity of contract (B) succeed, even though he had notice of the assignment (C) not succeed, because McQuirk had assigned his inter¬est in the premises before Donaldson brought suit (D) not succeed, because he did not object to the assignment

(A) Under the terms of their original leasehold agreement, McGuirk would remain liable for the rent in arrears. McGuirk's assignment to Moore would not operate as an extinguishment of his duty to pay rent to Donaldson under the terms of their original agreement. An assignment does not release the tenant from his contractual obligation to the landlord under the lease. To be sure a lease is a contract as well as a conveyance. Choice (B) is erroneous because Donaldson's notice (of the assignment) is irrelevant to the effectuation of Moore's obligations under the assignment. Choices (C) and (D) are erroneous for the above reasons.

Question 178 is based on the following fact situation. Grant was the owner of Redacre, a farm located in State Green. On June 5, Grant executed a deed conveying Redacre to my brother Ed for life, remainder to my sister, Beatrice. He gave the deed to Ed who recorded the conveyance on June 6. Unknown to both Grant and Ed, Beatrice had died on June 3 leaving Collin as her sole heir. On June 10, Grant died leaving all his real property to his son, Sam. 178. As of June 11, title to Redacre is best described as (A) in Ed, as a fee simple absolute (B) in Ed, for life, remainder to Sam (C) in Ed, for life, remainder to Collin (D) in Ed, for life, remainder to Grant's interstate heirs

(B) A remainder is vested if the holder of the remainder is (1) born (2) ascertainable and (3) there is no express condition precedent. Since Beatrice died 2 days before her remainder interest came into existence under the deed, she would fail to satisfy the first two requirements. Therefore, Beatrice has no interest in Redacre and cannot leave anything to Collin. Choice (C) is thus incorrect. Ed has a life estate which terminates upon his death. As a result, choice (A) is wrong. Grant would have a reversion which is devisable and would pass to Sam as a remainder interest. Consequently, choice (B) is correct. If Grant had not devised his interest to Sam, his intestate heirs would have taken Redacre (which would have been the (D) answer).

57. From the standpoint of Birch and Catskill, the drainage system would most likely be defined as a (an) (A) easement appurtenant (B) easement by implication (C) prescriptive easement (D) express easement

(B) An easement may arise by implication if the existence of the easement is strictly (or rea¬sonably) necessary for the beneficial use of the land. The creation of such an easement is based upon the presumed intent of the grantor and grantee. If the claim is made that the easement arose out of a prior conveyance, there must be proof of the fact that at one time both the dom¬inant and servient estates were under one ownership. Technically, alternative (A) is correct but choice (B) is the preferred answer because the present example is an illustration of an implied easement appurtenant. With respect to choice (C) an easement (or profit) may also be acquired by prescription in a manner similar to that by which ownership of a possessory estate may be acquired by adverse possession.

On May 2, 1972, Abigail Marcus entered into a leasehold agreement for a term of ten years with Ray Haynes, who intend¬ed to use Blue Bell Farm for the raising of cattle and hogs. Haynes immediately took possession of the farm and remained in possession as a tenant until November 1, 1979. Shortly after entering into possession of the farm, Haynes built thereon, at his own expense, a corn crib made of lumber which was fifteen feet wide and twenty feet long and set on loose bricks. At each corner of the corn crib, he set a wood post in the ground fifteen inches deep and nailed the corn crib to the four corner posts. In 1975, he also built a hog house of brick and mortar on the premises. The hog house was constructed on a cement foundation which extended into the ground twenty-four inches. This structure was twenty feet wide by fifty feet long and was used to keep the brood sows warm during the winter months. On October 1, 1979, The Farm Life Insurance Co. instituted foreclosure proceedings against Alvin and Abigail Marcus for default of the promissory note on Blue Bell Farm. When Haynes become aware of the foreclosure proceedings, he disengaged the corn crib from the four corner posts (which anchored it in the ground) and removed the corn crib and posts to another farm. Moreover, on October 15, 1979, Haynes dismantled the hog house, brick by brick. He loaded the bricks on his truck and hauled them away. 64. In an action by Abigail Marcus against Ray Haynes alleging waste for removal of certain fixtures (i.e., corn crib and hog house) which of the following is (are) correct? I. Haynes would be liable for removal of the corn crib. II. Haynes would be liable for removal of the hog house. (A) I only (B) II only (C) I and II (D) neither I nor II

(B) Haynes would be liable for removal of the hog house but not the corn crib. For an article of personal property to become a fixture on land, it is necessary that (1) there be an annexation of the article to the land either actually or constructively, (2) the article be adapted to the use for which the land is used, and (3) there be an intention on the part of the annexer that the arti¬cle become a fixture. With respect to the hog house, the facts clearly state that it was built of brick and mortar and affixed to a cement foundation which extended twenty-four inches into the soil. Thus, given the fact that the hog house was actually built into the soil and cannot be removed without lifting the cement foundation out of its resting place (and causing complete destruction of the hog house as a building) clearly indicates an intention to make it a fixture. Conversely, the fact that the corn crib can be removed from the property with practically no injury to the freehold would indicate that there would be little economic waste in its removal. Therefore, it seems proper to permit Haynes to remove the corn crib without being liable for waste.

Questions 162-165 are based on the following situation. Meadowview is a large tract of undeveloped land located in the state of Eureka. Candlestick Housing Corporation (hereinafter referred to as Candlestick), the owner of Meadowview, prepared a development plan creating a suburban residential community known as Brunswick on the Meadowview tract. Set out below is the Declaration of Covenants that has been duly recorded for the Brunswick development scheme. DECLARATION Whereas, the Candlestick Housing Corporation, sometimes referred to herein as Candlestick, a corporation organized under the laws of the state of Eureka, is the owner of Meadowview, parts of which Candlestick proposes to develop and improve, in accordance with an harmonious plan for the design and relative location of single-family dwellings, garages, stores, streets, walks, playgrounds, and other structures, so as to create a community to be known as "Brunswick" providing the greatest possible degree of health, safety, architectural beauty and amenity for the property owners and inhabitants thereof: and Whereas, Candlestick being about to sell and convey lots and buildings located in certain parts of the said tracts, more particularly hereinafter described, desires to assure to said purchasers and their several heirs and assigns owning such lots and buildings, and their tenants, employees and the inhabitants within said property, the use, benefit and enjoyment of the common amenities, facilities and utilities in accordance with the said harmonious plan, and desires to provide for the maintenance of such amenities, facilities and utilities, and to this end desires that certain parts of its lands may be subject to certain restrictions, reservations, servitudes, covenants, agreements, easements, liens and charges, as hereinafter set forth; and Whereas, for the more efficient protection of the community and control thereof by the inhabitants, Candlestick has deemed it desirable to provide an agency or instrumentality to which should be delegated and assigned the powers of maintaining and administering the community amenities, facilities and utilities aforesaid, and administering and enforcing the restrictions upon or affecting the said lands or portions thereof, and collecting and disbursing such charges hereinafter created; and Whereas, there has been incorporated under the laws of the state of Eureka, as an association not for pecuniary profit, the Brunswick Association, for the purpose among others of exercising the functions aforesaid. Now therefore, in consideration of the premises, Candlestick hereby covenants and agrees with the purchasers of plots or buildings thereon and each of them and with The Brunswick Association, which is hereby made a party hereto, that the property described in and referred to in this Declaration shall be held and shall be conveyed subject to the restrictions, reservations, covenants, conditions, servitudes, easements, agreements, liens and charges set forth in the various articles and clauses of this Declaration, to wit: ARTICLE I Section 1. Candlestick (for each parcel of property subject to this Declaration owned by it) hereby covenants, and each purchaser of any parcel of the property (whether purchased from Candlestick, or another) by the acceptance of a deed therefore, shall, whether or not it shall be so expressed in any such deed or other conveyance, be deemed to covenant and agree to pay The Brunswick Association an annual assessment or charge to be fixed, established and collected from time to time as hereinafter provided, for the creation of a Fund to be known as the "Brunswick Fund", each installment of which annual assessment or charge when due shall become a lien upon the parcel of the property against or on account of the ownership of which such assessment or charge is made. Section 2. Such charge shall be in an amount to be fixed from year to year by the Board of Trustees of The Brunswick Association, and may be determined upon the basis of the valuation of each portion of the property and the improvements thereon as fixed by the assessing authority of the municipality in which such portion is located, or upon the basis of the area thereof in square feet, or by any other measure determined by The Brunswick Association to be fair and equitable. Section 3. The monies collected by virtue of the charges or assessments or the lien created by this Article shall be paid to The Brunswick Association to be used in such manner and to such extent as the Board of Trustees of The Brunswick Association may determine to be for the benefit of the residents of the community and for the promotion of the health, safety and welfare of the property owners and inhabitants thereof. ARTICLE 2 Section 1. There shall never at any time be erected, permitted or maintained upon any part of the property any structure designed for or used as a saloon or place for the sale or manu¬facture of malt, vinous, or spirituous liquors; any cattle yard, slaughterhouse, hog pen, poultry yard, or stable (not including garage); any cemetery or crematory or any house of detention, reform school, asylum, or institution of kindred character; any building for the manufacture or storage of gunpowder or explosives; nor any trade, business, or use of the property (whether similar to the foregoing or not) which shall be determined by The Brunswick Association to be dangerous, or noxious or offensive because of the emission of odor, gas, dust, smoke (other than coal smoke) or noise, or unsuitable for the locality. Section 2. As to any building site in the property upon which Candlestick shall in the first instance prior to the sale thereof cause a building or structure to be erected, such building or structure shall not be used for any purpose other than that for which the original structure was designed without the approval in writing of The Brunswick Association; provided that in no event shall any dwelling designed for occupancy by a single family be used or occupied as a dwelling for more than a single family for which it was originally designed; and provided further that no trade, commerce or business shall at any time be con¬ducted in any part of a building designed as a dwelling house. Section 3. No profession or home industry shall be conducted in any part of a building designed as a dwelling house except upon the approval in writing of The Brunswick Association. ARTICLE 3 Section 1. All of the restrictions set forth or provided for in this Declaration shall be deemed covenants running with the land, and any and every conveyance of any part of the property shall be absolutely subject to the said restrictions whether or not it shall be so expressed in the deed or other conveyance thereof. ARTICLE 4 Section 1. The Brunswick Association hereby accepts each of the duties and obligations imposed on it by this Declaration sub¬ject to all the terms and provisions herein contained. Section 2. The Brunswick Association may assign any or all of the rights, powers, titles, easements and estates granted to or conferred upon it by the Declaration to any one or more corporations or associations, municipal or private, that will agree to assume such interests. The assignment is to become effective on filing it with the recording officer. ARTICLE 5 Section 1. Violation of any restriction contained or provided for in this Declaration shall give The Brunswick Association, in addition to all other remedies, the right to enter upon the land and summarily abate or remove, using such force as may reasonably be necessary, at the expense of the owner thereof, any erection, thing, or condition that may be or exist thereon contrary to the intent and meaning of the provisions hereof; and neither The Brunswick Association nor its officers, agents, or employees shall be deemed guilty or liable for any manner of trespass for such entry, abatement or removal. 162. Under Article 4, Section 2 of the Declaration, The Bruns¬wick Association assigned "all the rights, powers, titles, easements and estates granted to or conferred upon it by the Declaration" to a municipal corporation, the Borough of Maplewood. The Association was then terminated. Lang, the owner of lot 18 in the Brunswick development, proposes to convert his single family dwelling into a mas¬sage parlor. The Borough of Maplewood asserts an action against Lang to recover money damages. Which of the fol¬lowing is the best argument for Lang? (A) The restraint on alienation of his land is invalid. (B) The city is not in privity of estate with The Brunswick Association. (C) The benefit is in gross, hence the burden cannot run. (D) The burden is in gross, hence the benefit cannot run.

(B) In order for a covenant to run with the land, the following four requirements must be met: (1) there must be a covenant; (2) there must be an intention that the covenant shall run with the land; (3) the covenant must be of a type which "touches and concerns" the land; and (4) there must be privity of estate. With respect to the privity requirements, Smith and Boyer state that "where the running of covenants is allowed the cases seem to agree that if there is (i) privity of estate between the original covenanting parties and (ii) simultaneous ownership by the covenanting parties in the same land and (iii) privity of estate between either of the covenanting parties and his assignee, the covenant will run with the land." In our case, choice (B) is correct because under the terms of the Declaration, there is privity of estate between the developer (Candlestick Housing Corporation) and the purchasers of the lots (e.g., Lang). However, there doesn't appear to be any privity of estate, either mutual or successive, between Candlestick and the Brunswick Association. Candlestick simply owns Meadowview and the Brunswick Association has no interest therein either as a co-owner or as a conveyee of an interest therein. Between Candlestick and the Brunswick Association there is simply the covenant (i.e., Declaration) which constitutes privity of contract. Therefore, there is no successive privity of estate between the Brunswick Association and the city of Maplewood because the latter only succeeds to the interest or estate which the Association has in the land.

65. Assume for the purposes of this question only that The Farm Life Insurance Co. obtained a foreclosure decree against the Marcuses. Both Farm Life Insurance Co. and Abigail Marcus bring separate actions against Ray Haynes alleging waste for removal of trade fixtures. If Farm Life Insurance Co. prevails, it will be because (A) Haynes' removal of the corn crib and hog house constituted unpermissive waste (B) the claim of the mortgagee would take priority over the claim of the mortgagor to those fixtures which were annexed to the land (C) the claim of the mortgagee would take priority over the claim of the mortgagor, only if Haynes was aware of the existence of the mortgage at the time he entered into the leasehold agreement (D) the mortgagee would acquire a security interest to the chattel property, even though the latter did not become fixtures

(B) When land has been mortgaged and thereafter a chattel is attached to the land and becomes a fixture, the claim of the mortgagee takes priority over the claim of the mortgagor. Moreover, if a tenant of the mortgagor adds a chattel, as a fixture of the realty, it inures to the benefit of the mortgagee of the real property. As a result, Farm Life Insurance Co. would prevail over Abigail (the mortgagor), since the removal of the hog house would constitute waste. It is important to note that trade fixtures are chattels which are annexed to the land by a tenant for the purpose and convenience of his trade or business on the land. Usually this doctrine is applied to factory machinery, store and shop equipment or temporary partitions in industrial establishments. However, difficulty exists when this doctrine is applied to buildings or to additional extensions to the buildings already present on the land.

Question 157 is based on the following fact situation. In 1969, the city of Rehobeth installed a sewer line across a residential subdivision, known as the Oceanview Development. The city acquired a valid easement for construction of the sewer from the Oceanview Development Company, the owner of the subdivision. In 1970, Mike Marshall purchased a house within the subdivision. Unknown to Marshall, the sewer line ran across his property, approximately 10 feet beneath the foundation of his home, the deed which Marshall received from the grantor, Oceanview Development Company, made no mention of the easement. In 1980, a crack in the sewer line caused water to leak into Marshall's basement. The flooding resulted in extensive damage to his home. The city of Rehobeth has abolished governmental immunity. 157. In an appropriate action by Marshall against the city of Rehobeth to recover damages, the plaintiff will probably (A) not prevail, because the sewer line was installed before Marshall purchased the property (B) not prevail, because the city had acquired a valid easement for the sewer line (C) prevail, only if the sewer line was negligently maintained (D) prevail, because under the circumstances the sewer line constituted a public nuisance

(C) A holder of an easement in gross may make reasonable use of the servient tenement so long as his/her conduct does not unreasonably interfere with the use of the servient tenement owner. As such, by process of elimination, choice (C) is most accurate.

On January 1, 1979 Barnes entered into an agreement of sale to convey his farm, Winterthur, to Stevens. According to the con¬tract, settlement (or closing) was to take place on March 1, 1979. On January 15, 1979, Stevens conducted a title search on Winterthur and discovered the following transactions in Barnes' chain of title: In 1930, Smith, the record owner of Winterthur died and in his will he devised Winterthur to "my son, Smitty, and his heirs but if Smitty dies without issue to my daughter, Dixie, and her heirs." The next instrument of record is a deed from Smitty to "Bailey and his heirs" which was executed on July 1, 1940 and recorded on the same date. In 1941 Bailey died and in his will he devised Winterthur to "my brother Corley for life, then to Corley's heirs." In 1950 Corley executed a general warranty deed in which he conveyed "all my right, title and interest in Winterthur to my children, Jessie and James." James died intestate in 1955, leaving his son Butch, a four year-old as his only heir. Corley died in 1960 leav¬ing Butch and Jessie as heirs. In 1965 Butch and Jessie conveyed Winterthur by a general warranty deed to "Barnes and his heirs." Although both Butch and Jessie were married, their respectives wives did not join in the deed. Barnes has been in continuous possession of Winterthur since 1965. Smitty died on January 1, 1976 without issue and Dixie is still alive. 66. After Smith's death in 1930, Dixie's interest in Winterthur may best be described as a (an) (A) contingent remainder (B) springing executory interest (C) shifting executory interest (D) vested remainder, subject to complete divestiture

(C) After Smith's death in 1930, Dixie's interest in Winterthur could best be described as a shifting executory interest (or executory devise as it was created by will), which divests the preceding estate upon the happening or non-happening of a stated event. Executory devises are interests which are identical with springing and shifting uses, except that they are created by will, instead of by deed. Thus, in 1930 Smith devised a fee simple to his son, Smitty, and his heirs, which was subject to Dixie's executory interest. If the event happens (Smitty dies without issue) then Dixie will have a possessory estate in fee.

. Assume for the purposes of this question only that Marino does not want to return to the apartment after moving to Philadelphia. Pieper now asserts a claim against Marino to recover damages for breach of leasehold contract. Marino's best defense is that Pieper by permitting Speck to store materials on the third floor caused a (A) partial constructive eviction (B) partial actual eviction (C) constructive eviction (D) breach of implied covenant of habitability

(C) As a general rule, liability for rent is not affected by the conduct of a lessor (or third per¬son) which merely interferes with the lessee's enjoyment of the leased land. But if the disturbance is substantial in nature the lessee may give up possession of the land and thereby avoid further liability under the lease. His defense to a rent action is constructive eviction. In this situation if the lessee sees fit to give up possession of the leased land he thereby avoids liability for rent that becomes payable after that date.

Question 90 is based on the following fact situation. Bradley was the record owner in fee simple absolute of Greenacre, a 125-acre parcel of land located in Oxford. In 1992 Bradley conveyed the property "to my sister, Martina, for life with remainder to my son, Reed, if he be living." At the time of the conveyance, Reed was in medical school in Grenada. Martina immediately recorded the deed and took possession. Shortly thereafter, Martina discovered that the property con¬tained large coal deposits which she severed and began to sell. In 1995 Reed graduated from medical school and returned to Oxford. He then learned of the conveyance and also ascertained that Martina had not paid taxes on Greenacre for the last three years. After discovering that the property was subject to a pend¬ing tax foreclosure, Reed demanded that Martina pay the delin¬quent taxes. Even though the profits from the coal sales were quite substantial, Martina refused to pay the outstanding taxes. Reed thus paid the taxes himself. 90. If Reed sues Martina to recover the taxes and for an accounting of the proceeds received from the coal sales, judgment should be (A) in favor of Reed for the taxes but not for the coal (B) in favor of Reed for the coal but not for the taxes (C) in favor of Reed for both the taxes and the coal (D) against Reed for both the taxes and the coal

(C) Before attempting to answer this question, students must first determine what interest, if any, Reed has in Greenacre. As we know, Bradley (the grantor) conveyed the property "to Martina for life with remainder to Reed, if he be living." Does Reed take a vested or contingent remainder under this conveyance? The answer is that Reed takes a vested remainder. A vested remainder is one which is limited to an ascertained person who has the right to immediate possession if and when the prior estate is terminated. It may be subject to no other condition, in which case it is said to be indefeasibly or absolutely vested. In that case it is certain to become an estate in possession. But it may (as in this example) be subject to a condition sub¬sequent (i.e., Reed's surviving Martina) which will divest the remainderman of his estate. But it is, nevertheless, vested until divested. On the other hand, a contingent remainder is one which is subject to a condition precedent. It is one which will not vest until the happening of an event or the ascertainment of a person. In our case, Reed has a vested remainder although it may be totally divested since it is subject to the condition subsequent that he be living at the termination of Martina's prior life estate. Note that a tenant for life or for years is liable to the (vested) remainderman or reversioner for voluntary waste, therefore Martina is liable to Reed for an accounting of the proceeds from the coal sales. In addition, a vested remainderman has a right to compel the prior estate owner to pay taxes and interest on encumbrances to the extent of the value of rents and profits. Consequently, choice (C) is correct.

131. In an action by Dave to enjoin Johnson's use of the road in transporting sand, the court will most likely (A) issue an injunction against Johnson to prevent the further use of the road to haul sand (B) refuse to grant Dave's prayer for relief since the servient owner continues to have the right to use his own land (C) issue an injunction against Johnson unless it can be shown that Johnson's use did not unreasonably interfere with Dave's right of access (D) dismiss the cause of action because Dave's only remedy would be monetary damages

(C) Choice (C) is correct because although the owner of the servient tenement remains the owner of the land subject to the easement he has granted, he may only make such use of the land as long as he does not unduly or unreasonably interfere with the rights created for the dominant owner.

68. On January 20, 1979, a barn on the Winterthur property was destroyed by fire. When Stevens was notified of the loss the following day, he contacted Barnes and told him that the deal was off because of the fire. As a result, Stevens filed suit against Barnes on January 25, 1979 for a refund of his downpayment of $5,000. In his suit, Stevens will be unsuccessful and will be obliged to complete the contract if the court determines that on January 20th, (A) Stevens held the legal title to the farm (B) Barnes held the legal title to the farm (C) Stevens held the equitable title to the farm (D) Barnes held the equitable title to the farm

(C) Equitable conversion applies when there is an enforceable obligation to sell land. The doc¬trine treats interests in land as if the land had already been converted into personal property. The doctrine states that when the sales contract is made, equity then considers the vendee as the owner of the land and the vendor as the owner of the purchase money. Applying the facts of the case to this doctrine, Stevens would be construed as the equitable owner of the land on January 20, 1979, when the barn was destroyed by fire. As a result, Stevens as equitable owner, would become subject to all losses not occasioned by the fault of the seller. Therefore, if the court determined that Stevens held equitable title to Winterthur on January 20th, he would be unsuccessful in his lawsuit against Barnes.

67. In March 1976, Dixie, asserting that her title was held free of any claim by Barnes, instituted suit against Barnes to quiet title to Winterthur. Judgment for (A) Barnes, since his prior recorded deed would be deemed to be outside Dixie's chain of title (B) Barnes, since the devise to Dixie would be violative of the Rule against Perpetuities (C) Dixie, since she acquired fee simple interest at the time of Smitty's death in January 1976 (D) Dixie, since she acquired an indefeasible vested remainder under her father's will in 1930

(C) In a suit to quiet title to Winterthur, Dixie would prevail. At the time of Smitty's death on January 1, 1976, (without issue) Dixie's executory interest was executed into a legal estate in fee. In fact, Dixie's interest vested automatically upon Smitty's death without issue, without any affirmative act on her part.

Smith's will would devise Monicacre "to my friend, Monte, to whom I am everlastingly grateful for the devoted care he has lavished on my horses, but if ever my horses who survive me shall fail to receive proper care, then to my beloved daughter, Doris and her heirs, if she be living and own any horses otherwise to the Malibu Equestrian Society." 160. In an appropriate action to construe the will, the court will determine Monte's interest to be a (an) (A) fee simple determinable (B) fee simple subject to condition subsequent (C) fee simple subject to an executory interest (D) contingent remainder

(C) It is important to point out that Doris has an executory devise. The characteristics of this interest are exactly the same as attach to springing and shifting uses but the devise, of course, is created by will and not by deed. It is obvious that Doris' interest could not be a remainder for two reasons: (1) a remainder cannot follow a fee simple and (2) a remainder cannot come in and cut short the preceding vested estate. In this case, Monte has a fee simple subject to an executory devise in Doris.

Question 153 is based on the following fact situation. A, being fee simple owner of Greenacre, devised it "to B and C as tenants in common." Thereafter, B died intestate leaving daughter, D, as his only surviving heir. While B's estate was in administration, C agreed to convey his interest in Greenacre to D. 153. Which of the following would be a valid conveyance by C: (A) to the estate of B and its successors and assigns (B) to the heirs and assigns of B (C) to D and her heirs and assigns (D) to B as a former tenant in common with remainder to D

(C) Many students do poorly on the Multistate because they "out-psych" themselves into thinking that every MBE question involves some "trick". This is not true. Sometimes the simple, obvious answer is correct. Here, for example, the facts state that "C agreed to convey his interest in Greenacre to D". The question then asked is, "Which of the following would be a valid conveyance by C?" Obviously, the correct answer is "to D and her heirs and assigns." It is important to note that C's intention was to convey his interest in Greenacre to D not to the estate of B.

Pieper owned an old brownstone building in a rundown area in Greenwich Village, New York. This was the only piece of real estate owned by Pieper. The three-story building had a store on the ground floor and apartments on the other two floors. Pieper entered into a leasehold agreement for a three-year term with Hatsuhanna, who planned to open a Japanese Sushi restaurant on the ground floor. After identifying the parties, the operative words of the lease were "landlord hereby agrees to lease for the three years the first floor of his building in Greenwich Village to tenant reserving unto said landlord annual rental in the sum of $12,000 payable in advance in monthly installments of $1,000." Hatsuhanna took possession of the first floor of the building and paid rent for four months which Pieper accepted. Shortly thereafter, a deadly uncinariasis (or hookworm) epidemic swept through New York City killing several hundred people. A study by the New York Medical Examiner's Office concluded that the parasites were transmitted from raw fish which people had eaten. As a result, the New York City Council passed an ordinance prohibiting all Sushi restaurants from operating in the city. The day after the ordinance went into effect Hatsuhanna placed a sign in the window "Gone Fishing—Closed Until Further Notice." During the next couple of weeks, rats were attracted by the odor of fish from the restaurant, infesting both Pieper's second-floor apartment and the apartment of Marino, who lived on the third floor. Marino had the same lease as Hatsuhanna except that no description of the leased premises was contained in it. The original three-year term had long since expired, but Marino had continued to pay his rent and Pieper had accepted it. After the rat infestation, Marino served notice on Pieper that he was moving to Philadelphia and has not been heard from since. Pieper exterminated the rats and on November 1, 1983 by a formally valid lease rented the first floor to Speck who began to operate a bar review center. With Pieper's permission, Speck stored his bar review materials on the third floor. However, on November 17,1983 Hatsuhanna returned late in the evening (when Peiper wasn't there) and entered the premises with his key. Hatushanna proceeded to change the locks on the door and began installing equipment for a massage parlor which he planned to open. 170. At the moment of signing, the leasehold agreement entered into between Pieper and Hatsuhanna probably (A) created a periodic tenancy (B) could be terminated at will by either party (C) did not convey to Hatsuhanna a term of years at law (D) was void, invalid and had no legal effect

(C) The key to your success on the MBE will be your ability to eliminate the incorrect choices. In this regard, if you are able to eliminate two answers as wrong, you will increase your odds by 50% of choosing the correct answer. In this classic Multistate example, the only way you will be able to arrive at the correct answer is by process of elimination. First, choice (A) is wrong because a periodic tenancy (or an estate from period to period) is an estate which continues from year to year (or month to month) indefinitely until terminated by proper notice. A periodic tenancy is usually created by a tenant's holding over (after the expiration of the leasehold period) and then the landlord's accepting rent on the same terms as provided in the prior lease. Note: the essential characteristic of the periodic tenancy is that it is of indefinite duration whereas the lease for years is for a definite or specific term. Secondly, choice (B) is incorrect because a tenancy at will is always of indeterminate duration because it can be terminated by either the landlord or the tenant. Since the facts indicate that the lease between Pieper and Hatsuhanna was for three years, it did not create a tenancy at will. Thirdly, choice (D) is not correct because the description of the building property in the lease is adequate to identify it. Here, the facts indicate the brownstone building was the only piece of real estate owned by Pieper. Thus, the description of the building in the leasehold agreement is sufficient to identify it (even though it did not specify the exact street location or number). As a consequence, by process of elimination the correct answer is choice (C) i.e., the leasehold agreement did not convey to Hatsuhanna a term of years at law. Here, students are being tested on the following (rather obscure) common law rule: At the moment a leasehold contract is entered into (but before the tenant actually takes possession), he is deemed to have an interesse termini (or equitable) inter¬est in the leased premises. Once, however, the tenant takes possession and pays part of the rental, then his equitable interest vests into a term of years at law. That's why choice (C) is correct because at the moment of signing, the leasehold contract entered into between Pieper and Hatsuhanna did not convey a term of years at law. Rather, it created a term of years in equity, or an interesse termini interest.

113. Assume for the purposes of this question only that all of the Sunnyside development deeds contained the following provision: "No lot shall be used except for residential purposes." By 1978, the area surrounding Sunnyside was rezoned for commercial and business uses. One of the lot owners now decides to operate a small beauty parlor in the basement of her home. In an action by the home¬owners in Sunnyside to prevent such commercial use, the court will most likely hold that (A) the residency restriction is no longer enforceable, because of the change in the character of the neighborhood surrounding the development (B) the residency restriction is no longer enforceable, since the area surrounding the development was rezoned for commercial use (C) the residency restriction is enforceable, thus preventing any commercial use (D) the operation of a beauty parlor would not constitute a violation of the residency restriction

(C) The restriction "except for residential purposes" should be interpreted to prohibit the operation of a beauty parlor in defendant's home. Such commercial activity clearly violates the plan and obvious purpose of the covenant and is prohibited by its terms. Choices (A) and (B) are incorrect since the change in character of the surrounding neighborhood would not justify failure to enforce the restriction where the original purpose of the restriction could still be realized. Choice (D) is wrong because the operation of a beauty parlor in defendant's home would constitute a violation of the restrictive covenant so as to justify injunctive relief.

58. Assume for the purposes of this question only that Catskill's land began to subside over the drainage system. This was because of an especially pliable soil condition of which Ash was unaware prior to construction of the drainage system. In an action brought by Catskill against Ash to recover prop¬erty damage for the subsidence of his land, judgment for whom? (A) Catskill, because Catskill is entitled to lateral support for his land in its natural condition. (B) Catskill, because Catskill is entitled to subjacent sup¬port for his land in its natural condition. (C) Catskill, because Catskill is entitled to support for his land in its improved condition. (D) Ash, unless Catskill proves that Ash was negligent in constructing the drainage system.

(D) The right to subjacent support means support from underneath the surface of the land as distinguished from the sides, but the rights involved are substantially the same as those involved in lateral support. Thus, the right to lateral (and subjacent) support does not include the right to have the additional weight of artificial structures on the land held up by the neighboring land. However, if there is negligence on the part of the wrongdoer who removes lateral or subjacent support, then the wrongdoer is liable for all damages which naturally and proximately result. In this hypo, since Catskill's land was in an improved state with a dwelling erected on the land, he must show that Ash acted negligently.

Carson is the owner of a parcel of land known as Tall Acres which is situated upon the top of Candy Rock Mountain. Located below Tall Acres is Grasslands, a forty acre hillside estate which is owned by DuVall. Crystal Stream is a nonnav-igable watercourse that originates at the top of Candy Rock Mountain and runs all the way down into the San Vicente Val-ley. Both Tall Acres and Grasslands are within the watershed of Crystal Stream. When DuVall purchased Grasslands in 1956, he started tak¬ing water from Crystal Stream and used it to irrigate the south¬ern half of his property which he has used as a farm. Prior to 1956, the southern half of Grasslands had been cleared and placed in cultivation, while the northern half remained wood¬ed and unused except for an occasional hike or gathering of timber for use as domestic fuel. DuVall continued this estab-lished pattern of use. Now (January, 1992), he is still taking water from Crystal Stream and using it to irrigate the southern half of Grasslands. In 1990, Carson built a home on Tall Acres and started tak¬ing water from Crystal Stream for domestic purposes. During that year there was heavy rainfall, and this caused Crystal Stream to run down the mountain at a high water level. The next year, however, there was a drought. As a result, Crystal Stream flowed at a very low level. Consequently, there was only enough water to irrigate DuVall's farmland or, in the al¬ternative, to supply all of Carson's domestic water needs and one quarter of DuVall's irrigation requirements. Candy Rock Mountain is located in a jurisdiction where the period of pre¬scription is fifteen years. 175. Inasmuch as Crystal Stream is still flowing at a very low level and Carson is continuing to take water for his personal needs, there is insufficient water to irrigate Grass¬lands. As a consequence, DuVall brings an appropriate action to declare that his water rights to the stream are superior to those of Carson. In addition DuVall moves to have the full flow of Crystal Stream passed to him, notwithstanding the effect it might have on Carson. If this state follows the doctrine of prior appropriation, judgment for whom? (A) Carson, because as an upstream landowner, he would have superior rights to the water than a downstream owner. (B) Carson, because domestic use is superior to and is protected against an agricultural use. (C) DuVall, because he has obtained an easement by prescription to remove as much water as he may need. (D) DuVall, because he has put the water to a beneficial use prior to Carson's use and has continuously used the water.

(D) According to the prior appropriation doctrine, "first in time is first in right." In contradiction, under the riparian rights doctrine the use of water for natural purposes is paramount and takes precedence over use of water for artificial purposes.

Assume that the provision(s) contained in the deeds for the thirty-four lots sold between 1967 and 1972 were enforce¬able in equity. Such restrictions would most likely be construed as an example of a (an) (A) equitable easement (B) negative easement (C) covenant running with the land (D) equitable servitude

(D) An equitable servitude is a restriction on the use of land which is enforceable in a court of equity. An equitable servitude is more than just a "covenant running with the land" because it is an interest in the land. The term, equitable servitude, is broader than an equitable easement because it applies not only to land but also to chattel property such as a business. An equitable servitude may be created by (a) an instrument which complies with the Statute of Frauds and (b) an intention that there be a restriction on the use of the land. Thus, the real basis for the enforcement of equitable servitudes is the doctrine of notice as recognized in equity courts to the effect that a person who takes land with notice of a restriction thereon cannot in equity be permitted to violate the restriction.

Questions 130-134 are based on the following fact situation. Dave owned a summer cottage on Lake Tioga. In order to facilitate the access to the cottage, which is located on a knoll above the lake, Dave entered into an agreement with Johnson, an adjoining land owner, whereby Johnson granted Dave a right of way over a strip of land 30 feet in width and a quarter of a mile in length along the east margin of Johnson's property. Without notifying Johnson, Dave proceeded with his plan to improve the roadbed by having the road asphalted in order to make it more accessible for motor vehicle traffic. Several years later, Johnson started a sand business, which required him to do heavy hauling that subsequently destroyed the asphalted surface on the road. Ten years after Johnson started his sand business, Dave sold his lakefront property to Wilson. Shortly after Wilson took possession of the cottage and property, Johnson erected wooden barriers across the roadway, thus obstructing the access to Wilson's property. 130. The interest which Dave acquired in the access road may best be described as (A) an easement in gross (B) an easement implied by prior use (C) an easement by necessity (D) an express easement

(D) Choice (D) is correct because an express easement appurtenant was created by the written agreement between Johnson (owner of the servient tenement) and David (owner of the dominant tenement). Choice (A) is incorrect because an easement in gross is personal to the owner and the use is not connected in any way with the enjoyment of any other land. Choices (B) and (C) are incorrect because they are both implied easements viewed by the courts as arising by reason of necessity or prior use.

159. Tenant's statutory period for adverse possession began to run when (A) Ohner died (B) Tenant discontinued paying rent (C) Tenant's lease with Ohner expired (D) Tenant subjectively believed that he no longer had permission to possess Grassacre

(D) Remember that the statutory period on adverse possession begins to run when a cause of action accrues against the adverse possessor. Although both choices (B) and (D) are correct, the latter is the preferred answer because the subjective intent of the adverse possessor is all important. Certainly, adverse possession does not run each and every time a lessee discontinues paying rent. Rather, the adverse possessor must (in his own mind) consider his possession to be hostile and adverse, i.e., meaning a possession held against the whole world including the dispossessed owner, and that the possessor claims to be the owner.

After Marino took possession and Pieper accepted his rental payment for the third floor apartment, which of the following most accurately describes the legal relationship between the parties? (A) A tenancy at will was created. (B) A periodic tenancy from month to month was created. (C) A periodic tenancy from year to year was created. (D) Marino's equitable three-year term became a legal three-year term through part performance.

(D) Since Marino's lease was the same as Hatsuhanna's, he had an estate (or tenancy) for three years. An estate for years is an estate the maximum duration of which is measured by one or more years. At the time of the signing of the lease, the estate for years was not enforceable at law because the agreement failed to specify the exact beginning or commencement of lease¬hold period. Be advised that the most important requisite of an estate for years is that it must have a definite beginning and a definite ending (however, the ending can be inferred from the beginning). Once, however, Marino took possession, his estate began a legal three-year tenancy because a definite termination date could be measured. Furthermore, at common law an estate (for years) does not vest in the lessee until he takes possession of the land. Until that time he has a mere interesse termini (or equitable interest) in the term.

. Assume for the purposes of this question only that Marino's arguments in the previous question are rejected by the court. As a result, what remedy was available to Pieper before he accepted the first rental payment after the end of the initial three-year period: (A) eviction of Marino for non-payment of rent (B) recovery of the difference between the lease price and the fair market value of the leasehold for a year (C) recovery of double the rent pro rated on a monthly basis for the number of months Marino was on the premises beyond the three-year period (D) none of the above

(D) When a tenant rightfully in possession wrongfully remains in possession (holds over) after termination of the tenancy, he (or she) is called a tenant at sufferance. The tenant sufferance is not really a tenant at all since he (or she) is not holding with the permission of the landlord. The common law admitted no excuses for holding over. The tenant holding over was liable for another term regardless of extenuating circumstances. Choice (D) is therefore correct because the hold-over doctrine permitted the landlord to hold the hold-over tenant to another term. Here, since the original lease (between Pieper-Marino) was for a term of three years, under the hold-over doctrine Marino would be liable for another term.

Anderson owned Hillcrest, a seventy-acre tract, which was located in Coatsville. In 1989, Anderson sold fifteen acres of Hillcrest to Barrington. The deed of conveyance contained the following clause: "The parties hereby covenant that if the grantor, Anderson, proposes to sell any or all of the remaining fifty-five acres of Hillcrest during Barrington's lifetime, then the grantee, Barrington, shall have the right of first refusal to purchase said parcel on the same terms and conditions as proposed; and, in the alternative, if grantee , Barrington, proposes to sell any or all of the fifteen acres of his parcel during Anderson's lifetime, then Anderson shall have the reciprocal right of first refusal." In December, 1997, Barrington was approached by Carper who offered to purchase his fifteen acre parcel for $100,000. Barrington did not afford Anderson an opportunity to exercise his right of first refusal, and he went ahead and sold the proper¬ty to Carper. After Carper took possession, Anderson then learned about the sale. Anderson immediately brought suit against Barrington and Carper to enforce the right of first refusal in the deed. 114. Based on the facts as presented, Anderson will (A) win, because Barrington has a reciprocal right of first refusal (B) win, because Anderson's right of first refusal does not violate the Rule Against Perpetuities since it is limited to his lifetime (C) lose, because the rights of first refusal are unreasonable restraints on alienation (D) lose, because the rights of first refusal only relate to land that is not conveyed by deed

. (B) Be advised in this example the reciprocal rights of first refusal must "vest," if at all, during the respective lifetimes of Anderson and Barrington. As such, since Anderson is a measuring life, we will know within Anderson's lifetime whether he will exercise his option to purchase Barrington's land. Consequently, there is no violation of the rule. Multistate Nuance Chart: RULE AGAINST PERPETUITIES AS APPLIED TO OPTIONS TO PURCHASE LAND: OPTIONS CONTAINED IN A DEED INSTRUMENT R/P Applies OPTIONS CONTAINED IN A LEASEHOLD CONTRACT R/P Does Not Apply

Ash is the owner of record of a parcel of land designated as "A". It is the highest of the three neighboring properties on Big Rock Candy Mountain. Birch is the owner of parcel "B", which is situated lower than "A" on the mountainside. Catskill owns parcel "C" which lies below parcels "A" and "B" on the moun¬tain slope. In 1950 Ash, who owned all three parcels of land, construct¬ed a private drainage system. This system consisted of an under¬ground pipeline which extended across all three parcels of land. Sewage from parcel "A" drained through the system to a mu¬nicipal sewer which was located at the bottom of the mountain. In 1955, Ash sold parcel "B" to Birch. The following year, Catskill purchased parcel "C" from Ash. The deeds to Birch and Catskill did not contain any mention of Ash's drainage system. Located on each of the parcels are large ranch-style homes occupied by the respective owners. 56. Lately, the sewage drain has begun to emit an unpleasant odor across parcel "B". As a result, Birch sued Ash in tres¬pass praying for damages caused by the drainage system. Judgment for (A) Ash, since the deed to Birch did not contain any covenants restricting Ash's use of the land (B) Ash, since Birch's proper cause of action would be for nuisance, rather than trespass (C) Birch, because the drainage system was the proximate cause of plaintiff's damages as a matter of law (D) Birch, because Ash did not have the right to improve his own land in a manner which caused harm to another

. (B) Birch's proper cause of action should be based on nuisance, rather than trespass. The basic distinction which is now recognized is that trespass is an invasion of the plaintiff's interest in the exclusive possession of his land, whereas nuisance is an interference with his use and enjoy¬ment of it. In all likelihood, Ash has an implied easement for his sewer pipe from parcel "A" across parcels "B" and "C." It is generally agreed that in cases involving an implied reservation (or quasi-easement) in favor of the grantor, there must be reasonable necessity for the existence of the implied easement.

55. Assume for the purposes of this question only that Hunter claims the entire fee simple absolute title to Blackacre and brings an action against Trent to quiet title in himself. If the Commonwealth of Pennsylvania has an ordinary 20 years adverse possession statute, judgment for whom? (A) Trent, because one cotenant cannot acquire title by adverse possession against another. (B) Trent, because there is no evidence that Hunter has performed sufficient acts to constitute an ouster. (C) Hunter, because the acts of the parties indicate Trent's intention to renounce his ownership rights to Blackacre. (D) Hunter, because during the past 25 years Hunter has exercised the type of occupancy ordinarily considered sufficient to satisfy adverse possession requirements.

. (B) Under the unity of possession, each co-tenant is entitled to possess and enjoy the whole of the property subject to the equal right of his (or her) co-tenant. If one tenant wrongfully excludes another co-tenant from possession of the whole or any part of the whole of the prop¬erty, such conduct amounts to an ouster. Here, choice (B) is correct because there is no evi¬dence that Hunter was wrongfully excluding Trent from Blackacre. Note that choice (A) is wrong because Burby points out that "possession may become adverse if the co-tenants are excluded from possession (i.e., ouster) or if the co-tenants are specifically notified of the adverse claim or can be charged with notice because of the acts of the possessor." Real Property, pg. 232.

Questions 1-2 are based on the following fact situation. In 1995 Wilkes was the true and record owner of Blackacre, a 150 acre tract of undeveloped land in densely populated Lovette County, mostly wooded except for 5 acres of pasture in the northeast corner that Wilkes sometimes used for cattle-grazing in connection with his dairying operation on Creenacre, an adjoining tract of land. In that year officers of the Lovette Preservation League, a newly created, non-profit corporation devoted to conservation activities, sought to negotiate with Wilkes for the purchase of Blackacre. Wilkes declined to sell because, as he said, "that land has been owned by the Wilkes family for six generations and I couldn't let it out of the family now." Further discussions, however, led to Wilkes' executing and delivering to the League (in exchange for $100,000) the fol¬lowing instrument: "It is hereby agreed between the owner and the Lovette Preservation League (hereinafter "League") that from and after the date hereof (a) no entry shall be made, and no activity conducted, on Blackacre except as expressly permitted by such regulations as the League may from time to time adopt in the inter¬est of maximum enjoyment by the public of Blackacre's virgin, natural values consistent with due protection for those values; (b) agents and employees of the League shall at all times be allowed access to Blackacre as required for the pur¬pose of appraising the current conditions and needs of the land, and of discovering violations of the League's regulations; and (c) whenever so requested by the League, the owner shall initiate such actions at law and suits in equity, and shall file such com¬plaints in criminal trespass and other like proceedings, as the League may reasonably require to enforce its regulations." Wilkes continued his occasional pasturing of cattle on Blackacre until he died in 1998, leaving a will probated in that year, that devised Greenacre to his friend, Rambis. The will made no mention of Blackacre, and contained no residuary clause. Wilkes' closest surviving relative was (and is) his son, Jammal. In 1998, the Lovette Preservation League wound up its affairs and transferred all its assets and property holding to the Forum Audubon Club of Greater Lovette, Inc., another non¬profit conservationist organization. 1. In an appropriate action to construe the agreement between Wilkes and Lovette Preservation League, the court will determine provision (a) to be a (an) (A) easement (B) profit (C) covenant running with the land (D) negative covenant

1. (D) In order for a covenant to run with the land, four elements must co-exist: (1) there must be a writing; (2) there must be an intention that the covenant run with the land; (3) the covenant must "touch and concern" the land; and (4) there must be privity of estate. In the present example, no privity of estate exists between Wilkes and the Lovette Preservation League. As a con¬sequence, provision (a) cannot be viewed as a covenant running with the land. By process of elimination, choice (D) is therefore the best answer.

Quirk owned a four-story office building located in downtown El Paso. The building, named Quirk Towers, was old and badly in need of renovation. To finance the improvements, Quirk bor¬rowed $125,000 from his friend, Lama. As consideration for the loan, Quirk executed a promissory note for $1 25,000 payable to Lama in one year and secured by a mortgage on Quirk Towers. The mortgage was dated January 1, 1988, and was recorded January 2, 1988. Thereafter, on February 1, 1988, Quirk execut¬ed a deed absolute on Quirk Towers and named Lama as grantee. This deed, although absolute in form, was intended only as additional security for the payment of the debt. In order to make judicial foreclosure unnecessary and to eliminate the right to redeem, Quirk then delivered the deed to Uribe in escrow with instructions to deliver the deed to Lama if Quirk failed to pay his promissory note at maturity. On January 1, 1989, Quirk failed to pay the note when it came due. Thereupon, Uribe, in accordance with escrow instructions, delivered Quirk's deed on the office building to Lama, which he promptly and properly recorded. Two weeks later, Quirk tendered the $125,000 indebtedness to Lama. When Lama refused to accept it, Quirk brought an appropriate action to set aside and cancel the deed absolute and to permit the redemption of Quirk Towers from Lama. Conversely, Lama counterclaimed to quiet title and argued that the deed absolute was intended as an outright conveyance upon default. 13. The court should enter a judgment that will grant the relief sought by (A) Quirk, but only if Quirk can establish that the mortgage takes precedence over the deed absolute since it was executed earlier in time (B) Quirk, because the deed absolute did not extinguish his right of redemption (C) Lama, because the deed absolute effectuated an out¬right conveyance that extinguished the redemption interest sought to be retained by Quirk (D) Lama, because Quirk is estopped to deny the effect of the deed absolute in conjunction with the escrow arrangement

13. (B) One of the most popular Multistate testing areas deals with mortgages. The reason why mortgage problems are frequently tested on the exam is because the "general" bar review courses provide only a cursory review of this extremely important subject area. In this particu¬lar question, it has long been recognized in equity that a deed absolute intended for security, will in fact be construed as a mortgage. This is not really surprising when it is remembered that the traditional form of the mortgage was a conveyance subject to defeasance, and that the equi-ty of redemption was created by the equity court to protect the mortgagor after default. In order to preserve this equity of redemption various rules were formulated to prevent mortgages from limiting or clogging the equity of redemption. The most common example of such rules is the principle "once a mortgage always a mortgage." This, in effect, means that a mortgagee cannot circumscribe the mortgagor's right to redeem by disguising the transaction as an outright con¬veyance. In this example, the facts indicate that Quirk executed the deed absolute to Lama as additional security. Therefore, the deed will not extinguish Quirk's right of redemption since it (the deed absolute) will be construed as a mortgage and not an outright conveyance.

X, Y and Z are the owners of three contiguous lots in the City of Palm Grove. A downward slope exists from X's land to Z's land. X's and Y's lots were in an unimproved natural state. Z, how¬ever, had lived for ten years in a house which he had built on his property. In 1990, X planted trees and shrubbery on his land along the boundary of Y's lot. In 1991, Y, in preparation for building a house on his lot, care¬fully made an excavation eight feet deep for the purpose of building a basement. The west side of the excavation which was five feet east of the X-Y boundary, suddenly collapsed and a quantity of X's soil, his trees and shrubbery fell into the hole. Y hauled away the debris. Y completed construction of his house and macadamized an area for use as a driveway, without changing the former contours of the land. Shortly thereafter, Z began to make complaints to Y about the flooding of his basement, which he claimed had been previously free of water. 10. In an appropriate action by X against Y to recover for the damage to his land, judgment for whom? (A) Y, if he was conducting the excavation work non-negligently (B) Y, because he was under no duty to support X's land in its improved state (C) X, because a landowner is entitled to support of his land in its natural condition (D) X, because a landowner has an absolute right to have his land supported by the neighboring land

10. (C) One who by excavation or otherwise withdraws lateral support from her neighbor's land is liable for the injury done to such land in its natural condition, regardless of negligence. The common law right to lateral support is a right to the support of land in substantially its natural condition. It does not include the right to have the additional weight of artificial structures sup¬ported by the neighboring land. However, a defendant may be held liable for damage caused to structures if there is proof of negligence on his part. In the present example, X's land was essentially in its natural condition. As a result, choice (C) states the correct rule.

Question 102 is based on the following fact situation. A deed executed by A in 1992 conveyed Blackacre for a consideration of one dollar, receipt of which was acknowledged, "to B for life, but if liquor is ever sold on Blackacre, then to C and his heirs, and if for any reason the interest hereby conveyed to C is not valid, then I reserve the right to reenter Blackacre and take back my property." In 1994, B died before the wheat he had planted could be harvested. 102. Who is entitled to the proceeds of the crop? (A) B's heirs (B) C (C) A (D) divided equally between B's heirs and A

102. (A) For Multistate purposes, students are required to know that there are two types of crops: (1) fructus naturales, those which come from nature's bounty without the aid of man, such as trees, bushes, grasses and the fruits of these, and (2) fructus industriales, those which come pri¬marily from man's annual planting, cultivating and fertilizing, such as wheat, beans, corn and citrus fruits in orchards. According to the prevailing view, fructus naturales are and remain real property for all purposes until they are actually severed from the land. On the other hand, fructus industriales (also referred to as emblements) are usually annual crops which are for most purposes personal property. See Smith and Boyer, Law of Property, pp. 244-245. Since wheat is a fructus industriales crop (or an emblement), it is viewed as being personalty and therefore the proceeds from its sale pass to the life tenant's heirs. Consequently, choice (A) is correct.

Oliver was the owner in fee of two adjacent parcels of land (henceforth referred to as "A" and "B") in downtown Dallas, Oliver's title to parcel "B" was subject to an unrecorded 20 year mortgage given to mortgagee, The Confederate National Bank, in 1960 to secure repayment of a loan of $100,000. Commencing in November 1963, the following events occurred: November 1963: Oliver died, leaving all of his real property to his wife, Jackie. Oliver's will was admit¬ted to probate. December 1963: Having heard about Oliver's death, The Confederate National Bank recorded its mortgage to parcel "B". August 1969: Jackie executed and delivered to Onassis a mortgage deed on parcel "A" which Onassis immediately recorded. The mortgage instrument contained the following recitations. "This mortgage is secured by parcel "A" and all other real estate which I may own in Dallas or have an interest in." Jackie defaulted on her mortgage obligation to Onassis; the amount due on the debt was $100,000. February 1980: The Confederate National Bank brought suit to foreclose its mortgage on parcel "B." Recently, a real estate appraisal has indi¬cated that parcel "A" has a value of $50,000; while the value of parcel "B" was estimated at $40,000. The applicable Texas recording statute provides in part "No deed or other instrument in writing, not recorded in accordance with this statute, shall affect the title or rights to, in any real estate, of any devisee or purchaser in good faith, without knowledge of the existence of such unrecorded instruments." 103. In February 1980, The Confederate National Bank brought suit against Jackie to foreclose its mortgage on parcel "B." Judgment for: (A) Jackie, if she was unaware of the existence of the mortgage at the time she acquired title to parcel " B" (B) Jackie, only if parcel "B" was not subject to the mortgage when she acquired title (C) Confederate National Bank, since Jackie "assumed" the mortgage when she acquired title to parcel "B" (D) Confederate National Bank, since property "once mortgaged remains mortgaged" as against mortgagor's successors in interest

103. (A) The applicable Texas recording statute provides in part: No deed or other instrument in writing, not recorded in accordance with this statute, shall affect the title or rights to, in any real estate, of any devisee or purchaser in good faith, without knowledge of the existence of such unrecorded instruments. When Oliver died and his will was admitted to probate in November 1963, the Confederate National Bank's 1960 mortgage to parcel "B" had not been recorded. Therefore, Jackie's fee simple title in parcel "B" would be unaffected by the prior unrecorded mortgage (in accordance with the relevant Texas recording statute).

104. Assume for the purposes of this question only that the Confederate National Bank succeeded in obtaining a fore¬closure decree against Jackie in the previous question. From its foreclosure (sale) of Parcel "B", The Confederate National Bank received a judgment in the amount of $40,000. The Confederate National Bank now brings suit against Jackie, personally, to recover $60,000 in satisfaction of the original mortgage debt. Judgment for whom: (A) Confederate National Bank, since Jackie "assumed" the $100,000 mortgage when she acquired title to parcel "B" (B) Confederate National Bank, since parcel "B" was subject to the $100,000 mortgage when Jackie acquired title to the property (C) Confederate National Bank, since Jackie, as devisee, would remain primarily liable if the foreclosure failed to satisfy the outstanding indebtedness (D) Jackie, if she did not "assume" the mortgage when she acquired title to parcel "B"

104. (D) Students should note the difference between a transfer of the mortgage property "subject to" and "assuming" the mortgage is the difference between personal liability and the lack there¬of on behalf of the grantee. Thus, if A conveys land to B "subject to the mortgage," this means that B does not personally become liable for the outstanding indebtedness. Of course, the land was encumbered and could be foreclosed to satisfy the debt, but B is personally liable to no one for such debt. Alternative (D) is therefore correct because Jackie would not be personally indebted unless she "assumed" the mortgage.

Judd inherited the bulk of his father's very substantial estate in 1935, including Blackacre, a rambling "farm" of woods and fields in upstate Pennsylvania, Judd, who lived in Philadelphia, used Blackacre as a weekend and holiday retreat. Judd, his wife having predeceased him, died in 1967. His will devised Blackacre to his two sons, Trent and Hunter, "to share and share alike as tenants in common". At the time of their father's death, Trent lived in Pittsburgh while Hunter resided in Washington, D.C. After Judd's funeral, Trent returned to his residence in Pittsburgh, but Hunter decided to occupy Blackacre. He put his name on the mailbox and has paid the taxes and maintenance expenses. Trent has been gen¬erally aware of this, but since he cared little about Blackacre, Trent has never pressed Hunter about the property. Since 1967 Trent has not paid any rent or other compensation to Hunter, nor has Hunter requested such payment. In January, 1992, a series of disputes arose between Trent and Hunter for the first time concerning their respective rights to Blackacre. The Commonwealth of Pennsylvania recognizes the usual common law types of cotenancies, and there is no appli¬cable legislation on the subject. 54. Hunter brings an appropriate action for the partition of Blackacre by sale and requests that in the division of the proceeds he be awarded an equitable additional amount based on the fact that he paid the taxes and made improve¬ments. If Trent contests the award of this additional sum, judgment for whom? (A) Trent, because the maintenance expenses and improvements made by Hunter were voluntary and deemed to have been made for his own convenience. (B) Trent, because as soon as the maintenance work and improvements were made they became a part of the land owned by the cotenants in equal shares. (C) Hunter, provided Trent is allowed an equitable setoff for the value of Hunter's occupancy. (D) Hunter, without any allowance to Trent for a setoff based on the value of Hunter's occupancy.

54. (C) According to Smith and Boyer, a joint tenant has no right of contribution against the other joint tenants for repairs or improvements he has made, but if partition is had the court in mak¬ing an equitable division of the proceeds will take into consideration the expenditures made by one tenant for repairs and improvements. Law of Property, pg. 66. Students should be advised that partition proceedings are under the jurisdiction of a court of equity. So therefore, apply¬ing the maxim "that one who seeks equity must do equity, the court is free to adjust accounts between the parties in a fair and equitable manner." Be advised that choice (C) is correct because "one concurrent owner may be charged for use and occupation of the property (rea¬sonable rental value) and he may be given credit for the value of improvements erected, repairs made and taxes paid." Burby, Real Property, pg. 233.

Kirk Kirksey, a seventy-three year-old widower, owned a forty-acre tract, known as West Grove, located just outside the city limits of Chadds Ford. Kirk had two children, Kevin and Katrina. After Katrina married, she and her husband, Von, lived on West Grove in a small cottage. From 1985 to 1989, Katrina and Von helped Kirk farm and maintain the property. Kirk, whose health was deteriorating, needed the services of Katrina and her husband in order to continue to live on West Grove. In December, 1989, Katrina told Kirk that she and Von were planning to move out of state. Worried that he could not survive without their help, Kirk said to Katrina, "Please don't ever leave. I'm totally dependent on you and Von. If you stay and continue to care for me and help with the farming, West Grove will be yours when I die." Katrina and Von decided to do as Kirk requested. For nine years, Katrina cared for her father while Von handled most of the farming operations. In 1998 Kirk died intestate with Katrina and Kevin as his only surviving heirs. The period required to acquire title by adverse possession in the jurisdiction is seven years. 105. In an appropriate action to determine the legal and equitable rights of Katrina and Kevin, respectively, in West Grove, the result will depend upon the application of the principles of and exceptions to the (A) Statute of Frauds (B) parole evidence rule (C) law for adverse possession (D) doctrine of resulting trusts

105. (A) The Statute of Frauds requires a writing for enforceability of any contract to create or trans¬fer any interest in land, which includes not only legal estates but also equitable interests and liens, rents, expectant interests and estates, leases, easements, and restrictions upon the use of land. Equity, however, will decree specific performance of an oral land contract, despite the Statute of Frauds, where there has been substantial part performance in reliance on the oral contract. Since this question concerns itself with Katrina and Von's reliance on Kirk's oral promise to convey West Grove, the Statute of Frauds is the applicable legal principle. Note that choice (B) is wrong because parol evidence requires a complete and integrated writing. Here, no writing was entered into between the parties. Adverse possession, as denoted in choice (C), is inapplicable because Katrina and Von's possession of West Grove was not hostile and adverse. By the same token, choice (D) is not the best answer because the facts do not indicate Kirk intend¬ed to establish a trust.

107. Suppose that in May, 1980 BTCC wishes to commence construction of its new high-rise complex (containing the hotel, shopping mall, apartments etc.) on the 25 acres of vacant land which it possesses. In an action by Resident, one of the home owners in the West Lake Shore subdivision, to enjoin construction of the complex, plaintiff will most likely (A) succeed, since Resident (or any other landowner in East Lake Shore or West Lake Shore) as assignee of WLDC could reenter the land upon breach of condition subsequent (B) succeed, since a common development scheme had been established for the entire 400 acre tract, BTCC's proposed complex would constitute a non-conforming use (C) not succeed, because WLDC's instrument of March, 1980 effectuated an abrogation of the deed restrictions on BTCC's use of its property (D) not succeed, because in accordance with deed restriction (b), only a homeowner in East Lake Shore would have standing to challenge BTCC's proposed construction plan

107. (C) Choice (C) is the best alternative because WLDC's March, 1980 instrument effectively abrogated the deed restrictions on BTCC's use of its property. Clearly, a common developmental scheme was not established for the 400 acre tract. Since WLDC developed a shopping center on 25 acres of its retained land and also authorized BTCC to construct a country club and golf course on its 50 acres, WLDC intended that the residential developmental scheme only should extend to the 300 acres within the East Lake Shore and West Lake Shore subdivisions.

In 1965, Oliver purchased a one hundred-acre tract located in Tamiami County, Florida. Shortly thereafter, Oliver prepared a sub¬division plan called Sunnyside, which created ninety one-acre residential building lots on this tract with the remaining ten-acre lot proposed for a public school building. In Oliver's sales brochure promoting Sunnyside, he stated that "In addition to the close proximity of the proposed school for Sunnyside residents, the County School District would not need to expend tax money to acquire this property for school construction." In 1966, the Sunnyside subdivision plan was recorded with the Tamiami County Recorder's Office. During the next few years, Oliver sold fifty of the residential lots to individual purchasers. Each deed specified that every lot designated on the Sunnyside subdivision plan was to be record¬ed in the Tamiami County Recorder's Office. Each deed also provided the following: "No house trailer or mobile-home shall be built or maintained on any lot with in Sunnyside." In 1972, Oliver conveyed the remaining forty lots and the ten-acre tract to Marino Builders by deed which included language identical to that contained in the first fifty deeds. By 1977, Marino had sold all of the forty lots. Each of these deeds identified each lot as being a part of the Sunnyside subdivision, but did not include the clause relating to mobile-homes. On January 9,1978, Marino sold the ten-acre tract to Williams. This deed made no mention of the Sunnyside subdivision. On January 15,1978, the School Board of Tamiami County voted to build a new school on the ten-acre tract. On January 30,1978, Arthur, who had purchased one of the residential lots from Marino placed a mobile-home on his property. The same day, Williams began construction of a Pizza Pub on the ten-acre tract. 109. In an action by the Tamiami School Board against Williams to enjoin construction of the pizzeria on the ten-acre tract, the court would grant judgment for (A) Williams, since he was a bona fide purchaser without notice of the proposed school building (B) Williams, since the school dedication was not accepted within a reasonable period of time (C) School District, since the ten-acre tract was designated for public use (D) School District, since the ten-acre tract constituted an equitable servitude

109. (C) The proposal in the subdivision plan and also in the development brochure to set aside a 10 acre tract for the location of a school building would constitute a public dedication offering. Dedication at common law required no particular form and could be made by any method by which the dedicator expressed his intention: by words, conduct or writing. When acceptance by public adoption is manifested, there arises a conveyance of an easement for such public use, with the fee remaining in the grantor. In the present case, the school district would be entitled to erect a school on the tract, thereby enjoining William's pizzeria.

11. Assume for the purposes of this question only that Z builds a concrete wall three feet along his border with Y to prevent the flow of rain water running onto his land from Y's prop¬erty. This causes the surface water to stand and become stagnant on the northerly corner of Y's land. Y demanded that Z remove the wall and upon Z's refusal, Y brought an appropriate action to compel removal. The most likely result is (A) Z must remove the wall because he has no right to obstruct the flow of such surface water (B) Z must remove the wall at Y's expense (C) Z may leave the wall without being liable to Y for money damages (D) Z may leave the wall but he will be liable to Y for money damages

11. (C) A popular Multistate testing area deals with surface waters. Surface waters are those which come from rain, springs and melting snow and ice, and simply follow the contours of the land and have not yet reached a natural water course or basin with well defined bed and banks. Under the common law rule (still followed in a majority of jurisdictions), sometimes called the "common enemy rule" because surface water is considered a common enemy, the lower tract is not burdened with any servitude in favor of the higher land and the owner of the lower tract has the right to protect his lower tract from "the common enemy" or the flow of surface water by making any improvements which are suitable for the purpose.

111. Suppose that Perry, a Sunnyside lot owner, initiates suit against Arthur to force him to remove the mobile-home. Which of the following would be the most accurate statement of law? (A) There is no enforceable restriction because the mobile home proviso did not "run with the land". (B) There is no enforceable restriction, since Arthur's deed did not include the mobile-home provision. (C) The mobile-home restriction would be enforceable, since a common development scheme had been established for the entire Sunnyside subdivision. (D) The outcome turns on whether a common development scheme had been organized for the entire subdivision.

111. (C) The original deed provision between Oliver and Marino prohibiting "house trailers and mobile homes to be built or maintained on any lot within Sunnyside" is an example of a restrictive or negative covenant "running with the land." Therefore, the restriction would be enforce¬able against Arthur even though the prohibition wasn't contained in his deed. In order for a covenant to "run with the land," the following four requirements must be met: (1) the covenant be in writing, (2) it must "touch and concern" the land, (3) privity of estate between covenantor and covenantee and (4) intent that the covenant "run with the land." Since the covenant does "run with the land" and is enforceable, choices (A) and (B) are incorrect. Choice (D) is wrong because Sunnyside was an example of a common residential development scheme.

133. Assume for the purposes of this question only that Dave and his family have not used the cottage for 11 years because of his employment commitment in Europe. During Dave's absence, Johnson constructed an access ramp to his dock, which obstructed the road to Dave's cottage. Upon Dave's return from Europe, he goes to his lakefront cottage and discovers the obstructing ramp. Dave initiates suit against Johnson to compel him to remove the ramp. The court would most likely (A) hold in Johnson's favor, because Dave's absence for 11 years constituted an abandonment of the easement (B) hold in Johnson's favor, since the dominant owner is under an affirmative duty to notify the servient tenement holder of non-use (C) hold in Dave's favor, since mere non-use however long continued, will not as a rule effectuate an abandonment (D) hold in Dave's favor, because of the validity of the agreement for the express grant of the easement

133. (C) Choice (C) is correct because mere non-use, however long continued, will not terminate an express easement. In addition, there must be evidence of some affirmative or overt act on the part of the easement holder, i.e. dominant owner, manifesting his intent to abandon the easement.

112. Which of the following statements is LEAST accurate with respect to Arthur's deed? (A) The covenant prohibiting mobile-homes ran with the land as far as Marino but not as far as Arthur. (B) The covenant prohibiting mobile-homes could be enforced by any Sunnyside lot owner. (C) Arthur should have had constructive notice of the restriction against mobile-homes. (D) All subsequent grantees of Oliver would be in privity of estate.

112. (A) Choice (A) is the LEAST accurate since a covenant "running with the land" will be enforce(C) The restriction "except for residential purposes" should be interpreted to prohibit the operation of a beauty parlor in defendant's home. Such commercial activity clearly violates the plan and obvious purpose of the covenant and is prohibited by its terms. Choices (A) and (B) are incorrect since the change in character of the surrounding neighborhood would not justify failure to enforce the restriction where the original purpose of the restriction could still be realized. Choice (D) is wrong because the operation of a beauty parlor in defendant's home would constitute a violation of the restrictive covenant so as to justify injunctive relief.able against successors of the original parties where, again, there is (1) privity of estate, (2) the covenant "touches and concerns" the land, (3) intent that the covenant "run with the land" and (4) the covenant be in writing. Here, although the restrictive covenant was not written into Arthur's deed, he would, nevertheless, be held to have constructive knowledge of the prohibition against mobile-homes and house trailers. Since the covenant prohibiting mobile-homes could be enforced by any Sunnyside lot owner (as part of the common development scheme), choice (B) is also correct.

Question 115 is based on the following fact situation. Winslow was the record title owner in fee simple absolute of Blackacre, a 100-acre of farmland located in the township of Devonshire. In 1980, Winslow devised the property to his daughters, Amy and Brooke, as joint tenants with right of survivorship. The next year, Amy executed a deed to Cooke as follows: "I hereby convey all of my right, title and interest in the North East Quarter of Blackacre to Cooke and his heirs." Thereafter, Amy borrowed $100,000 from Drake and a promissory note was executed as evidence of the debt. In 1990, Amy defaulted on the loan and Drake, as judgment creditor, levied upon and sold to Zorn on execution sale all of "Amy's right, title and interest in the South Half of Blackacre." In December, 1991, Amy died intestate leaving Hilton, her husband, as her sole surviving heir. 115. Who owns Blackacre? (A) Brooke and Cooke are tenants in common of the North East Quarter of Blackacre; Brooke and Hilton are tenants in common of the North West Quarter of Blackacre; and Brook and Zorn are tenants in common of the South Half of Blackacre. (B) Brooke and Cooke are tenants in common of the North East Quarter of Blackacre; Brooke is the owner in fee of the North West Quarter of Blackacre; and Brooke and Zorn are tenants in common of the South Half of Blackacre. (C) Brooke and Cooke are tenants in common of the North East Quarter of Blackacre; Brooke and Hilton are tenants in common of the North West Quarter of Blackacre; and Zorn is the owner in fee of the south Half of Blackacre. (D) Brooke and Cooke are tenants in common of the North East Quarter of Blackacre; and Brooke is the owner in fee of the remaining three-quarters of Blackacre.

115. (B) It is important to point out that in a joint tenancy each joint tenant owns the whole of the property and that he does not own a share or fractional part thereof. In this regard, a joint tenant may convey away his entire interest in jointly owned property or dispose of a fractional part thereof. Thus, in the present example Amy's deed to Cooke carved out and vested in Cooke an undivided one-half interest in the North East Quarter of Blackacre which Cooke owned as a tenant in common with Brooke. Next, since a joint tenant has the right and power voluntarily to dispose of his interest in jointly owned property, his creditors have the right and power to take that interest involuntarily. Therefore, Amy's judgment creditor, Drake, had the right to levy upon and sell Amy's interest in the South Half of Blackacre. As such, Brooke and Zorn are tenants in common of the South Half of Blackacre, each owning an undivided one-half interest therein. Finally, with respect to the North West Quarter of Blackacre Amy and Brooke remained joint tenants of that Quarter until Amy's death. Brooke's right of survivorship defeats the right of Amy's surviving heir, Hilton. Consequently, Hilton can claim no interest in the North West Quarter of Blackacre.

118. Which of the following is the most accurate statement regarding the enforceability of the single family residency restriction which Collins recorded in February 1972? (A) The residency restriction would be enforceable against all purchasers of lots in the Devon Farms tract, since a common development scheme had been established. (B) The residency restriction would be enforceable only against the owners of the thirty-four lots in the north¬western and southern sectors of the Devon Farms tract. (C) The residency restriction would be unenforceable, since it would not be binding on all lot owners. (D) The residency restriction would not be enforceable since the covenant does not "touch and concern" the land.

118. (B) In the present case, Collins' intention was to create single family residency restrictions only on the lots in the northwestern and southern sectors of the Devon Farms tract. Clearly, Collins did not place any deed restrictions on the lots in the northeastern sector of the tract. In order for the burden of an equitable servitude to run with the land and be enforceable against a transferee of one of the original parties three elements must co-exist: (a) there must be a writing which complies with the Statute of Frauds and (b) there must be an intention that there be a restriction on the use of the land involved and (c) the transferee must take the land with either actual or constructive notice of the existence of the servitude.

119. Assume that in 1975, Volley decided to build a swimming pool in the area on which the tennis courts were located. In an action by Boris to enjoin construction of the swimming pool, he will most likely (A) succeed, since the 1973 Volley-Collins deed restriction would be enforceable by successors in interest (B) not succeed, since it was the covenantor's intention to enforce the restriction only as long as he would reside on the property (C) not succeed, since there would not be privity of estate between Collins and Boris (D) not succeed, since the construction of the swimming pool would be a conforming use within the Volley-Collins deed provision

119. (A) Boris will succeed since the 1973 Volley-Collins deed restriction reciting that "the property hereby occupied by the tennis courts shall not be used for construction of any building whatsoever..." would be enforceable by successors in interest. It is important to remember the transferees of the original parties to an equitable servitude are bound by the servitude if it is intended to bind the land. There are four situations, on the other hand, that a court of equity may refuse to enforce an equitable servitude: (1) if its purpose is contrary to public policy or (b) the granting of relief would do more harm than good or (c) when the granting of the relief prayed for would be futile, or (d) the plaintiff is guilty of laches or violating the servitude.

On March 1, 1979 Oscar, the sole owner and occupant of Lot 1, died and devised Lot 1 to Delbert and Desiree "as their community property." Delbert and Desiree were brother and sis¬ter, and neither was married. Lot 1 consisted of a single family house with a yard, garage, and driveway, as illustrated below: Driveway House House Ga-rage House LOT 3 LOT 2 LOT 1 (JORDON) (WILLARD) (DELBERT & DESIREE) On May 1, 1979 Desiree moved into the house on Lot 1. One year later, Delbert and Desiree executed and delivered the fol¬lowing deed instrument to Willard "...hereby grant to Willard the Northerly 30 feet of Lot 1, consisting of the paved driveway now existing, to be used for the ingress and egress of motor vehicles, but should Willard, or his heirs and assigns use said property for any other purpose all the rights, privileges and immunities herein granted shall cease and determine." In consideration for the said deed, Willard paid Delbert and Desiree $2,000 (which they divided equally). The deed was never recorded by Willard. Since Desiree didn't own a car, she never used the driveway. Similarly, Willard never used the driveway because he unexpectantly had his driver's license suspended shortly after executing the above instrument. Desiree died intestate on May 1, 1981, leaving Dawn, her daughter, as her sole heir. During her occupancy, Desiree paid $3,500 each year in insurance and property taxes. In addition, the premises on Lot 1 had a fair market value during this period of $1,500 each year. Following her mother's death, Dawn moved into the house on May 2,1981. On June 1, 1981 Willard sold Lot 2 to Jordan, the owner of Lot 3, by a deed which con-tained no mention of the driveway located on Lot 1. Willard and Jordan assumed that the latter had the right to use the driveway, so they didn't insert any recitations in their deed instrument regarding the driveway. Immediately upon her taking possession of the premises, Dawn began to use the driveway as her means of access to Lot 1. Consequently, she objected to Jordan's use of the driveway. Moreover, as the administrator of Desiree's estate, Dawn sought to collect from Delbert one-half of the cost of insurance and property taxes that Desiree paid. Conversely, Delbert claimed that he was not liable for any of the expenses. Furthermore, Delbert's attorney, Percy, advised him that Dawn did not own any interest in Lot 1 and that since Desiree's death he (Delbert) owned the entire property. Percy also informed Delbert that Dawn owed him rent for the entire period of her occupancy, and if she continued to occupy the premises, then she would be liable for insurance and property taxes as well. 135. In an appropriate action to determine the title of Lot 1, what, if any, interest does Dawn have in the said property? (A) none, because Delbert acquired title to the whole of the property by right of survivorship. (B) an undivided one-half interest, because upon the death intestate of a tenant-in-common, the latter's interest descends to his heirs. (C) an undivided one-half interest, because in a tenancy by the entirety either sibling can make a testamentary disposition. (D) an undivided whole interest, because in a joint tenancy every joint tenant is part and parcel of the unit group which owns the whole.

135. (B) Students should note that a tenancy in common was created by Oscar's testamentary devise to Delbert and Desiree. Since they were not husband and wife, choice (C) is obviously incorrect. Upon the death intestate of a tenant in common, his interest descends to his heirs. Therefore, Dawn (as Desiree's sole heir) would have an undivided one-half interest in Lot 1. It is important to point out that there is no right of survivorship in a tenancy in common.

Looney was the owner of a 300-acre tract of land that was located in Pittman County. Over the course of time, she developed the property into a residential subdivision known as Devonshire. Lincoln Boulevard, a four-lane public highway, ran along the northern boundary of Devonshire. When Devonshire was first plotted, Looney constructed a private road called Looney Lane across the western boundary of the subdivision. Looney Lane was used only for the benefit of the owners who purchased lots in the subdivision. Dooley owned a 20-acre tract that was situated just below Lincoln Boulevard and immediately adjacent to the west side of Looney Lane. Dooley's property was divided into two 10-acre parcels: Parcel 1 was the northern half and bordered along Lincoln Boulevard; Parcel 2 was the southern sector and abutted the Peekskill River. In 1997, Dooley conveyed Parcel 1 to Perez by warranty deed. No encumbrances were mentioned in the deed. Six months later, Dooley conveyed Parcel 2 by warranty deed to Eagleson. Both Perez and Eagleson promptly recorded their deeds with the Pittman County Recorder's Office. 75. Assume for the purposes of this question only that no part of Parcel 2 adjoins a public road. Consequently, Eagleson constructed an access road connecting his property to Looney Lane. After Eagleson used Looney Lane for approximately two months, Slotnick, a Devonshire lot owner, ordered Eagleson to discontinue using the private road. In an appropriate action by Slotnick against Eagleson to enjoin the use of Looney Lane, Eagleson will (A) win, because there is an easement by necessity over Looney Lane (B) win, because there is an implied easement appurtenant over Looney Lane (C) lose, unless Looney Lane is the only access road by which Eagleson can gain ingress and egress to his property (D) lose, because he has no legal right to travel over Looney Lane

75. (D) In Slotnick's action to enjoin Eagleson's use of Looney Lane, Eagleson will lose because he has no legal right to drive over Looney Lane. Choice (D) is correct. Looney constructed Looney Lane as a private road for use only by owners of the lots in the subdivision. The road was not intended to benefit outside landowners, such as Eagleson, who did not reside in the subdivision. Choice (B) is incorrect because an implied easement is based on the inferred intention of the parties, namely the grantor and the grantee. Use of Looney Lane arises between Looney, the grantor, and her grantees, such as Slotnick. Eagleson, on the other hand, was Dooley's grantee, and constructed the access road to Looney Lane after he purchased Parcel 2 from Dooley. Choice (A) is incorrect because an easement by necessity requires, inter alia, that at the time of the conveyance one part of the land is being used for the benefit of the other part (a quasi-easement). Smith and Boyer, Survey of the Law of Property, p. 384. Looney Lane was not used by Eagleson until after he built the access road connecting Parcel 2 with it. Choice (C) is incor¬rect under the facts stated since Eagleson could use the access road he constructed to reach Lincoln Boulevard.

Assume for the purposes of this question only that Y became engaged in experimentation with artificial means to seed clouds in an attempt to suppress damaging rain storms. The rain storms apparently originated in and over the area with¬in which Z's land is located. Z brought suit to enjoin such cloud seeding activity. At trial, it was found that Y had been conducting its activities over clouds above Z's land. According to further testimony, Y's seeding operations did, in fact, cause the complete dissipation of the clouds. The court should (A) enjoin Y's seeding operations because it constitutes an unreasonable interference with the space above Z's land (B) enjoin Y's seeding operations because one does not have the right to interfere with natural rainfall (C) not enjoin Y's seeding operations if they were neces¬sary to protect his property from the rain storms (D) not enjoin Y's seeding operations because Z does not own the space above his land

12. (A) The possessor of real property has the right to the exclusive possession of the surface of the ground, the airspace above and the soil underneath, the extent of which is determined by his exterior boundaries extended vertically upward and downward. This is commonly referred to as the "ad coelum" doctrine as to the upper space and was expressed in the Latin maxim, "cujus est solum ejus est usque ad coelum." Translated literally, "whose is the soil, his is to the sky or high heavens." In applying this rule, choice (A) is correct because any use of the space above one's land which is unreasonable, improper, or interferes with the use and enjoyment of the surface can constitute a trespass.

120. When Tenant had been informed of the condemnation of the parking lot, he contacted Collins and told him that he was terminating the leasehold agreement. Assume that 85 percent of Tenant's customers drove to the store and would be adversely affected by the lack of parking facilities. Which of the following is the LEAST accurate statement concerning Tenant's obligations under the leasehold agreement? (A) The condemnation of the parking lot would not relieve Tenant of his obligation to pay rent under the lease. (B) The "taking" of the parking lot under eminent domain would not entitle Tenant to an abatement of the rental. (C) Collins would be required to abate the rent as just compensation for the condemnation of the parking lot. (D) Tenant would only be relieved of his obligation to pay rent if the condemner takes the entire fee to the leased premises.

120. (C) The question of law presented here, namely when there is a partial taking of a leasehold estate by condemnation and the remaining portion is susceptible for occupation, shall the contractual monthly rental be abated pro tanto for the part taken during the remainder of the time of the lease? The majority rule is that rentals are not abated in this situation, but rather the ten¬ant is obligated to continue the payment of the rentals provided in the lease contract and must look to an apportionment of the damages assessed against the condemning authority based on the reduced value of his lease.

Andrews, Boswell, Conrad and Davies own adjoining property. All four are enthusiastic badminton players. They have decided that instead of having each person construct his own badminton court, it would make much more sense if they all could get together to build a set of courts in one place with all parties having equal rights to use the courts. The parties envision a proposed badminton court layout as appears in the diagram below: (A) Andrews Proposed Access (B) Boswell (C) Conrad (D) Davies Proposed Access Proposed Badminton Courts I 122. The parties want the badminton courts physically to occupy part of Boswell's and part of Conrad's land, but they want Andrews and Davies to have equal right of access to, and use of, the courts with Boswell and Conrad. Which of the following devices would most likely accomplish the implementation of their proposed badminton court layout and be most readily acceptable to the parties? (A) a covenant against partition (B) an indenture granting cross easements (C) an equitable servitude (D) a fee simple upon condition subsequent

122. (B) An indenture granting cross easements would most likely accomplish the plan as well as be most acceptable to all of the parties involved. Students should be aware that an indenture is a deed to which two or more persons are parties in which the parties enter into reciprocal and corresponding grants or obligations to each other. The cross easements would grant equal rights of access and use to Andrews and Davies who own the adjoining properties identified as (A) and (D) on the diagram above. Please note that Boswell and Conrad are owners of the servient tenement [lots (B) and (C) on the diagram]. Andrews and Davies are owners of the dominant tenement [lots (A) and (D)], since they are benefited by the easements. Thus, easements appurtenant are created by the indenture.

123. Suppose that the parties want to set up some mechanism to insure that the construction and maintenance costs of the badminton courts will be shared fairly among all of the parties. The best way to arrange this legally is by (A) an affirmative covenant (B) a cooperative organization (C) co-ownership in tenancy in common (D) implied reciprocal servitudes

123. (A) An affirmative covenant would provide the best legal arrangement under our circumstances. Promises respecting land are referred to as covenants. In our hypothetical, the covenant would require an affirmative action, thus binding the parties to share in the construction and maintenance costs of the badminton courts. In effect, the obligation to pay money would be utilized for purposes relating to the land, e.g., it "touches and concerns" the land. Furthermore, the payment relates to the land in which all four of the parties have an interest. Therefore, such a covenant to pay money could be enforceable at law as a covenant running with the land. It is important to note that there are four prerequisites for the creation of a covenant at law: (1) a writing in compliance with Statute of Frauds (2) an intent that the covenant run with the land (3) the covenant must "touch and concern" the land and (4) privity of estate must exist between the covenantor and covenantee.

35. If Leonard brings suit against Thomas for payment of rent in arrears, he would most likely: (A) succeed, because the tenant is estopped to deny Leonard's title (B) succeed, because of his security interest in the property (C) fail, because the purchase by Thomas vitiated any fur¬ther contractual obligations (D) fail, because he was under a duty to keep the demised premises in reasonably good repair

35. (C) After purchasing the property at the sheriff's sale, Thomas, as record title owner, would no longer remain obligated under the leasehold contract. As such, choice (C) is correct because the sheriff's sale would vitiate Thomas' obligation to make any further rental payments to Leonard.

124. Suppose further that the parties want the badminton court layout to be a more or less permanent agreement. Yet, at the same time, they are not quite sure what they want to do when one of the group sells his property to someone else. On the one hand, they want to include the new purchaser in the arrangement if they like him, but, on the other hand they want the right to exclude him if they don't. They are particularly uncertain what to do if one of the sellers is Boswell or Conrad, on whose property the courts will be built. The best advice about how to solve their problem is (A) partition into four separate strips and an indenture granting cross easements (B) a trust to hold the four adjoining properties in perpetuity (C) exclusionary zoning provided that the appropriate authority could be persuaded to zone the property in accordance with their plans (D) an indenture granting cross easements containing a restrictive covenant

124. (D) An indenture granting cross easements containing a restrictive covenant would best serve the interests of the parties involved here. To reiterate, an indenture is a deed containing reciprocal rights and obligations. The cross easements in the indenture would assure equal rights of access and use to Andrews and Davies. A restrictive covenant in the indenture would place a restriction on the use to which any of the four lot owners could make of their land. In effect, the restrictions would take the form of "equitable servitudes." Students should note that an equi¬table servitude which is enforceable in a court of equity is more than just a covenant running with the land, because it is an interest in land. Students should note that in order for equitable servitudes to be enforceable against a subsequent grantee of one of the original parties, four requirements must coexist: (1) a writing in compliance with the Statute of Frauds, (2) an intention that there be a restriction on the use of land involved, (3) the servitude must "touch and concern" the land, and (4) the subsequent transferee must take with actual, constructive or implied notice of the existence of the restriction.

On October 1, 1920, Jackson Stone, owner of several hundred acres in the County of Los Angeles, drafted a general development plan for the area, to be known as the Del Mar addition. The duly recorded plan imposed elaborate limitations and restrictions upon the land in the addition, which was to be developed as a residence district. The restrictions were to extend to all persons acquiring any of the lots and to their heirs, assigns and lessees. It was further provided that all subsequent owners would be charged with due notice of the restrictions. Among those restrictions in the general plan were the following: (22) A franchise right is created in a strip of land ten feet in width along the rear of each lot for the use of public utility companies with right of ingress and egress. (23) No house or structure of any kind shall be built on the aforementioned strip of land running through the said blocks. In 1940, Bernie Smith purchased one of the lots, built a house and erected a fence in the rear of his property within the restrict¬ed area. In 1944, Wilber Hemmings purchased a lot adjacent to the Smith property and built a new house. Two years later, Sam Roberts purchased the lot which adjoined Wilber's property. The three deeds to those properties each contained references to the deed-book where the general plan was recorded. In 1948, Sam Roberts began the construction of a seven foot post and rail fence along the line dividing his lot with Wilber's and along the center of the area subject to the franchise right. Although Wilber objected to its construction, the fence was completed. 125. The franchise right created for public utility companies would most likely be an example of a (an) (A) license (B) equitable servitude (C) easement appurtenant (D) easement in gross

125. (D) The franchise right would be construed as an easement in gross. This nonpossessory interest is created when the holder of the easement interest acquires his right of special use in the servient tenement independent of his ownership or possession of another tract of land. In an easement in gross, the easement holder is not benefited in his use and enjoyment of a possessory estate (i.e., there is no dominant tenement) by virtue of the acquisition of that privilege. Choice (A) is incorrect since a license is a revocable privilege to enter upon the land of another. Alternative (B) incorrect since an equitable servitude is a restriction on the use to which an owner may make of his land. Choice (C) wrong because the utility company did not possess an adjoining dominant tenement.

126. The court will most likely construe Restriction (23) as a (an) (A) negative easement (B) equitable servitude (C) affirmative covenant (D) fee simple absolute

126. (B) The proviso prohibiting lot owners to construct a house or structure on the strip of land would properly be construed as an equitable servitude. An equitable servitude is a restriction on the use to which an owner may make of his land. It is created by (a) a writing complying with the Statute of Frauds, (b) concerning a promise that "touches and concerns" the land and (c) indicating an intention that the servitude exists. The basis for enforcing an equitable servitude is that one who takes land with notice (actual or constructive) of a restriction on the land cannot in equity be allowed to violate that restriction. Since the restriction, by itself, does not create an easement, choice (A) is incorrect. Alternative (C) is wrong because the lot owners are under a negative, not affirmative, obligation to refrain from constructing on the said strip. Choice (D) is inapplicable.

127. If Wilber seeks a mandatory injunction to compel removal of Sam Robert's fence, the court will most likely (A) grant relief, since the fence was in violation of the easement restriction (B) grant relief, since the encroachment of the fence violated the restriction in the original plan (C) deny relief, since Wilber failed to enforce the restriction against Bernie Smith (D) deny relief, since the fence would not be construed as "a structure" within the terms of the restriction

127. (B) Since the encroachment of the fence violated the restriction (i.e., equitable servitude), Wilber would be entitled to injunctive relief. Choice (A) is incorrect since the restriction would be construed as an equitable servitude, rather than an easement. Choices (C) and (D) are incorrect because any lot owner in Del Mar may enforce the restriction.

Whiteacre, Greenacre and Blueacre are three business lots abutting each other with Greenacre between the other two lots. Mr. Green owns Greenacre in fee simple and maintains his dwelling house thereon. He has a right of way, granted in written agreement, across Blueacre for crossing Blueacre on foot, by bicycle, or automobile. Mr. Blue is the owner of Blueacre. Mr. White owns Whiteacre and conveys it to Mr. Green in fee simple. Mr. Green then builds a fifteen story office building cov¬ering Whiteacre and Greenacre, which houses 6,000 persons during working hours each day. All of these persons use the right of way over Blueacre and Mr. Green uses the way for delivering as many as thirty loads of supplies per day to the office building. 128. Mr. Green's property interest in the use of his right of way across Blueacre may best be described as a (an) (A) license (B) easement in gross (C) easement appurtenant (D) prescriptive easement

128. (C) An easement is deemed appurtenant when the right of special use benefits the holder of the easement in his physical use or enjoyment of another tract of land. For an easement appurtenant to exist, there must be two tracts of land. One is called the dominant tenement (which has the benefit of the easement). The second tract is the servient tenement (which is subject to the easement right). In the case at bar Green's property interest in the use of his right of way across Blueacre would be construed as an easement appurtenant.

129. In an action by Mr. Blue to enjoin Mr. Green and the office workers from using the right of way across Blueacre, the court will most likely hold that (A) Mr. Green's right of way would be extinguished due to excessive use by the office workers (B) Mr. Green's right of way would be forfeited due to the unauthorized use by the office workers (C) Mr. Green, by making use of the right of way beyond the scope of the original privilege, would permanently be enjoined from using the servient tenement (D) Although Mr. Green may continue to use the right of way, the office workers would be enjoined from making such use

129. (D) Students should note that mere excessive use of an easement does not forfeit or extinguish the easement. In the case of excessive user the owner of the easement is simply making use of the servient tenement beyond the scope or extent of the use permitted by the easement in its creation. The easement as it was originally created still exists and within its scope can be used. But the user in excess of the scope of the easement can be enjoined and damages may be had for injury caused by the excess user. Thus, although Mr. Green may continue to use the right of way, the office workers use of the right of way is not merely excessive, it is wholly unauthorized. See 18 Harv. L.R. 608.

136. After Dawn refused to permit Jordan to use the driveway, he brought suit to determine his right to continue use of the driveway. Jordan should (A) win, because he acquired an implied easement to use the driveway as owner of the dominant tenement. (B) win, because Willard's easement to use the driveway was conveyed to Jordan. (C) lose, because the Statute of Frauds was not satisfied. (D) lose, because Willard's non-use of the driveway effectuated an abandonment of the easement.

136. (B) Choice (A) is incorrect because an easement by implication arises from the circumstances surrounding the dividing by the owner of a piece of land into two pieces and conveying one of such pieces to another. From such surrounding circumstances an inference is drawn that the parties intend the creation of an easement. In the present case, Delbert and Desiree created an "express" easement in the deed instrument permitting Willard to use the driveway. Choice (C) is wrong because the written deed instrument does satisfy the Statute of Frauds. In addition, choice (D) is incorrect because mere non-use does not constitute an abandonment of the easement. Thus, choice (B) is the best alternative because a conveyance of a dominant tenement (Lot 2) carries with it all easements and profits appurtenant thereto as incidents unless it is otherwise expressly provided.

In 1960, Ezekiel Green, advancing into old age, realizes that he is no longer able to farm his 1000 acres; therefore, he decides to sell some of the farm land in parcels of 250 acres. Bill Jones, president of Up-swing Development Company, is interested in purchasing three of the parcels. Jones buys the three parcels from Green and begins formulating plans for constructing single family dwelling units on the land which is located in an upper middle class area of northern Tower County. The original deed between Green and the Up-swing Development Co. contains a provision expressly binding "upon all subsequent grantees, their heirs and assigns" stipulating that any further subdivisions by any such persons shall be restricted to minimum two acre lots to be used only for single family dwelling units. The Development Co. immediately subdivided two of the parcels into lots of 3, 4, and 5 acres and began construction of homes thereon. The original deed restrictions were enumerated within the special warranty deeds which were given to the purchasers of the homes in the new development named Tower Hills. Two years later, Jones sold the remaining parcel which was not included in the Tower Hills subdivision plan to Tonney Contractors, Inc. Tonney, in turn, drafted a subdivision plan for the last 250 acre parcel dividing it into 1 acre lots. Tonney then commenced construction of single family dwelling units in the new development to be known as Tower Farms. There was no mention of the restriction for 2 acre minimum lots in the deeds to the purchasers of the new homes in Tower Farms. Meanwhile, after Ezekiel Green's death, his estate is required to sell the remaining 250 acre parcel of his farm land. The buyer is Sid Slick Associates, who propose to construct a two level shopping center and parking lot on the property. 137. In the original deed between Green and Bill Jones' Up¬swing Development Co., the stipulation contained in the deed which restricted the size and residential character of any subsequent subdivision of the parcels is an example of a (an) (A) easement (B) affirmative covenant (C) covenant for quiet enjoyment (D) negative covenant

137. (D) The original deed provision between Ezekiel and Jones restricting subdivision of the parcels to minimum two acre lots for residential use is an example of a restrictive, or negative, covenant running with the land. In order for a covenant to run with the land, the following four requirements must be met: (1) the covenant be in writing, (2) it must "touch and concern" the land (3) privity of estate between covenantor and covenantee and (4) intent that the covenant run with the land. Choice (B) is incorrect since an affirmative covenant imposes an obligation to perform a duty on the part of the covenantee, i.e. obligation of condominium owner to pay an annual fee for maintenance of common areas.

138. In a subsequent action to enjoin Tonney Contractors from subdividing the parcel into 1 acre lots, by any of the present owners of lots in the Tower Hills development, the most probable judicial determination would be (A) that the action should be dismissed because the owners lack standing to sue (B) that the action should be dismissed because there is no privity of estate between the owners of the lots in Tower Hills and Tonney Contractors, Inc. (C) that the action would be successful if Tower Farms was considered by the court as part of a common development scheme (D) that the action would be successful since the restrictions in the original deed between Green and Jones will be enforceable

138. (D) (D) is correct since a covenant running with the land will be enforceable against successors of the original parties, where, again, the following four requirements are met: (1) there is privity of estate between the original parties and their successors, (2) the covenant "touches and concerns" the land, (3) the parties intend the covenant to run with the land and (4) the covenant is in writing. Thus, the subsequent grantees, i.e. owners of Tower Hills lots, may enforce the original restrictive covenant between Ezelkiel and Jones.

139. Assume for the purposes of this question only that the area surrounding the Tower Hills and Tower Farms developments was re-zoned for commercial and industrial uses in 1972. Sid Slick's shopping center has grown to include 150 stores. Now one of the lot owners in Tower Hills con¬tracts to sell his property to two physicians who plan to start a suburban medical practice. In an action by the home owners in Tower Hills to prevent such commercial use, the court will most likely hold that (A) the restrictions are still enforceable, thus preventing such commercial use (B) the restrictions would no longer be enforceable, because the offering of personal services (i.e., medical) would be a conforming use (C) the restrictions would no longer be enforceable, because of the change in the character of the neighborhood (D) the restrictions would no longer be enforceable, since the opening of a physician's office in a private home would not be construed as a commercial enterprise

139. (A) Choice (A) correct because the change in the character of the neighborhood was not of such a magnitude so as to render the restriction unenforceable.

14. For this question only, assume the following facts. On January 1, 1989, Quirk failed to pay the note when it came due. The next day, Uribe, the escrow agent, delivered the deed to Quirk Towers to Lama. Lama then properly record¬ed this deed on January 3. One week later, on January 10 Lama conveyed Quirk Towers by warranty deed to Gonzales for the purchase price of $200,000. On January 12, Quirk tendered the $125,000 balance due to Lama which he refused to accept. Quirk now brings an appropri¬ate action against Lama and Gonzales to set aside the con¬veyance and to permit the redemption of the property by Quirk. Which of the following best states Quirk's legal rights, if any, in his action against Lama and Gonzales? (A) Quirk has no rights against Gonzales but Quirk does have an action for redemption against Lama for the value of the property. (B) Quirk has no rights against Lama but Quirk does have an action for redemption against Gonzales for the value of the property. (C) Quirk has the option of seeking redemption against either Lama or Gonzales for the value of the property but Quirk cannot set aside the conveyance. (D) Quirk has no rights against either Lama or Gonzales because he defaulted on the promissory note.

14. (A) When the "mortgagee" under a deed absolute mortgage transfers to a bona fide purchaser, the mortgagor has no rights against the bona fide purchaser, but he does have an action for redemption against the "mortgagee "for the value of the land, or, at his election, the proceeds of the sale. The theory is that the mortgagee now has the value of the land in his hands as a separate fund, and such fund as a substitute for the land may be redeemed by the mortgagor. Applying this rule to our given set of facts, Quirk has no right against Gonzales, the bona fide purchaser, but he does have an action for redemption against Lama, the mortgagee.

36. In a subsequent proceeding by Leonard against Thomas to recover for the damage to the parquet floors resulting from the leak, liability would most likely be imposed upon: (A) Leonard, because he was under an implied obligation to keep the premises in a habitable condition (B) Leonard, because he was under an affirmative obliga¬tion to deliver the premises in a reasonable state of repair (C) Leonard, because of the contractual obligation under the lease to make all necessary repairs during the term of the lease (D) Thomas, because a tenant for years is obligated to make such ordinary repairs on the leased property

36. (D) Thomas is liable for the damage to the floors caused by his failure to replace the shingles since a tenant for years is legally bound to make such ordinary repairs on the leased property as will avoid serious injury to the property. Thus, Thomas' failure to make the necessary repairs would make him liable for such permissive waste.

Questions 141-142 are based on the following fact situation. Adam, Andy and Archie are brothers who inherited their father's farm when he died testate, leaving his farm to Adam, Andy and Archie as joint tenants. Soon after the father's death, Andy died and Archie moved to another part of the country. Adam has not heard from Archie in many years. Prior to their father's death, Adam and Andy operated their father's farm. They continued doing so after their father died, sharing all expenses equally and dividing the profits between them. Following Andy's death, Adam has continued to operate the farm for his own sole benefit. Recently, Adam has granted the Pittsburgh Coal Co. rights to strip mine coal from underneath the farm. Their agreement stipulated that the Coal Company would pay Adam a per ton royalty for the coal extracted. In addition, the Coal Company agreed to fill in the excavated area and replace top soil on the surface of the land. After Pittsburgh Coal Co. commenced their strip mining operations, Adam noticed that the Coal Company was not filling in the excavated area as previously agreed. However, since the Coal Company paid Adam all the coal royalties from the strip mining, he did not voice any objections regarding their failure to replace the top soil. Two years later, Pittsburgh Coal Co. has now completed its strip mining operations under their arrangement with Adam. 141. Pittsburgh Coal Co.'s right to strip mine coal from Adam's property would be an example of a (an) (A) profit-a-prendre (B) license (C) easement in gross (D) voluntary waste

141. (A) A profit (sometimes called a "profit-a-prendre") is a nonpossessory interest in land and consists of a right to take the soil or a substance of the soil. If the exercise of the right is restrict¬ed in the sense that it may be exercised only in connection with the use and enjoyment of a dominant estate, the right is a profit appurtenant. If the right is not so restricted, it is a profit in gross. Students should note that the difference between a license and a profit (or an easement) is that a license may be revoked at the will of the licensor. However, it is generally recognized that a license personal is irrevocable if it is coupled with an interest.

Question 143 is based on the following fact situation. In 1950 Jason Woodstock owned Twin Oaks, a forty acre tract just outside of White Plains in Yorkshire County. The following year Jason devised Twin Oaks "to my son, Socrates, and my daughter, Delilah, for their respective lives as tenants in common, then to the heirs of my son." Jason died in 1952. On January 1, 1953 Sampson went into adverse possession of Twin Oaks. On July 1, 1962 Socrates died leaving Homer as his only heir. Homer was exactly nine years old at Socrates' death. Two weeks after her brother's death, Delilah committed suicide. Sampson has been in continuous possession of Twin Oaks since January 1, 1953. The period of majority in this jurisdiction is twenty one (21) years of age. The Statute of Limitations on adverse possession in this Jurisdiction is twenty years (20). In addition, this jurisdiction has the following statute in effect: "An action for the recovery of land shall be commenced only within twenty years after the right of action first occurred, but if a person entitled to bring such action, at the time the cause thereof occurs if within the age of minority, of unsound mind, or imprisoned, such person after the expiration of twenty years from the time the cause of action occurs, may bring such action within ten years after such disability is removed." 143. On July 1, 1975, Homer brought an action for ejectment against Sampson. Judgment for (A) Homer, even though he was not a life in being at the time of the devise in 1951 (B) Homer, since his minority tolled Sampson's period of adverse possession (C) Sampson, since he acquired a fee simple interest to Twin Oaks by adverse possession (D) Sampson, only if his possession of Twin Oaks was open, hostile and notorious

143. (B) At the time of the commencement of Sampson's adverse possession, Socrates and Delilah had life estates while Homer had a vested remainder in fee simple. When Sampson took possession of the premises, he trespassed on the possession of Socrates and Delilah, not on the possession of Homer. Indeed, Homer had no right to possess Twin Oaks until the death of the life tenants. Hence, Homer had no cause of action against Sampson and no statutory period began to run against Homer. When Socrates and Delilah died in 1962, Homer was 9 years of age. Based on the tolling statute in effect, Homer is given an additional ten years after the disability is removed to bring his cause of action against Sampson.

A owns a tract of land located in the state of Suffolk. On June 1, 1989, A sells to H and W, husband and wife, an undivided one-half interest in this land for $100,000 (the entire consideration being paid by H). The deed to H and W reads as follows: "to H and W and their heirs as tenants by the entirety and not as joint tenants and not as tenants in common." On June 1, 1990, A sells his interest in the tract of land to H and his brother B for $125,000 (H pays $80,000 of the purchase price and B pays the balance). The deed to H and B provides: "to H and B and their heirs as joint tenants and not as tenants in common." All deeds are promptly recorded. Note: Suffolk has the common law rules regarding dower and curtesy in effect. 144. H conveys to C all his right, title and interest under the two deeds from A. H then dies. B then dies. C is thus the owner of: (A) an undivided one-third interest in the land (B) an undivided one-quarter interest in the land (C) an undivided one-half interest in the land (D) an undivided three-quarter interest in the land

144. (B) At common law, a tenancy by the entirety was created by a deed or will conveying to a husband and wife. The incident of survivorship attached such that neither spouse acting alone while the marriage existed could sever the tenancy or defeat the right of survivorship. See Smith and Boyer, Law of Property, pg. 65-66. Therefore, when H conveyed all his interest under the June 1, 1989 deed to C, the tenancy by the entirety remained intact, and C received no inter¬est. A joint tenancy, on the other hand, can be severed by conveyance of the interest of one of the joint tenants. Such a conveyance destroys the unities of time, title, and interest and results in the creation of a tenancy in common (which contains only a unity of possession). Therefore, when H conveyed all his interest under the June 1, 1990 deed to C, the joint tenancy was severed with C and B each holding an undivided one-half interest as tenants in common. Upon B's death, no right of survivorship inured to C, so C retains only his one-half interest. Since the two deeds from A to H each comprised one-half of the entire tract, C's interest is an undivided one quarter interest in the land. Choice (B) is thus correct. Note that the fact that H paid more of the purchase price than B under the 1990 deed ($80,000 of the $125,000 purchase price) is irrelevant because H and B took as joint tenants—the language of the deed so expressed this intent— and therefore, by definition, they had to take the same interest (i.e., unity of interest). Had they taken as tenants in common, then H would have owned a greater interest that B, even though both co-tenants would have shared an equal right of possession.

145. Is this interest in C subject to dower rights in W. (A) Yes, upon H's death W is automatically entitled to a life estate in one-third of C's land. (B) Yes, but W has a mere inchoate right of dower until such time as she actually initiates a cause of action. (C) No, because a widow is entitled to dower only if her husband was actually or constructively seised of a freehold estate of inheritance during the marriage. (D) No, because W's dower rights were subject to defeasance upon H's conveyance to C.

145. (A) At common law, a widow was entitled, on the death of her husband, to a life estate in a third of the lands of which her husband was seised in fee simple or in fee tail at any time during the marriage. This life estate of W is called the widow's estate of dower. See Smith and Boyer, Law of Property, pg. 14-15. However, the widow cannot claim dower unless the husband was seised of an estate of inheritance. The widow's right to dower cannot be defeated by any conveyance by the husband, including to a bona fide purchaser for value, unless the wife 1) joins in the conveyance or 2) releases dower. In this question, W's dower interest has not been defeated because both deeds from A are conveying estates of inheritance since the words "and their heirs" appear. Thus, a 1/3 interest in C's land passes to W upon H's death. Choice (A) is correct. Note that dower rights are still honored in some American jurisdictions.

On May 10, 1935, Max Venerable, owner of Hillsdale, a 300-acre cattle ranch in Alpine County, Texas, conveyed a 20-acre strip across the property as follows: "To the Western & Pacific Railroad, its successors and assigns, to have and to hold so long as the same shall be used for railroad purposes." In 1952, Venerable made the following conveyance: "To my daughter, Velma, and her husband, Farris, and their heirs, so much of Hillsdale as not described in my deed to the Western & Pacific Railroad dated May, 10, 1935. The following year, Venerable, a widower, died intestate, survived by two daughters, Velma and Veronica. In 1980, the Western & Pacific Railroad decided to discontinue operating its trains over the strip conveyed in 1935. Velma died in 1982, survived by her husband, Farris, his 33 year old daughter, Fain, and her two children. Her will, duly executed, devised and bequeathed her entire estate in trust. The trustee, Tim Troy, was directed to distribute the income from the trust property, which he could sell and reinvest, "to my descendants from time to time living until the death of the survivor of Fain's children", when the property was to be distributed to such descendants then living, per stirpes. By 1984, the growth of nearby Buckstown had made Hillsdale valuable as a potential site for homes or for an industrial park. However, as of January 1, 1985, the governing body of Alpine County took appropriate action, in accordance with Texas statutes, to zone Hillsdale as single family residential property with a minimum lot size of one acre. The ordinance provided that the exclusion of ranching operations and industrial development was necessary to protect the health of county residents by limiting the extent of waste disposal and preventing pollution of air and the Del Rio River, the county's major source of its water supply. The Texas State Power Authority has now taken appropriate action to condemn half of Hillsdale, which will be flooded as a result of construction of a dam for a proposed hydroelectric facility. The 150 acres taken includes the 20-acre strip described in the deed to the Western & Pacific Railroad. 147. Is the Western & Pacific Railroad entitled to any compensation from the Texas State Power Authority. (A) Yes, because if all the leased land is condemned for the full balance of the lease term, the lessee is entitled to compensation for the taking of the leasehold estate. (B) Yes, because the holder of an easement benefit is entitled to compensation for the value lost. (C) No, because by discontinuing its operation of trains, Western & Pacific's determinable fee terminated. (D) No, because the holder of an easement is not entitled to compensation when the servient tenement is extinguished by condemnation.

147. (C) First, it is necessary for students to determine what interest does the railroad company have across the Hillsdale property. The answer is a fee simple determinable, which is a fee simple created to continue until the happening of a stated event. Since the duration of the estate is cor¬related to the happening of a named event, here the ceasing to use it for railroad purposes, the estate terminates automatically by operation of law upon the happening of that event. Students should be aware that the very important characteristic of this type of estate is this: the instant it is no longer used for railroad purposes it "reverts" back to the grantor or his heirs. Remember that the following words are usually used to create a determinable fee: "until" or "so long as." Choices (B) and (D) are incorrect because the railroad company does not have an easement. By the same token, choice (A) is wrong because nothing in the deed instrument would indicate the creation of a leasehold interest.

Perry signed a contract to purchase from Vickers a tract of land. The contract was signed on May 1, 1976, and the closing is set for June 15, 1976. The land was located in a jurisdiction that has in force the following statute: Statute of Limitations—"an action to recover the possession of land shall be brought within twenty-one years after the cause thereof accrued, but if a person who is entitled to bring such action, at the time the cause accrued, is within the age of minority (under 21) or of unsound mind, such action may be brought within ten (10) years after such disability is removed." This statute was enacted in the jurisdiction in 1900. The land in question had been owned by Owens In 1930. On September 1, 1930, Owens died intestate leaving Harold as his only heir. Harold had been born on October 1, 1924. In addition, there is a deed from Owens administrator, Adams, to Vickers dated November 1, 1930, which the latter recorded on the same date. During his title search, Perry learned that Adams had never obtained any license to sell the land in question; and also he (Adams) never formally received the approval of any court with respect to the administration of Owens' estate. Moreover, Perry ascertained that Vickers entered into possession of the land in question on November 1, 1930. 148. On the assumption that there are no additional facts, Perry should be advised that Vickers became or will become the owner of the land in question on (A) November 1, 1930 (B) November 12, 1951 (C) October 1, 1955 (D) when Harold dies

148. (C) Based on the present factual situation, Harold, as Owens' only heir, would have until October 1, 1945 (when he reached majority) plus ten years in which to bring his ejectment action. Therefore, Vickers became the owner on October 1, 1955 which, in accordance with the Statute of Limitations, was the last date on which Harold could initiate an action to recover possession of the land in question.

149. Assume for the purposes of this question only that the statutory period for adverse possession in this jurisdiction is 20 years. Suppose that Vickers was ousted from the land in question by Lucas on October 1, 1950, and that he recovered possession of the land from Lucas in an action of ejectment on December 31, 1950. Perry should be advised that Vickers became or will become the owner of the land in question on (A) November 1, 1930 (B) November I, 1950 (C) October 1,1955 (D) December 31,1970

149. (D) Under the doctrine of adverse possession, for one to hold real property adversely his possession must be: (a) actual, (b) exclusive, (c) open, (d) visible, (e) notorious, (f) continuous and (g) peaceable. Since Vickers was ousted from the land, his possession would not be continuous. Therefore, Vickers will not become owner of the property (through adverse possession) until December 31, 1970. Since Adams was not in a position to convey legal title (because his administration of Owens' estate was never formally approved by the court) of the property to Vickers, the latter will not acquire legal title to the property until the expiration of his period of adverse possession.

Fegen owned two adjacent ten story commercial buildings in Beverly Hills. The buildings were respectively known as Trump Towers and Galleria Plaza. The first floors of both buildings were occupied by various retail establishments. The buildings' other floors were rented to tenants and used as offices. There was an enclosed walkway which connected the second floor of each building. Thus, shoppers and office workers could walk across the common walkway and gain access to each building. While the buildings were being used in this manner, Fegen sold Trump Towers to Toyota by warranty deed, which made no mention of any rights concerning the walkway. The walkway continued to be used by the occupants of both buildings. Thereafter, the walkway became unsafe as a consequence of wear and tear. As a result, Toyota hired a contractor to repair the walkway area. When Fegen saw the contractor removing the carpeting along the walkway, he demanded that Toyota discontinue the repair work. After Toyota refused, Fegen brought an action to enjoin Toyota from continuing the work. 15. The most likely result will be a decision for (A) Fegen, because Toyota does not have rights in the walkway (B) Fegen, because Toyota's rights in Trump Towers do not extend to the walkway (C) Toyota, because Toyota has an easement in the walk¬way, and an implied right to keep the walkway in repair (D) Toyota, because he has a right to take whatever action is necessary to protect himself from possible tort liability from persons using the walkway

15. (C) Apparently, Toyota would have an implied easement in the walkway. An implied easement is created and proved not by the words of the conveyance but by all the circumstances surround¬ing the execution of the conveyance. It is based on the intention of the parties as inferred from the surrounding circumstances. There are five distinct requirements for the existence of an implied easement all of which are present in the facts of this example. First, there must be two properties owned by one person who uses one of the pieces of property to serve the other piece of land. Second, there must be a conveyance of one part of the property to another person, the other part being retained by the conveyor. Third, the quasi-easement must be apparent at the time of the con¬veyance. Fourth, the quasi-easement must be continuous, which means that the use of the quasi-servient tenement must be permanently adapted to serve the needs of the quasi-dominant tenement. Fifth, the quasi-easement must be (a) "reasonably necessary" to the convenient enjoyment of the quasi-dominant land if that tract is the property conveyed to the grantee, and (b) "strictly necessary" to the enjoyment of the quasi-dominant tenement if that tract is retained by the grantor. By virtue of the implied easement, Toyota has the right to enter the walkway for the purpose of repairing, maintaining and improving the means by which the easement is enjoyed.

The State of Gulf Coast has the following statute in effect: "Conveyances, mortgages, and leases for terms in excess of three years, shall be valid against a subsequent grantee, mortgagee or lessee only if the first conveyance, mortgage or lease is duly recorded in the county where the land is located." With respect to mortgages, Gulf Coast is a "lien theory" state. The recording statutes do not provide for a tract index. On December 1, 1975, Owens mortgaged a ten acre tract of improved land, known as Blackacre, to Morgan to secure a note for $25,000, due on December 2, 1978, with 10% interest per annum. This mortgage was not recorded. On February 1, 1976, Owens executed a written lease of Blackacre with Lewis, which included the following provisions: "(1) Owens hereby leases Blackacre to Lewis for a term of five years commencing March 1, 1976, and terminating February 28, 1981: (2) As rent for the said premises, Lewis promises and agrees to pay Owens Five Hundred Dollars ($500.00) per month and Lewis also agrees to pay all taxes and insurance premiums on the leased premises; (3) Lewis shall have the option to purchase said premises at the expiration of said lease at a price of Forty Thousand Dollars ($40,000), payable March 1, 1981, if Lewis shall give Owens written notice of his intention to exercise the option prior to December 31, 1980." Lewis went into possession of Blackacre on March 1, 1976, but did not record his lease. On December 31,1976, Biers, purchased Blackacre from Owens for $40,000, receiving a general warranty deed. Owens orally informed Biers that Lewis had only a month-to-month tenancy. Not wishing to disclose his interest in Blackacre, Biers did not record his deed. Moreover, Owens agreed to cash Lewis' rent checks as received and turn the money over to Biers so that it would be unnecessary to tell Lewis about the sale. On June 1, 1977, the Gulf Coast Highway Department brought suit to have a three acre tract of Blackacre condemned in an eminent domain proceeding. After Owens informed Biers of the eminent domain proceeding, Biers then recorded his deed on June 3, 1977. When Lewis was informed of the condemnation, he contacted Owens, and told him that he was terminating the leasehold agreement. 150. Which is the most accurate statement concerning Lewis' obligation to pay rent following the condemnation proceedings? (A) The condemnation would not relieve Lewis of his obligation to pay the full rental under the terms of the lease. (B) The "taking" of the three acre tract under eminent domain would entitle Lewis to an abatement of the rental. (C) Regardless of the condemnation, Lewis would be relieved of his obligation to pay rent since the lease¬hold agreement was invalid. (D) Lewis would only be relieved of his obligation to pay rent if the condemner were to take the entire fee to the leased premises.

150. (C) Lewis would be relieved of his obligation to pay rent in this situation, because of the particular recording statute in effect. The statute provides that conveyances, mortgages or leases in excess of three years are invalid as against a subsequent grantee, etc. Hence, the Lewis-Owens leasehold contract could not be enforced by Biers (regardless of the condemnation). It should be pointed out, however, that the leasehold contract between Lewis and Owens was enforce¬able until Owens' conveyance to Biers.

In 1948, Elton was the actual and record owner of Forestacre, 20 acres of certain undeveloped timberland, located in northern Colorado. In September 1948, Elton mortgaged Forestacre to the Golden Bank by a mortgage deed (in the traditional form of a conveyance in fee simple subject to a condition subsequent) which was not recorded until January 1949. The mortgage deed contained the following clause immediately after the legal description of Forestacre: "together with all the real property now owned by Elton or which shall be owned by Elton during the continuance of the indebtedness secured by this mortgage." This mortgage was given, as the recorded instrument revealed, to secure a note for $100,000 repayable over a forty-year term. In March 1979, Elton, using money loaned by Nugget Finance Comp., purchased Rockyacre, a 50-acre mountainous terrain, situated outside the city of Denver. This deed was recorded immediately. In April 1979, Elton executed and delivered to Nugget Finance Comp. a mortgage to secure its loan. This mortgage was promptly and duly recorded. Nugget Finance Comp. had no actual notice of the prior mortgage to Golden Bank. In February 1982, Elton defaulted on both mortgages. Golden Bank then initiated foreclosure proceedings against Elton and Nugget Finance Comp. as joint defendants. In its foreclosure action, Golden avered that its mortgage was a first mortgage on both Forestacre and Rockyacre. Nugget Finance Comp. filed a cross-complaint alleging that its mortgage was a first mortgage on Rockyacre and praying for foreclosure on that parcel of property. 151. In order to decide this case in favor of the Golden Bank, the court must resolve which of the following issues? I. Is Nugget Finance Comp. charged with record notice of Golden Bank's mortgage. II. Does the quoted clause in the mortgage instrument to Forestacre cover Rockyacre. III. Is Nugget Finance Comp.'s mortgage a purchase money mortgage? IV. Can Nugget Finance Comp. rely on the doctrine of implied purchase money liens? (A) I and II only (B) II and III only (C) I, II and III (D) I, II, III and IV

151. (C) Questions covering mortgages are frequently tested on the MBE. By definition, a mortgage is an interest in land created by a written instrument providing security for the performance of a duty or the payment of a debt. Mortgages and assignments thereof should be recorded. Failure to record the mortgage may make it possible for the mortgagor to convey to a bona fide purchaser who would take free of the mortgage under the recording act. As such, statement I is correct. By the same token, statement II is also correct because the deed instrument stated that the mortgage would apply to property "now owned by Elton or which shall be owned" in the future. Consequently, when Elton subsequently purchased Rockyacre, Golden Bank would argue that their mortgage interest covers that property as well as Forestacre. In this regard, students should note that such "after-acquired" property clauses in mortgage instruments are enforceable as long as the after-acquired property is sufficiently identified to put third parties on notice. In addition, statement III is true because purchase money mortgages take priority over all earlier executed mortgages. Therefore, if Nugget Finance Company's mortgage is viewed as a purchase money mortgage, it would clearly take priority over the "after-acquired" property clause contained in Golden's mortgage instrument. Lastly, purchase money mortgages are subject to operation of the recording acts and must be recorded to be enforceable. As such, statement IV is not persuasive because purchase money mortgages are expressly recorded and generally not implied.

37. Assume for the purposes of this question only that 2 years after Thomas entered into the leasehold agreement with Leonard, that he subleases the house to Steve. Thereafter, Steve defaults on the rental payments. If Leonard brings an action to recover the past rent due from Steve, Leonard will: (A) recover, because privity of estate exists between a landlord and sublessee (B) recover, because there is privity of contract between a landlord and sublessee (C) recover, because there is both privity of estate and priv¬ity of contract between a landlord and sublessee (D) not recover, because privity of estate does not exist between the landlord and sublessee

37. (D) Leonard may not recover for rent due from sublessee since there is no privity of estate between landlord and sublessee. Leonard's only cause of action remains against the tenant under their original leasehold agreement.

Several years ago Howe sold the eastern two-thirds of his land to Richardson who soon thereafter constructed an abode there. One year later, Richardson cleared a path from her abode across Howe's retained land to a road that abuts the western boundary of Howe's land. This path is very convenient to Richardson's use of her land and Richardson uses it daily. The path is readily apparent to anyone. In fact, Howe stood by and watched Richardson clear it. Recently, Howe put a barrier across the path. Richardson now has brought an action to have the barrier removed. 152. The theory which would give Richardson her best chance of success would be: (A) Richardson has an easement by necessity. This would depend on the strength of the court's feeling about the use of the land. (B) Richardson has an easement by implication. This would depend on whether the convenience was sufficiently strong to amount to "reasonably necessary." (C) Richardson has an easement by prescription. This would depend on the length of time she used the path prior to the erection of the barrier. (D) None of the above.

152. (B) An easement by implication arises from the circumstances surrounding the dividing by the owner of a piece of land into two pieces and conveying one of such pieces to another. From such surrounding circumstances an inference is drawn that the parties intend the creation of an easement. In all likelihood, Richardson's best chance of success would be to show that she acquired an easement by implication, rather than an easement by necessity. This is so because a greater degree of necessity is required by the latter easement. For an easement by implication, Richardson must prove that the easement is "reasonably necessary." On the contrary, for an easement by necessity, Richardson must prove that the easement is "strictly necessary." Choice (B) is preferred because an easement by implication does not require as great a burden of proof as an easement by necessity.

Hill owned a two acre tract just outside North Plainfield in Morristown County. In 1980, he subdivided the parcel into twelve lots, numbered 1-12, which he sold to eleven different buyers. Hill retained lot 12 for himself to live on. Each deed to the eleven lots which he sold contained the following restriction: "It is an express covenant and condition that the property hereby conveyed shall not be used for other than single family residences." By 1982, the purchasers of lots 1-11 had each built residences on their property. In December, 1981, Hill dug a well on his property. From that well, Hill supplied water to all of the lot owners. In February, 1983, Beverly, who lived on the lot adjacent to Hill's, dug a well in her backyard. Her well caused water to be diverted from Hill's well, and he is no longer able to get any water. 154. In an appropriate action by Hill against Beverly, what rem¬edy, if any, is available? (A) An injunction should be decreed enjoining Beverly from using her well. (B) Hill should be entitled to recover money damages from Beverly. (C) Beverly should be required to supply water to Hill. (D) No remedy.

154. (D) Percolating water is another subject area commonly tested on the MBE. Percolating water is water beneath the surface of the earth that is not confined to a known and well defined channel or bed. The common law rule is one of absolute ownership. A landowner is not restricted in the withdrawal of percolating water located beneath the surface of his (or her) land even if this causes drainage of water from adjoining land to the damage of other landowners. CAVEAT: While the common law is applied in a majority of jurisdictions, an increasing number of states now apply the "reasonable use" rule. Under this rule all of the landowners have a correlative right with respect to percolating water located beneath the surface of their land. Since the Multistate follows the majority view, choice (D) is correct because Beverly is not restricted in the withdrawal of percolating water located beneath the surface of her land.

Question 155 is based on the following fact situation. In 1969, Chamberland was the record owner in fee simple absolute of Roundacre, a 30 acre tract of land located in the township of Forum. In 1981, Chamberland made a conveyance thereof in these words, "I hereby convey Roundacre to Elgin and Gale as joint tenants with right of survivorship." On November 11, 1983, Elgin executed a mortgage on Roundacre to West to secure a loan. Thereafter, on January 18, 1984 Elgin died intestate, leaving Erickson as his only heir at law. At the time of Elgin's death, the indebtedness had not been paid to West. The jurisdiction in which Roundacre is located recognizes a title theory of mortgages. 155. In an appropriate action, the court should determine that title to Roundacre is vested (A) in Gale, with the entire interest subject to the mortgage (B) in Gale, free and clear of the mortgage (C) half in Gale and half in Erickson, with both subject to the mortgage (D) half in Gale, free of the mortgage, and half in Erickson, subject to the mortgage

155. (D) According to the common law rule, a mortgage results in a transfer of ownership to the mortgagee (in a title theory jurisdiction). Applying this rule, a mortgage of joint tenancy property, given by one of the joint tenants, results in a termination of the joint tenancy because it terminates the unity of title as well as the unity of interest. See Burby, Law of Property, pg. 221.

Questions 16-18 are based on the following fact situation. In 1956, Obie conveyed Blackacre to John "for life, remain¬der after John's death to his heirs." Two years later John entered into a real estate agreement for the sale of Blackacre to Ed, whereby John agreed to convey the premises to Ed in fee simple absolute. Prior to the settlement date, Ed contacted John, telling him that he would not perform his part of the agreement because John could not convey a fee simple. 16. If the Rule in Shelley's Case is followed in this jurisdiction, the outcome of a suit by John for specific performance of the real estate contract would result in (A) John not succeeding, since he could not convey mar¬ketable title (B) John succeeding, because he had a fee simple to convey (C) John not succeeding, because his heirs have to join in the transaction in order to convey marketable title (D) John succeeding, because the conveyance of his life estate divested the contingent remainder of his heirs

16. (B) John would succeed because Shelley's Rule operates as follows: if a life estate is conveyed to A and in the same instrument a remainder is given to A's heirs, then A will take a remainder in fee simple. In other words, A's life estate merges with the remainder to his heirs, thus giv¬ing A a fee simple absolute.

Recreational Systems, Inc., under the authority of a statute of the State of Creenora, sued to have condemned 1000 acres of forested land owned by the Great Lakes Timber Co. which it planned to develop for use as a state recreational area and state gamelands. After a hearing, the state court ordered possession of the land surrendered to Recreational Systems, prior to determi¬nation of compensation, upon deposit in court of a sum deemed adequate to cover damages which might be awarded. Great Lakes Timber Co. immediately commenced an action to enjoin the court ordered sale of their property. 38. Indicate which of the following would be the best ground for upholding the state court's order: (A) the power of eminent domain may only be delegated directly to a private enterprise for a public related use or activity (B) the power of eminent domain may only be delegated to a public authority through a legislative determination (C) the injured party has not proved such irreparable injury to use as amounts to a "taking" (D) the Fifth Amendment power of eminent domain incor¬porated by the Fourteenth Amendment as applicable to the States does not require that payment be made prior to condemnation of the property

38. (A) The power of eminent domain may be delegated directly or indirectly to a private person or enterprise subject to the requirements that the taking be (a) for a public use and (b) just com¬pensation be given.

Knox, the owner of lot 62 in the Brunswick development, leases his home to Warner for a term of one year. Warner sells liquor on the premises, a use prohibited by Article 2, Section 1 of the Declaration. The Brunswick Association sues Warner in an action for damages. Which of the fol¬lowing is the best argument for Warner? (A) The Rule in Spencer's Case prevents Warner from being liable. (B) Warner is not in privity of contract with The Brunswick Association. (C) Warner is not in privity of estate with Knox. (D) Other lots in the immediate vicinity are used for commercial purposes.

163. (B) This is an extremely tricky Multistate example. As already mentioned, practically every Multistate question will test an area involving a very subtle hornbook distinction or nuance in the law. Here, for example, students must distinguish between legal and equitable remedies. Note that the Brunswick Association is suing Warner in an action for money damages. So, there¬fore, this is obviously a breach of covenant (or contract) action. As a consequence, Warner's best argument is that he is not in privity of contract with the Brunswick Association. That's why choice (D) is incorrect because a plaintiff seeking to enforce an equitable servitude may be denied relief on equitable grounds if the purpose of the servitude in a development scheme is impossible of attainment because of changed conditions. Be advised that choice (A) is wrong because the rule in Spencer's Case provides that an assignee of either the reversion or of the leasehold estate cannot be held liable for breach of covenant if the covenant is of a type that "does not touch and concern the land." Since Warner is a sublessee, not an assignee, of the lessee, the Rule in Spencer's case is inapplicable.

165. Alston, the owner of lot 24 in the Brunswick development, sells his land to Reese. The deed contains the following provisions: "These premises are conveyed to Reese, his heirs and assigns so long as they are used for residential purposes." This deed is not recorded. Thereafter, Reese sells liquor on the premises. The Brunswick Association brings an action of ejectment against Reese. Which of the following is the best argument for Reese? (A) The deed to Reese created a fee simple subject to a condition subsequent, and no right of entry has been asserted by The Brunswick Association. (B) Not having been recorded, the condition cannot be enforced against Reese. (C) The Brunswick Association is entitled only to an injunction against Reese. (D) The law prohibits a fee simple determinable.

165. (C) According to the prevailing view, transferees of the original parties to an equitable servitude are bound by the servitude if it is intended to bind the land, and not merely the persons, and the benefits and burdens of the servitude are intended to run to such transferees. As such, an equitable servitude may be enforced against any person who takes land with either actual or constructive notice that such land is burdened with a restriction on its use. Since the violation of an equitable servitude is remedied in equity (e.g., by injunction), choice (C) is the best answer. Note that choice (B) is wrong because despite the fact that the deed was not recorded, the restriction would, nevertheless, be enforceable because Reese would have constructive notice of the servitude. Smith and Boyer point out that a declaration of restrictions which is duly recorded, evincing an intention to restrict the use of the lots in a subdivision to residential purposes and disclosing an intention to set forth a comprehensive development scheme for the sub¬division, is competent to create an equitable servitude on each lot within the tract and each owner of a lot therein is bound by such restrictions. Real Property, pg. 379.

Hessler lived in a single-family dwelling within the city limits of Boulder. Adjacent to his home was a vacant lot that measured 100 feet by 175 feet. The lot, which Hessler owned, was situated on the corner of Davis Street and University Way. The tract measured 100 feet along Davis Street and 175 feet along University Way. In 1996 Hessler executed a deed purporting to convey the vacant lot to Wydick for the consideration of $28,000. After escrow, the deed was delivered to Wydick who immediately filed it with the Recorder's Office. The recorded instrument described the property conveyed as "all the tract of land beginning at the northwest corner of Davis Street and University Way; thence west along Davis Street 100 feet; thence north 175 feet; thence west 100 feet; thence south 175 feet along University Way to the place of beginning. Three months after his conveyance to Wydick, Hessler died intestate. His heirs have now filed an appropriate action contesting Wydick's title to the vacant lot. 166. Which of the following statements is most accurate concerning the outcome of this suit? (A) Hessler's heirs will prevail because metes and bounds, rather than streets, are appropriate boundary descriptions. (B) Wydick will prevail because equity will not permit a forfeiture for a mere technicality. (C) The outcome will depend on whether the tract of land was plotted as a lot in a subdivision. (D) The outcome will depend on whether the last call (175 feet along University Way) prevails over the third call (west 100 feet).

166. (D) The area of land descriptions and boundaries has frequently been tested on the MBE. As a general rule, no conveyance is valid unless the description of the land sought to be conveyed is sufficient to identify the land. In this question, the deed from Hessler to Wydick purports to convey a 100 foot by 1 75 foot lot, but the recorded instrument describing the property fails to describe an enclosed area, since the third call goes in the wrong direction. In the heirs' action contesting Wydick's title to the vacant lot, the outcome will depend on whether the last call— which establishes an enclosure by stating that the boundary line returns "to the place of beginning"—prevails over the ambiguous third call (west 100 feet). Choice (D) is correct. Note: When a deed describes the boundaries of the land to be conveyed by reference to monuments, natural or artificial, the intention of the parties is the controlling factor and all rules of construction are mere aids in determining such intention. Smith and Boyer, Real Property, p. 300. Choice (C) is incorrect because had the tract of land been platted in a subdivision it would have been described by a specific parcel number or lot number, not by a boundary description. Choice (B) is incorrect because an improper boundary description is never considered a "mere" technicality and if no land is enclosed, the result may be an invalid conveyance. Choice (A) is incorrect because street descriptions (i.e. monuments) are appropriate boundary descriptions.

Question 167 is based on the following fact situation. Kitty and Jessie lived together in a one-bedroom apartment in West Los Angeles. They were engaged to be married when they purchased an undeveloped parcel of land on a hillside over-looking Malibu. The deed, which was properly executed and recorded, named Kitty and Jessie as grantees "not as tenants in common but as joint tenants with right of survivorship." Thereafter, Jessie, who was experiencing financial difficulties, offered to sell the property to Luca. Without Kitty's knowledge, Jessie executed a deed that purported to convey the hillside property to Luca in fee simple. Jessie signed his name and forged Kitty's name. He then delivered the deed of conveyance to Luca, who paid Jessie $150,000 which was the fair market value of the property. Luca immediately recorded the deed received from Jessie. Note: the common-law joint tenancy is unmodified by statute. 167. Title to the property in question is now in (A) Jessie and Kitty as joint tenants with right of survivorship (B) Luca and Kitty as joint tenants with right of survivorship (C) Luca and Kitty as tenants in common (D) Luca as owner in fee simple

167. (A) According to Smith and Boyer, a forged or an undelivered deed is a nullity and no one can claim any interest through such. Placing such a deed on record does not add any legal efficacy to such a forged or undelivered instrument. The recording statutes are not intended for the purpose of assisting wrongdoers, tortfeasors, criminals and forgers in depriving innocent own¬ers of their real property. The original owner continues his ownership over one who claims even as an innocent purchaser through a forged or an undelivered deed.

82. If Molly died without issue before the beginning of Austin's oil drilling operations, the likelihood of Rosalina's success in her lawsuit against Austin would probably have been (A) unaffected, because the right to remove oil is an incident of the right to possession (B) unaffected, because the nature of Austin's estate would not be altered by Molly's death (C) improved, because Molly's death without issue would convert Austin's fee into a reversionary interest (D) improved, because although Austin still has a fee, it would now be certain to terminate

82. (D) Here's a rather confusing Multistate example where all four answer choices are technically incorrect. However, by process of elimination alternative (D) is the "best of the worst." Choices (A) and (B) are incorrect because once Molly died without issue, Rosalina's shifting executory interest "vested" and Austin's fee was cut off. Choice (C) is wrong because Austin's fee does not "revert" back to the grantor-Longhorn but shifts to the benefit of the grantee-Rosalina. Thus, choice (D) is the best answer even though Austin's fee terminated at Molly's death.

Question 168 is based on the following fact situation. W, a widower, died in 1972 leaving $100,000 to T Trust Company in trust to pay the income to S, his son, for life. S, who was married to M, had two children, A and B. W's will provided in part: "The net income from this trust shall be paid to my son S, for his life. Upon S's death, the net income is to be paid to any widow of S. Upon the death of S's widow, the T Trust Company shall then pay the income (from said trust) to the living children of my sister, H, in equal shares." H's only surviving child, G, was born in 1976. Both S and M died in an airplane crash in 1984. There is no statute modifying the common law in this jurisdiction. 168. H, on behalf of G, brings an appropriate action against the T Trust Company and W's estate to allow the distribution of the income from said trust to be paid to G. Is H likely to prevail in this action? (A) No, because G was not a life-in-being at the time of W's death. (B) No, because the provisions under which G was intended to take violate the Rule Against Perpetuities. (C) Yes, because that was the intent of W. (D) Yes, because all other persons who would have had any claim to the income from the trust corpus are deceased.

168. (B) The Rule Against Perpetuities in its simplest form provides: "No interest is good unless it must vest, if at all, not later than twenty-one years after some life in being at the creation of the interest". Choice (B) is correct because postponing the vesting of an interest until after the death of the "wife" or "widow" of a living person violates the Rule Against Perpetuities. The rationale is that the eventual "wife" or "widow" may be a person not in being at the effective date of the gift. See Burby, Real Property, pg. 416.

Question 169 is based on the following fact situation. Wendell owns a summer resort near Arrowhead Lake. Wilkie owns a piece of waterfront property located between Wendell's land and the lake. For $25,000, Wilkie granted Wendell an easement of way for any guests staying at his (Wendell's) resort to cross Wilkie's property during the months of June, July and August. Four years later, Wendell sold his resort to Willard who converted it into a winter resort. Wilkie then erected a fence stopping Willard's guests from crossing his land to use the lake for ice skating. Willard brings an action to have the fence removed. 169. Willard is most likely to prevail if: (A) At the time of the grant Wendell's property could reasonably have been expected to have been developed as a winter resort. (B) The capacity of the summer resort was175 guests and the capacity of the winter resort is 95. (C) Willard had bought the property immediately after the easement had been granted and Wilkie had allowed winter guests to use the easement for the entire four years. (D) None of the above.

169. (D) Excessive use of an easement does not forfeit or extinguish the easement. In the case of excessive use the owner of the easement is simply making use of the servient tenement beyond the scope or extent of the use permitted by the easement in its creation. The easement as it was originally created still exists and within its scope can be used. But the use in excess of the scope of the easement can be enjoined and damages may be had for injury caused by the excess use. In the present case, Wilkie granted Wendell an easement of way (for any guests staying at Wendell's resort) to cross his (Wilkie's) property during the months of June, July and August. Certainly, it is beyond the scope of the easement to cross Wilkie's property during the winter months to use the lake for ice-skating. Choice (C) is incorrect because the mere fact that Wilkie had permitted Willard's guests to use the easement during the winter months for four years does not create an easement by prescription. To acquire an easement by prescription the use must be continuous and uninterrupted for the statutory period of prescription (which at common law was 20 years).

Assume for the purposes of this question only that Obie conveyed Blackacre to John "for life, with remainder to Obie's heirs." Later John conveyed Blackacre to Sam "for as long as he would continue to farm the property." After John's death, Obie's heirs brought suit against Sam to quiet title. Which Common Law doctrine would be most appli-cable in the court's determination of the ownership of Blackacre? (A) Rule of Wild's Case (B) Doctrine of Destructibility of Contingent Remainders (C) Doctrine of Worthier Title (D) Rule against Remainders in the Grantees' Heirs

18. (C) Choice (C) is correct because the old common law Doctrine of Worthier Title is construed today as a rule of construction whereby the grantor presumes not to create a remainder in his heirs, but rather intends to retain a reversion in himself. Refer to Justice Cardozo's opinion in the leading case of Doctor v. Hughes, 22 N.E. 211,1919.

Question 180 is based on the following fact situation. Able owned Blackacre, a vacant tract of land. On June 1, Able sold Blackacre to Baker and provided Baker with a warranty deed. On June 2, Credita recorded a judgment lien against Able. The jurisdiction in which Blackacre is located has a recording statute that provides that recorded judgment liens will attach to all property owned by the judgment debtor as against subsequent purchasers for value. On June 7, Baker recorded the warranty deed Baker received from Able. When Credita attempted to enforce her judgment by execution on and sale of Blackacre, Baker brought a quiet title action. 180. If the court rules in favor of Baker it will be because (A) Baker's one-week delay in recording was not significant. (B) Baker took title under a warranty deed. (C) All deeds take priority over claims of judgment creditors. (D) The judgment lien never attached to Blackacre.

180. (D) is correct. In most jurisdictions, a judgment lien (once recorded) becomes a lien on all of the defendant's real property in the county where the lien is recorded. The problem is whether the defendant's real property includes real property that the defendant has transferred by an unrecorded conveyance. Most courts hold that such a lien only attaches to property the defendant actually owns—not inclusive of property conveyed by recorded or unrecorded conveyance. Here, Able conveyed Greenacre to Baker before Credita recorded her judgment lien. Thus, the judgment lien never attached to Blackacre.

Norton owned and operated the Roadhouse Cafe in Sedalia. The restaurant was located at the busy intersection of Routes 66 and 99. On account of its prime location, the Roadhouse Cafe was a popular truck stop for both motorists and truckers. In 1995, Norton leased the restaurant to Samuels under a writ¬ten seven-year lease. During the third year of the lease term, the state constructed a new highway that bypassed the leased property. As a consequence, practically all of the traffic from Routes 66 and 99 was re-routed along the new highway. This resulted in a substantial reduction in the Roadhouse Cafe's business. At the time Norton and Samuels entered into their leasehold contract, neither party foresaw or could have anticipated the state's action. Under these unfavorable circumstances, Samuels requested that Norton renegotiate the lease. Norton, a tough businesswoman, refused to do so. In the beginning of the fourth year of the lease term, Samuels, who was now operating the restaurant at a loss, decided to abandon the Road¬house Cafe and has since refused to pay any more rent to Norton. 181. If Norton brings suit against Samuels to recover the unpaid rent, judgment for whom? (A) Norton, because the doctrines of impossibility and frustration of purpose do not apply to real estate transactions. (B) Norton, because the court will not infer that continued existence of heavily traveled roadways at the restaurant site was an assumption upon which the parties contracted. (C) Samuels, because the purpose of the leasehold contract was frustrated, it would be inequitable for him to continue to make long-term rental payments. (D) Samuels, because in the event of an unforeseen change of circumstances, long-term leasehold con¬tracts are impliedly subject to renegotiation.

181. (B) This question covers an interrelationship of Property and Contract law. First, it is necessary to remember that a lease is a contract. The modern trend is toward treating leases more as a contract than as a conveyance, and applying contract rather than property doctrines to the landlord-tenant relationship. Under the common law, however, landlord-tenant rules were governed by property rather than contract principles. Thus, a tenant's liability for rent was not affected by physical destruction of the leased premises (either by fire, flood, or any other act of God). By analogy, the contract doctrine of frustration of purpose (e.g., where property is leased for a specified purpose that later becomes impossible to fulfill because of unforeseeable changes) does not, in the absence of a stipulation in the lease, relieve the tenant from his/her obligation to pay rent according to the common law rule. Therefore, choices (C) and (D) are incorrect. Likewise, choice (A) is not the best answer because in most jurisdictions the doctrines of impossibility and frustration of purpose do apply. For example, if one apartment is leased in a large apartment house and the entire building is destroyed by fire, the tenant under modern law is relieved from his duty to pay rent. By process of elimination, choice (B) is correct.

Pope, a devout Roman Catholic, was the owner of South Bend, a 100-acre tract of undeveloped land. In 1994, Pope executed a complete deed purporting to convey South Bend "to the executives of St. Bonifazieus German Roman Catholic Church on condition that the land be used for church and school purposes." Thereupon, he placed the deed in his safe deposit box in the bank. In 2000, Pope died and the deed to the Church was found. In the safe deposit box Pope also left a will bequeathing all of his personal property to Genevieve, a daughter, who was Pope's sole surviving heir. In the will, Pope named Attucks, his attorney, as executor and instructed him to deliver the deed to the St. Bonifazieus German Roman Catholic Church upon his death. A dispute then arises between Genevieve and the Church as to who is the owner of South Bend. 182. In an appropriate action to quiet title to South Bend, who is the rightful owner? (A) Genevieve, because there was not an effective delivery of the deed during Pope's lifetime. (B) Genevieve, because the deed is void due to the fact that the grantee is not reasonably identifiable. (C) The St. Bonifazieus Church executives, because a posthumous deed delivery is effective to convey title. (D) The St. Bonifazieus Church executives, because Pope clearly intended to make a conveyance for a charitable purpose.

182. (B) A deed is not operative if the grantee is not designated with reasonable certainty. For example, a conveyance to an unincorporated group or association will fail unless the membership is relatively small and readily ascertainable. In Rixford v. Zeigler, 88 P. 1092 (1907), the grantee was named as the "Community styling itself as the German Roman Catholic St. Bonifazieus Church." The conveyance failed for indefiniteness. For the same reason, choice (B) is correct because "executives" of a church or organization covers a broad group and may be subject to dispute.

Question 183 is based on the following fact situation. In 1990, Gaye was the record owner in fee simple absolute of Blackacre, a 20-acre parcel of land in Motown. A deed executed by Gaye in 1991 conveyed Blackacre to "Ross for life then to Turner and her heirs." In 1991, Ross leased Blackacre to Franklin for four years and Franklin went into possession. Under the terms of the lease, Franklin covenanted to pay Ross a monthly rental of $500. At the beginning of the second year of the lease (1993), Ross died in an automobile accident. There is no applicable statute in this jurisdiction. 183. Which of the following best describes Franklin's present rights and obligations? (A) Franklin must vacate on Turner's demand. (B) Franklin must vacate on Turner's demand, but if Turner ousts Franklin from the land then Franklin can recover from Ross' estate for breach of the covenant of general warranty. (C) Franklin has the right to remain on the land until the end of the leasehold term, provided timely payment of rent is made to Turner. (D) Franklin has the right to remain on the land only until the end of the month for which Franklin has paid rent.

183. (A) Gaye conveyed a life estate to Ross, and Turner received a vested remainder. The com¬mon law imposed four requirements for an interest to qualify as a remainder: (1) it must be in favor of a transferee who is one other than the conveyor; (2) it must be created at the same time and in the same instrument as its preceding freehold estate; (3) the preceding freehold estate must be of lesser duration than the conveyor's interest at the time of the conveyance so that an interest can pass in remainder; and (4) it must take effect, if at all, as a present possessory inter¬est at the natural and regular termination of the particular preceding freehold estate. Smith and Boyer, Survey of the Law of Property, p. 81. Under these facts, the fourth requirement would signify that Turner's vested remainder would become presently possessory at Ross' death, not at the end of Franklin's lease. Natural termination of Ross' life estate, a freehold estate, deter¬mines when Turner's possession begins, not termination of Franklin's estate for years, since an estate for years is defined as a non-freehold estate and therefore cannot determine when the remainder becomes possessory. Choice (A) is correct. Choice (B) is incorrect because a covenant of general warranty — which defends the grantee from lawful claims of the grantor or third parties who would evict the grantee — is a deed covenant and generally not contained in a lease.

Jasper was the record title owner of Wintergreen, a 300-acre tract, located in Perrytown. Jasper inherited the property from his father, Trevor, and lived on Wintergreen for over 60 years. As he advanced in age, Jasper found it more difficult to properly maintain the estate. As a consequence, in 1984 Jasper decided to retire to Florida and to convey Wintergreen to his sister, Alexis. Jasper thus executed a valid deed conveying the property "to my sister, Alexis, during her lifetime, with remainder to the children of my cousin, Marvin." At the time of this conveyance, Marvin was unmarried and did not have any children. Alexis took possession of Wintergreen and resided on the property from 1984 to 1993. In 1989 Marvin married Molly. In 1993 Alexis died leaving her husband, Lanny, and daughter, Dale, as her surviving heirs. When Alexis died Marvin had two children, Candace and Clover. In 1995 Marvin and Molly had a third child, Zeke. 185. Marvin has now instituted an appropriate action to determine the state of title to Wintergreen with all minors duly represented. The court should rule that title belongs to (A) Candace and Clover only (B) Candace, Clover and Zeke only (C) Candace, Clover, Zeke and any additional children that Marvin may have during his lifetime (D) Candace, Clover, Zeke and Dale, each owning an equal share per stirpes

185. (C) At the time of the conveyance, the remainder to Marvin's children was contingent because they were not in existence. However, a contingent remainder becomes a vested remainder if any condition precedent is fulfilled and if the remainderman is ascertained before the termination of the preceding estate. Moynihan, pg. 123. In this hypo, at the time of Alexis' death Marvin had two children, Candace and Clover. As a result, their remainder becomes vested but "subject to open" if after born children to Marvin come into existence. As a result, choice (C) is correct. Choice (D) is incorrect because Alexis is only given a life estate so her heirs have no inheritable interest.

Question 186 is based on the following fact situation. Macon Ford was an extremely wealthy investment banker who owned substantial real estate interests in the Atlanta area. In 1975, Macon inherited the bulk of his father's estate which included Blackacre, a rambling "farm" of woods and fields in upstate Georgia. Marrietta Ford, Macon's sister, inherited very little from her father, and, embittered, had not spoken to Macon since the funeral. Twenty years later, Macon arranged a meeting with his sister. Feeling pangs of guilt, Macon executed an instrument in the proper form of a deed purporting to convey Blackacre to Marrietta "for and during her natural life and thereafter to the then living daughters of Marrietta." Macon handed the instrument to Marrietta. Still bitter and resentful, Marrietta tore the document into little pieces, threw them at Macon, and said, "Macon, my family doesn't need your charity." Macon replied, "Even though you may not, your daughters certainly do." A short time later, Marrietta died in an automobile accident and was survived by her two daughters, Omni and Augusta. After learning the foregoing facts, Omni and Augusta now claim ownership in Blackacre. 186. In an appropriate action to quiet title to Blackacre by Omni and Augusta against Macon, if Omni and Augusta prevail, it will be because (A) Macon did not accept Marietta's rejection of the deed (B) Marrietta's attempted rejection was not by an instrument in writing (C) even though Marrietta refused to accept the deed, her rejection is not attributable to Omni and Augusta (D) the deed was irrevocable because a right to revoke was not expressly reserved

186. (C) This question deals with acceptance of a deed by the grantee. The general rule is that acceptance will be presumed if the conveyance is beneficial to the grantee. Although Marrietta made the decision to reject her life estate, she did not have the power to terminate the vested remainder given to her two daughters. Thus, choice (C) is correct. Choices (A) and (B) are incorrect because neither answer addresses the rejection being attributable to Omni and Augusta.

Question 187 is based on the following fact situation. Aunt Amy owned record title to Amity Villa, a 30-unit apartment complex, in fee simple. Amy orally promised to convey the property to her nephew Nathaniel when he graduated from college and obtained his degree. On May 10th, Nathaniel graduated from college and received his B.S. degree. On May 20th, Amy threw a graduation party for Nathaniel at her home. During the party, Nathaniel was sitting in Amy's study making a telephone call. He opened the desk drawer looking for a writing utensil. As he was rummaging through some papers, Nathaniel found a deed to Amity Villa that was signed and executed by Amy naming him as the grantee. He took the deed and recorded it at the Recorder's Office. This jurisdiction has the following recording statute in effect: "No deed or other instrument in writing, not recorded in accordance with this statute, shall affect the title or rights to, in any real estate, of any devisee or purchaser in good faith, without know¬dge of the existence of such unrecorded instruments." 187. In a suit to quiet title to Amity Villa with all interested parties represented, the court should rule in favor of (A) Aunt Amy, because there was never a valid delivery of the deed to Nathaniel. (B) Nathaniel, because Developer failed to record the deed instrument as required by statute. (C) Nathaniel, because based on Aunt Amy's prior representations coupled with her execution of the deed, a valid conveyance occurred. (D) Developer, because he would acquire title as a sub¬sequent bona fide purchaser.

187. (A) In order to have a valid conveyance, there must be a delivery of the deed. Delivery of a deed means an intention on the part of the grantor that it shall operate or take effect as a conveyance. If the grantor intends the deed to be effective, delivery takes place irrespective of whether the physical paper be in the possession of the grantee or a third person. In this question, the facts clearly indicate that Amy changed her mind and did not intend to make the con¬veyance to her nephew Nathaniel. Therefore, choice (A) is correct. Choice (C) is wrong because the earlier execution of the deed did not ipso facto effectuate a valid conveyance. In fact, Amy planned to destroy that deed but was unable to find it because Nathaniel had taken it without permission.

Question 188 is based on the following fact situation. Leona and Harry Helms owned Greenwichacre, a pastoral 80-acre tract in upstate New York, in fee simple as tenants by the entirety. After Harry died, Leona executed and delivered a deed conveying Greenwichacre "to my daughter, Harriet, her heirs and assigns, but if she fails to have a child within twelve years of the date of this deed, then to my friend, Marty, his heirs and assigns." Two years later, Leona, who had been convicted of tax evasion, died in prison after being stabbed to death by another prisoner. At the time of her death, Leona's only surviving heir was Harriet, who had no children. 188. Harriet's interest in Greenwichacre is (A) a fee simple by deed (B) a fee simple subject to defeasance (C) a fee simple by inheritance and merger (D) an estate for years, plus an executory interest

188. (B) The wording of a deed conveying Blackacre "to A and her heirs" creates a freehold estate in the form of a fee simple absolute. If additional words of limitation are used to denote some form of future interest in either the grantor or a third party, then the fee simple interest is defea¬sible, since the present possessory interest may be divested upon the happening of some event. In this question Leona's conveyance of Greenwichacre "to my daughter Harriet, her heirs and assigns,..." creates a fee simple. Choice (A) is incorrect because the fee simple is subject to defeasance by Marty, his heirs and assigns if Harriet remains childless for the next 12 years. Choice (B) is correct because Harriet's interest is a fee simple subject to defeasance. Choice (C) is incorrect because there is no future interest called a fee simple by inheritance and merger. Choice (D) is incorrect since by definition an estate for years, a non-freehold estate, cannot be followed by an executory interest.

189. Which of the following items will Glendon NOT be en¬titled to remove prior to his closing on the property? (A) The dishwasher only. (B) The chandelier and dishwasher. (C) The dishwasher, garbage disposal unit and tractor. (D) The dishwasher, garbage disposal unit and chandelier.

189. (D) On the July, 2000 MBE a chandelier was considered to be a fixture. According to Burby, "electric light fixtures may be classified as fixtures." Apparently chandeliers are viewed as electric light fixtures for exam purposes. Next, Burby states that kitchen items such as "gas stoves and refrigerators" are also classified as fixtures. As a consequence, the dishwasher and garbage disposal unit will also be classified as fixtures. MBE Exam Tip: Don't be distracted by the fact that the garbage disposal unit is "easily removable." Even a refrigerator is removable. Nonetheless, kitchen units are viewed as fixtures.

Rockne Faust owned Scenicacre, a 40 acre tract of farmland, which was located in the township of South Bend. Rockne leased the property and building thereon to Dan Devine for a term of seven years commencing on February 15,1970 and ter¬minating at 12 noon on February 15, 1977. The lease contained the following provision: "Lessee covenants to pay the rent of $1,000 per month on the fifteenth day of each month and to keep the building situated upon said leased premises in as good repair as it was at the time of said lease until the expiration thereof." The lease also contained a provision giving Devine the option to purchase 10 acres of Scenicacre for $30,000 at the expiration of the lease term. Before the lease was executed, Faust orally promised Devine that he (Faust) would have the 10 acre tract surveyed. During the last year of the lease Devine decided to exercise the option to purchase the 10 acres of Scenicacre. Without Faust's knowledge, Devine began to build an irrigation ditch across the northern section of the property. When Devine noti¬fied Faust that he planned to exercise the option, Faust refused to perform. Faust also informed Devine that he never had the 10 acre tract surveyed. 19. If Devine brings suit for specific performance, which of the following is Faust's best defense? (A) The option agreement was unenforceable under the parole evidence rule. (B) Faust's failure to survey the 10 acre tract excused him from further obligations under the contract. (C) The description of the property was too indefinite to permit the remedy sought. (D) The option was unenforceable because it lacked sepa¬rate consideration.

19. (C) Smith and Boyer note that no conveyance is valid unless the description of the land sought to be conveyed is sufficient to identify the land. In this example, the facts indicate that Scenic-acre is a 40 acre tract of farmland. The leasehold agreement provided that Devine would have an option to purchase 10 acres of Scenicacre. Since the lease failed to identify or describe a dis¬tinct piece of Scenicacre, Faust's best argument is that the option should fail for lack of description. Law of Property, pg. 300.

Faith Farpo owned a small shopping center in Rosemead and an adjoining tract which was used in part for parking for customers of the center and in part was vacant. Faith got an attractive offer for a section of the adjoining tract from Roger Reek who wanted to construct a drive-in theatre on it. How¬ever, she was concerned that if she made the sale, it might unduly restrict parking for the center. In addition, any use for retail trade would compete with stores in the center. So she agreed with Reek on the following provisions which were included in the deed which was recorded on January 16, 1991: "This deed is subject to the following express covenants and conditions, which shall run with the land and be binding upon any successor or assign of the grantee herein named, and in favor of the grantor, her heirs and assigns: 1. Not less than one-half of the premises herein conveyed shall be maintained as a parking lot for use during the day by patrons of the ad¬joining shopping center, and no charge shall be made for such use. 2. No part of the premises herein conveyed shall be used for the sale of merchandise, except for such sales as are incidental to the operation of a theatre. 3. If at any time before the year 2222 the grantee has a bona fide offer for purchase of said premises, which he wishes to accept, he shall submit such offer to the grantor, who shall have a right of first refusal within 30 days of such submission to purchase said premises on the terms contained in such offer." Reek constructed the drive-in theatre but after a few years it became unprofitable. He has offered the property to Westin Hotel, Inc. The officers of Westin would like to buy it. They believe that Faith Farpo would not exercise her right of first refusal if they made an offer. In fact, she is now negotiating for the sale of the shopping center. However, they are concerned about the possible effect of the provisions in the deed from Farpo to Reek. 190. Would the provision for parking and the ban on retail sales be binding upon Westin if it acquired the property? (A) Yes, such restrictions would be enforceable as affirmative covenants running with the land. (B) Yes, such restrictions would be enforceable as equitable servitudes. (C) No, because such restrictions would be unenforceable on grounds of public policy as unreasonable restraints of trade. (D) No, because such restrictions would be unenforceable as direct restraints upon alienation.

190. (B) The restrictions set forth in the January 16, 1991 deed from Farpo to Reek would be enforceable as equitable servitudes. An equitable servitude is a restriction on the use of land enforceable in a court of equity. An equitable servitude is more than "a covenant running with the land in equity" because it is an interest in land. It is enforceable against a transferee of one of the original parties if the following elements are present: 1) notice of the existence of the servitude (either actual, inquiry, or constructive notice will suffice); 2) intent that there be such a restriction on the use of the land; 3) a showing that the restriction touches and concerns the land; and 4) an instrument which complies with the statute of frauds. See Smith and Boyer, Law of Property, pg. 366-369. Farpo's deed instrument was recorded, thus providing constructive notice to subsequent transferees. The restrictions — to maintain at least half the premises as a parking lot and to ban retail sales — took place on the covenantee's burdened land and bene¬fitted the grantor Farpo's land (i.e., "touch and concern" is satisfied). The intent that the con¬veyed premises be restricted is evidenced from the language of the deed which states that the restrictions "run with the land and be binding upon any successor(s)." Choice (B) is therefore correct.

Question 192 is based on the following fact situation. Betsy and Bing Blout owned Devonshire, an antebellum mansion located in the colonial section of Savannah. Betsy and Bing purchased Devonshire in 1962 and held title to the property as "joint owners in fee simple." This jurisdiction does not recognize a tenancy by the entirety. The Blouts had twin daughters, Millie and Tess. After Millie and Tess got married, Bing conveyed an undivided one-fourth interest in Devonshire to each daughter. 192. Title to Devonshire is now in (A) Betsy one-half, Millie one-fourth, Tess one-fourth, as tenants in common (B) Betsy and Bing one-half, as joint tenants with right of survivorship; Millie one-fourth and Tess one-fourth, as tenants in common with Betsy (C) Betsy one-fourth, Bing one-fourth, Millie one-fourth, Tess one-fourth, as tenants in common (D) Betsy and Bing as joint tenants with right of survivorship

192. (B) At common law, a conveyance to two or more persons may result in the creation of a joint tenancy unless there is expression of a contrary intent. Conversely, under modern law, by statute, a conveyance of property to two or more owners results in a tenancy in common "unless expressly declared to be a joint tenancy." Burby, Real Property, p. 215-216. In this question no statute is given. However, Betsy and Bing are husband and wife, a fact which would otherwise create a tenancy by the entirety with a right of survivorship, but for the jurisdiction's failure to recognize such an estate. Since Betsy and Bing took the same title ("joint owners") at the same time with each expressly owning the same fee simple interest and an equal right to undivided possession of Devonshire, a joint tenancy (not a tenancy in common) appears to have been created by the wording "joint owners in fee simple." Therefore, the conveyance by Bing of a one-fourth interest to each of his two daughters, Millie and Tess, severed the joint tenancy to the extent of the interest conveyed. Each daughter will hold an undivided one-fourth interest as a tenant in common with Betsy. However, the other one-half interest in Devonshire remains unaffected as to the unities of time, title, interest and possession and as to the right of survivorship. Therefore, Bing and Betsy remain as joint tenants as to this one-half interest. Choice (B) is correct.

Royce Royal was a third-year law student at Tulane in New Orleans. During law school he leased a three bedroom home from Kay Paul. Royce and Kay entered into a three-year writ¬ten lease for a $750 monthly rental. After the lease was signed, Royce asked Kay for permission to make improvements on the property. Kay gave consent and told Royce he could remove any improvements at the expiration of the lease. After moving into the home, Royce installed wall-to-wall carpeting in the living room. He then purchased a new built-in stove and freestanding refrigerator which he installed in the kitchen. Royce also paid Orleans Window Co. to replace the old windows with storm windows. During the last month of Royce's lease, Kay sold the rental home to Commander Cody. When he was ready to vacate the premises, Royce told Commander that he would be removing all the improvements which he made on the premises. Commander responded that he would not permit Royce to remove any fixtures which are deemed part of the realty. 193. In all likelihood Royce will be allowed to remove (A) all of the items which he installed (B) all of the items except the carpet and storm windows (C) the refrigerator only (D) none of the items

193. (A) Fixture questions have appeared on the last three administrations of the MBE. A fixture is a chattel which has become part of the realty. In order to determine whether a chattel is a fixture, the following factors will be taken into account: (1) the intent of the annexer; (2) the nature of the chattel; (3) the mode of annexation; and (4) the extent of the damage the removal will cause the property. For anyone picking (B) or (D), your mistake was due to reading comprehension not knowledge of the law. Certainly, the wall-to-wall carpeting, built-in stove and storm windows would likely be classified as fixtures. However, the facts clearly state that the landlord gave Royce consent to remove any improvements which he made on the premises. Therefore, choice (A) is correct.

Calipari relocated to Newark and was in the market for a new home. Calipari asked Van Horn, a real estate agent, to look for a three bedroom single-family home under $175,000. Van Horn found a two-story dwelling in a nice residential neighborhood listed for $155,000. After inspecting the property, Calipari decided to purchase the home and made an offer for $150,000 which Cassell, the seller, accepted. Calipari made a $50,000 down payment and obtained a mortgage from the Trinity Trust Company for $100,000 to finance the balance of the purchase price. Thereafter, Calipari was involved in an automobile accident and broke his pelvis. Since he could no longer walk up and down the stairs, he decided to sell his home and move into a single-story dwelling. Calipari then sold the home to Kittles for $165,000. The terms of the sale provided that Kittles would pay Calipari $75,000 in cash and assume the existing mortgage held by Trinity which now showed a principal balance of $90,000. Three years later, Kittles lost his job and discontinued making payments on the mortgage. Trinity then instituted a fore¬closure action on the property and served Kittles with notice of such proceedings. Trinity, however, did not serve Calipari with notice of the foreclosure. One week before the scheduled foreclosure sale, Kittles quitclaimed the property to Trinity in lieu of the foreclosure. At the time of the quitclaim, the property had a market value that was substantially less than the outstanding mortgage indebtedness. This jurisdiction has not enacted any anti-deficiency legislation. 195. Assume for the purposes of this question that Trinity, without reopening the foreclosure, now brings suit for a deficiency judgment against Calipari. The court should rule that Calipari is (A) liable, because Calipari stands as surety for the unpaid balance of the mortgage debt (B) liable, because Calipari was not a party to the quit¬claim conveyance (C) not liable, because Calipari was not served with notice to the foreclosure action (D) not liable, because by accepting the property in lieu of foreclosure, Trinity, as mortgagee, effectively discharged the outstanding debt obligations that were owed by both Calipari and Kittles

195. (A) Foreclosure by sale under court direction is usually regulated by statutes which vary from state to state. Equity courts have power, apart from statute, to decree and control the process. Statutes or court decrees usually call for a public sale with requirements for giving notice of time, place, manner and terms of sale. This is a rather ambiguous Multistate mortgage question because Calipari, the original mortgagor, apparently was not given notice of the foreclosure sale. Nevertheless, that by itself would not extinguish his debt obligation. When the mortgagor conveys land to a grantee who assumes the mortgage, the assuming grantee becomes primari-y liable for the mortgage debt. The mortgagor's liability is secondary or similar to that of a surety. Choice (A) is thus correct because Calipari remains secondarily liable for the mortgage debt regardless of whether he received notification of the foreclosure.

Waldo Wolfson owned a 100-unit apartment building in Miami Beach overlooking the ocean. Waldo had a brother, Warren, who lived in Asbury Park. Warren's daughter, Willow, planned to move to Florida after being accepted at University of Miami Law School. Warren contacted Waldo and asked whether Willow could rent one of his apartments at a reduced rental. Since Willow was his favorite niece, Waldo told Warren that he would rent a one-bedroom apartment to her for $300 per month. Waldo normally rented one-bedroom apartments in his building for $1,000 per month, which was the fair market value. Willow agreed to the rental fee and entered into a one year lease with Waldo beginning August 1, 1996 to July 31, 1997 at the rate of $300 per month. There was no provision in the leasehold agreement prohibiting assignments. Willow occupied the apartment for four months but then decided to drop out of law school. She proceeded to assign the lease to her girlfriend, Cassandra Cordova, who moved into the apartment cm December 1, 1996. Waldo refused to accept Cassandra's December rental payment and told her he wouldn't allow her to occupy the apartment for only $300 a month. Waldo told Cassandra that he only charged Willow a $300 rental fee be¬cause she was a relative. Waldo advised Cassandra if she wanted to occupy the apartment, she would have to pay $1,000 per month or the fair market value. 196. Cassandra indicates that she will not pay more than a $300 rental which Waldo refuses to accept. Waldo brings an ejectment action against Cassandra who counterclaims asserting that the assignment from Willow was valid. Judgment for (A) Cassandra, because the assignment was valid with¬out the landlord's consent (B) Cassandra, because the lease did not contain a non-assignability clause (C) Waldo, because he did not consent to the assignment (D) Waldo, because he rented the apartment to Willow on special terms, she was not empowered to make an assignment

196. (A) and (B) Sometimes on the MBE credit is given for two correct answers. Here, both (A) and (B) are equally correct because absent a non-assignability provision in the lease, the assignment is valid without the landlord's consent. To be sure, a lease is a contract and all contracts are assignable. After an assignment without a novation both the tenant and the assignee are liable to the landlord for rent thereafter becoming due. After an assignment there is privity of estate between the landlord and the assignee. However, the assignment has no effect on the privity of contract between the landlord and the tenant who remains liable on the basis of privity of contract.

92. The provision in the 1950 agreement between Oscar and Groves Mining Co. granting "the use of the pathway" to Groves may best be described as a (an) (A) license (B) easement in gross (C) easement appurtenant (D) prescriptive easement

92. (C) An easement is deemed appurtenant when the right of special use benefits the holder of the easement in his physical use or enjoyment of another tract of land. For an easement appur¬tenant to exist, there must be two tracts of land. One is called the dominant tenement (which has the benefit of the easement). The second tract is the servient tenement (which is subject to the easement right). In the present case Oscar granted Groves an express easement to use the roadway as an access to Groves' adjacent property. Choice (A) is incorrect since a license is a revocable privilege to enter upon the lands of the licensor. Choice (B) is wrong because an easement in gross is created where the holder of the easement interest acquires his right of special use in the servient tenement independently of his ownership or possession of another tract of land. Alternate (D) is erroneous since a prescriptive easement is not in writing, but arises through the long continued use of the property (as analogous to adverse possession).

Wanda and Hal were married and had four children, Alan, Betty, Carla and David. Wanda owned Blackacre as her own separate property. Wanda executed a valid will leaving Black-acre "to Hal until such time as Hal dies or remarries, then in equal shares to my children. If any of my children shall die prior to Hal's death or remarriage, then to their surviving issue." Subsequently, Wanda and Hal divorced. In the divorce settlement, Hal waived all rights to any interest in Blackacre in exchange for valuable consideration. Prior to Wanda's death, Carla and David died in an automobile accident. The following year, Betty died after a long illness. Betty was survived by her two children, Eddie and Francis. 197. At Wanda's death, the rights of Wanda's children in Blackacre are best described as (A) nonexistent, until Hal dies or remarries (B) nonexistent, since the gift to the issue of Wanda's children violates the Rule against Perpetuities (C) a fee simple absolute belonging to Alan (D) one-half to Alan, one-fourth to Francis, and one-fourth to Eddie

197. (D) Under the terms of Wanda's will, her property was to pass to her husband first, then to her children and the surviving issue of the predeceased children upon Hal's death or remarriage. However, Hal waived all his rights to Blackacre in the divorce settlement. Thus, the only remaining interests belong to the children and the surviving issue of the predeceased children. The gift to the surviving issue of Wanda's children is not violative of the Rule Against Perpetuities because their (surviving issue) interests, if any, will vest or fail within all lives in being (Wanda's, Hal's, and any of Wanda's children) plus twenty-one years. The only surviving issue Wanda's children can have are those who are alive at the death of Wanda. When Carla and David died (without leaving surviving issue) their interests in Blackacre terminated and Alan and Betty would each have a one-half (expectancy) interest in Blackacre. When Betty died, her one-half interest passed to her issue. Thus, when Wanda died, Alan would take a one-half interest and Betty's issue would share the other half.

Question 198 is based on the following fact situation. Cramden owned a two-story colonial style home that is located in the town of Steamboat Springs. Cramden purchased the house in 1965 from Doyle for $25,000. In 1980, Cramden borrowed $80,000 from the Steamboat Savings and Loan Company. To secure the loan, Cramden gave the bank a mortgage on his property. The following year, Cramden sold the house to Bay. The deed of conveyance which Cramden delivered to Bay provided that the buyer purchased the property "subject to the mortgage" held by Steamboat Savings and Loan Company. During the next few years, Bay failed to properly maintain the home. His gross negligence has caused the property to diminish in value far below the mortgage amount. 198. Now in 1997 (the year the mortgage comes due) Steam¬boat Savings and Loan seeks your legal advice on how best to proceed. You should advise the bank that it is entitled to a (A) foreclosure action only (B) deficiency judgment against Cramden only (C) deficiency judgment against Bay only (D) deficiency judgment against Cramden and Bay

198. (B) A mortgage is an interest in land created by a written instrument providing security for the performance of a duty or the payment of a debt. Upon default by the mortgagor, the mortgagee has both an "in personam" claim against the mortgagor for the debt and an "in rem" action against the property or security. In a foreclosure action the security (i.e., the mortgaged proper¬ty), or proceeds from the sale thereof, is applied to the satisfaction of the debt. Steamboat Savings and Loan, as mortgagee, would be able to bring a foreclosure action in 1997 if the mortgage debt was not paid in full by Cramden. No action by Steamboat against Bay would lie because Bay did not assume Cramden's liability. The Cramden-Bay deed provided that Bay, the buyer, purchased the property "subject to the mortgage/'

Seller owns a single-family tract home that was built over 40 years ago. Seller advertised the home for sale and entered into a land sale contract with Buyer. Buyer hired a contractor who inspected the house and found no material defects. After the close of escrow and Buyer had taken possession, Buyer heard some odd sounds coming from the attic. An exterminator hired by Buyer determined that there were several small mice who had nested underneath the attic insulation and that the mice had chewed up several electrical wires that needed to be replaced immediately due to a potential fire hazard. Buyer paid to have the mice removed and the wiring replaced and then brought suit against Seller to recover for the costs of repair. At trial, Seller acknowledged that Seller was aware that there were mice living in the attic but that Buyer had not asked about any problems in the attic and that Seller did not try to cover up the problem in any way. 199. If the court rules in favor of Buyer it will be because (A) buyer's contractor failed to reveal the existence of mice and damage to the wiring (B) the damage to the wiring presents a serious safety hazard (C) seller never revealed the existence of a hidden defect (D) seller breached the warranty of quiet enjoyment

199. (C) is the correct answer by process of elimination. (A) is incorrect because the fact that Buyer's contractor failed to see the mice and damage to electrical wires is understandable (since the problem was hidden under the insulation) but does not supply a legal ground for recovery from the Seller. (B) is incorrect because a defect that presents a safety hazard does not necessarily compel a finding that the Seller must pay to remedy that hazard once title has passed to the Buyer. (D) is incorrect because the warranty of quiet enjoyment applies only to a conveyance made pursuant to a warranty deed or other deed expressly containing the warranty of quiet enjoyment. There are no facts suggesting that a warranty of quiet enjoyment was included in this transaction. Thus, (C) becomes the best choice. This is a tough question because most students understand that, in general, a seller of real property is only liable for affirmative mis¬representations that a buyer relies on to her detriment. Liability for omissions is rare, but possible in many jurisdictions that impose liability for failing to disclose material defects that a seller knows about and also knows the buyer is not likely to discover.

32. Which of the following is the most accurate statement regarding a tenant's covenant to keep the premises in repair? (A) A tenant is liable for normal wear and tear under the covenant to repair. (B) A tenant is liable under such a covenant for all defects, including the acts of God. (C) A tenant's covenant to repair is void as against public policy, since the landlord is under an affirmative duty to make repairs on the demised premises. (D) The tenant's duty to keep the premises in good repair relieves the landlord of his obligation to disclose the existence of any latent defects.

20. (B) The most accurate statement regarding a covenant (in the lease) which requires the tenant to keep the premises in repair is that he is liable under such a covenant for all defects (except normal wear and tear) regardless of their cause. As a result, a tenant is liable for any defects caused by himself, third persons, and even acts of God.

Lord was the owner of a large, high-rise apartment building in Detroit. On June 1, 1981, Mr. and Mrs. Tenner took possession of a 3 bedroom apartment in Lord's building under a 3 year lease to them from Lord at a rental of $825 per month. Their lease (as all other leases given by Lord) contained the following provisions: "Tenants hereby agree that...the premises are not to be occupied by any two or more persons not relat¬ed to one another by blood or marriage..." "The term of this lease shall be three years from the date hereof as long as all the agreements herein shall be faithfully performed." Mr. and Mrs. Tenner lived in the apartment for two years. On June 10, 1983, however, a fire destroyed the apartment building. As a result, all the apartments in the building were rendered uninhabitable. After Mr. and Mrs. Tenner were dispossessed from their apartment, Lord brought suit against them to recover the rent due for the balance of the lease. Mr. and Mrs. Tenner claim that they are no longer liable for rent or any other obliga¬tions under the lease. The Lord-Tenner leasehold contract con-tained no provision regarding liability for fire. 20. If the decision is in favor of Mr. and Mrs. Tenner, it will be because (A) there was nothing in the lease regarding liability for fire (B) Mr. and Mrs. Tenner did not own an interest in the property (C) Lord should not be unjustly enriched (D) Lord did not contract to convey the property to Mr. and Mrs. Tenner

20. (D) At common law, a tenant remains liable to pay rent even though because of fire, floods, storms or other action of the elements or otherwise, the property is rendered totally uninhabitable unless the lease otherwise provides. So, therefore, if the decision is in favor of Mr. and Mrs. Tenner, it will most likely be because the doctrine of equitable conversion is inapplicable. Note that the equitable conversion doctrine applies only when there is an enforceable obligation to sell land. Caveat: Choice (C) is wrong because since this is an action at law (to recover money damages), the court will not apply equitable considerations (e.g., unjust enrichment) in resolving the dispute. Choice (B) is also wrong because a leasehold is considered an interest in land.

Question 200 is based on the following fact situation. Johnny and Joanna Appleseed married in 1955. Two years later, they acquired ownership to Marshacre "as joint tenants with right of survivorship." Marshacre was comprised mostly of swampland and was located five miles outside of Secaucus. When the Appleseeds purchased the property, it had little monetary value and was not very desirable. Recently, however, the state conducted a large scale land-fill operation of the swamp¬land. This made Marshacre suitable for residential lots and resulted in the commercial development of the area. As a consequence, the market value of Marshacre increased substantially. In 1985, the Appleseeds wished to convey an interest in Marshacre to their daughter, Joy, and her husband, Joshua. Thereupon, Johnny and Joanna executed a deed wherein they conveyed an undivided 15 percent interest in Marshacre to Joy. In 1988, the Appleseeds then conveyed a 15 percent interest in Marshacre to Joshua. The common-law joint tenancy is unmodified by statute, and the jurisdiction in which Marsh¬acre is located does not require tenancy by the entirety or community property. 200. Title to Marshacre is in Johnny and Joanna Appleseed as (A) joint tenants as to 70 percent, Joy as a tenant in common as to 15 percent, and Joshua as a tenant in common as to 15 percent (B) joint tenants as to 70 percent and Joy and Joshua as joint tenants as to 30 percent (C) tenants in common as to 70 percent, Joy as a tenant in common as to 15 percent, and Joshua as a tenant in common as to 15 percent (D) tenants in common as to 70 percent and Joy and Joshua as joint tenants as to 30 percent

200. (A) As a general rule, any conveyance inter vivos by a joint tenant constitutes a complete severance of her interest in the jointly owned property and destroys the joint tenancy as to the extent of the interest conveyed. Thus, by conveying 30% of Marshacre to Joy and Joshua, the joint tenancy was destroyed with respect to that portion of the property. However, the joint tenancy interest still applies to the 70% tract of Marshacre retained by Johnny and Joanna. Choice (A) is correct because as to the undivided 70% interest Johnny and Joanna remain joint tenants while Joy and Joshua are tenants in common to their individual 15% interests.

Questions 21-22 are based on the following fact situation. In 1995, John Jones executed a deed by which he conveyed Blackacre for a consideration of one dollar, receipt of which was acknowledged, "to Burkhart for life, then to Carr for life but if Carr moves to another state, to Drew for the life of Carr, then to the heirs of Carr if Carr does not move to another state and to the heirs of Drew if Carr does move to another state." This deed was promptly recorded. 21. During Burkhart's lifetime, Carr's interest may best be described as a (an) (A) contingent remainder (B) shifting executory interest (C) vested remainder subject to complete divestiture (D) vested remainder subject to partial divestiture

21. (C) Choice (C) is correct. Carr's interest in Blackacre would be a vested interest subject to com¬plete divestiture. At the expiration of Burkhart's life estate, Carr would immediately be entitled to take possession of Blackacre. However, Carr's vested remainder (i.e., life estate in futuro in Blackacre) would be subject to complete divestiture upon the contingency of his redomiciling. Choice (A) is incorrect since a contingent remainder is a remainder limited so as to depend on an event or condition which may not happen or be performed until after the termination of the preceding estate. Conversely, we are certain, here, that Burkhart's life estate will eventually ter¬minate. Thus, although Carr may predecease Burkhart thereby terminating his actual enjoyment, his right to such enjoyment (i.e., vested remainder in Blackacre) is not uncertain. Choice (B) is incorrect since a "shifting" executory interest is a contingent remainder whereby one party's expectant interest is transferred or "shifted" to another upon the happening or non-happening of a particular contingency.

In 1970 Rufus Reefer owned Grassacre, a thirty acre tract located just inside Weedville in Humboldt County. Grassacre included the family home, The Rookery, a decaying antebellum mansion complete with tennis courts, stables and a smaller second house which once was occupied by tenants who farmed Grassacre. The second house, however, had long been vacant as a result of the economic decay of Humboldt County. Prosperity burst upon Weedville in 1971, and Reefer began selling acre lots in Grassacre. By 1978, Reefer had sold twenty five acres, retaining five acres which included The Rookery, tennis courts, stables and the former tenants' house. On May, 19, 1979, Reefer entered into a valid written contract with Doper. According to the terms of their agreement, Reefer agreed to sell and convey his remaining interest in Grassacre for a consideration of $500,000. The land sale contract provided a closing date of November 19, 1979 and stipulated that "time was of the essence." On July 2, 1979, a fire destroyed The Rookery. Reefer had The Rookery insured for $450,000 against fire loss and collected that amount from the insurance company. At the closing on November 19th, Doper tendered a cashier's check for $50,000 and demanded a deed conveying a fee simple interest in the property. Conversely, Reefer tendered a deed of conveyance and demanded the full purchase price of $500,000. Doper refused Reefer's demand. 23. In an appropriate action for specific performance against Doper, Reefer demanded $500,000. If Reefer prevails, which of the following is the best rationale for the outcome? (A) The fact that The Rookery was insured for $450,000 is irrelevant. (B) Reefer and Doper each had an insurable interest in the property. (C) The doctrine of equitable conversion has been abolished. (D) The doctrine of equitable conversion requires such a result.

23. (D) This is exactly the same situation that Smith and Boyer discuss in their hornbook on pg. 255. An executory contract for the sale of land requiring the seller to execute a deed con¬veying legal title upon payment of the full purchase price works an equitable conversion so as to make the purchaser the equitable owner of the land and the seller the equitable owner of the purchase price. The result is that the purchaser, the equitable owner of the land, takes the benefit of all subsequent increases in value and, at the same time, becomes subject to all losses not occasioned by the fault of the seller. Thus, the purchaser, to protect himself, either must procure his own insurance, or by appropriate provision in the contract, cast the risk upon the seller. He is not, however, entitled to recover insurance payments payable to the vendor. Note that choice (D) is a better answer than (A) because the equitable conversion doctrine which places the risk on the buyer requires such a result.

Question 39 is based on the following fact situation. Southhampton is a state with a long stretch of scenic, eco¬logically unique, and relatively unspoilt sea coast. Most of the coastline is privately owned by persons who have permanent or vacation residences on their coastal lands. In 1954 Little pur¬chased a two acre tract of littoral (i.e. coastal) land in Southhampton. When Little bought the property, there was a narrow concrete walkway (about 20 feet in length and 5 feet in width) adjoining the land. The walkway, which was designed to provide pedestrian access to the beach, was owned by the Southhampton Coastal Commission. When Little moved into his beach house in 1954, he noticed that the walkway was rarely used. As a result, Little decided to enclose the walkway and con¬struct a paddle ball court over the entire area. He did so without notifying the state Coastal Commission. He has continued to use this area in excess of the period required for adverse possession. 39. In an appropriate action brought by Little to establish title to the walkway area, which of the following must he establish if he is to prevail? I. Real property interests can be abandoned by a govern- mental entity without an official vote. II. Lack of use of the walkway by the state created an irrevocable license in him. III. Real property interests can be lost by a state by adverse possession. IV. His use of the walkway area is proof of his intent to assert dominion over that section. (A) I and II only (B) I and III only (C) II and IV only (D) III and IV only

39. (D) In answering "tier" questions of this type, you should read each statement (i.e., I, II, III and IV) and determine whether it is true or false. Be advised that statement II is false because this question deals with adverse possession not licenses. As a consequence, choices (A) and (C) can be eliminated. In choosing between alternatives (B) and (D), the latter is preferred because it establishes the necessary state of mind required for adverse possession. Possession of real prop¬erty requires acts of dominion and control with an intent to possess and exclude others.

Question 24 is based on the following fact situation. On November 1, 1984, Beeson contracted to purchase from Sloan for $250,000 certain property on which was erected a hotel. The contract required Beeson to pay Sloan a deposit of $12,500 with the balance of the purchase price payable at clos¬ing on December 1, 1984. On November 24, 1984, a fire caused the hotel to burn down. As a consequence, Beeson refused to honor the contract. Sloan now brings an action for specific performance arguing that the doctrine of equitable conversion places the loss on the buyer. Beeson argues that to enforce the contract would be harsh and oppressive to him. 24. If judgment is for Beeson, it will be because (A) Sloan assumed the risk (B) Sloan would be unjustly enriched (C) legal title remained in Sloan (D) equity will relieve Beeson of a bad bargain

24. (B) Under the doctrine of equitable conversion the risk of loss from casualty and other fortu¬itous events is normally placed on the purchaser in the absence of controlling provisions in the contract. Equity thus considers the vendee as the owner of the land and the vendor as the owner of the purchase money. Smith and Boyer point out, however, that this rule is limited in its application to cases where the intention of the parties will not produce an inequitable result. For example, assume that A contracts to sell to B a certain piece of land which was to be used for the purpose of erecting a hotel. However, between the time the contract of sale was made and the time for delivery of the deed, the city council rezones the lot so that it could only be used for residential purposes. A now brings suit for specific performance. In this situation, Smith and Boyer note that the granting of specific performance would be unduly harsh and oppressive to B. Since the intent of the parties was defeated by the supervening event, specific performance should be denied. However, the denial of specific performance does not end the matter, but, instead, the vendor may proceed against the purchaser in a suit at law. Real Property, pg. 255. By analogy, if the court rules in favor of Beeson, it will be to avoid unjust enrichment.

The state of Runnymede has the following recording statute in effect: "No conveyance is good against a subsequent pur¬chaser for a valuable consideration and without notice, unless the same be recorded prior to subse¬quent purchase." Lau is the owner in fee simple of Blackacre, a 20-acre tract of unimproved land, situated in Madison, Runnymede. On May 1, Lau sold Blackacre to Eto for the purchase price of $40,000 under a quitclaim deed. Eto did not record the deed. On May 5, Lau died leaving a valid will by which he devised Blackacre to his son, Landon. After the administration of Lau's estate, Landon, in consideration of the sum of $75,000, conveyed Blackacre to Fong by warranty deed. This transaction took place on August 1. When Fong acquired title to Blackacre, he had no actual knowl¬edge of Eto's deed (which was still unrecorded). On August 10, Eto recorded his deed to Blackacre. Fong did not record the deed he received from Landon until August 15. 25. In an appropriate action to quiet title to Blackacre, in which all interested parties have been joined, title will be found to be in (A) Eto, because his deed preceded Fong's deed (B) Eto, because his deed was recorded prior to Fong's deed (C) Fong, because he is protected by the recording statute (D) Fong, because he took title by warranty deed and Eto took title by quitclaim deed

25. (C) The "key" to this question is carefully interpreting the wording of the recording statute. According to the statutory language, the Runnymede recording act protects subsequent bona fide purchasers for value and without notice. As such, this is an example of a pure "notice" type recording statute which generally provide: an unrecorded conveyance or other instrument is invalid as against a subsequent bona fide purchaser for value and without notice. Under this type of recording statute, the subsequent bona fide purchaser prevails over the prior interest whether the subsequent purchaser records or not. As a consequence, Fong will prevail over Eto because Fong was a subsequent bona fide purchaser without notice of Eto's deed.

Question 26 is based on the following fact situation. A deed executed by A in 1977 conveyed Blackacre for a con¬sideration of one dollar, receipt of which was acknowledged, "to B for life, then to the heirs of B." A life interest in Blackacre for the life of B is worth $20,000 on the date of the conveyance. The total worth of Blackacre is $50,000. B accepted but didn't record the deed. The recording statute in this jurisdiction pro¬vided "unless recorded all written instruments affecting title to land are void as to subsequent purchasers who paid value and without notice." In 1981 A purported to convey Blackacre in fee simple absolute to his two sons, C and D, by a warranty deed as a gift. C and D recorded the deed. Shortly thereafter, B ascertained that C and D were about to take possession of Blackacre. As a con¬sequence, B promptly recorded his deed. 26. In a dispute between B and A's children as to the ownership of Blackacre, if B prevails it will be because (A) B paid valuable consideration for his deed (B) B recorded his deed before A's children sought to oust him from the land (C) C and D are not protected by the recording statute (D) A's knowledge is imputed to his children

26. (C) In order to be a bona fide purchaser protected under the recording act, one must (a) be subsequent, (b) pay value, (c) be without notice (the value must have actually been paid before notice), and (d) be of good faith. Students must be aware that recording statutes do not protect a subsequent claimant who has not paid more than a nominal consideration because he is not a purchaser. Therefore, in our case C and D (A's children) are not protected by the recording statute because they are not purchasers (as A purportedly conveyed the property to them as a gift).

Fletcher, a noted conservationist, died four years ago leaving a will by which he devised Greenacre, a 100-acre tract, to his daughter, Fawn. At the time of Fletcher's death, he believed that he owned all of Greenacre. However, Fletcher actually owned ninety-five acres of Greenacre. The other five acres were owned by Oswald. After taking possession of Greenacre, Fawn execut¬ed and delivered a warranty deed to Scott purporting to convey all 100-acres of Greenacre. The agreed purchase price was $100,000. According to the terms of the sale, Scott paid Fawn a down payment of $25,000 with the unpaid portion of the pur¬chase price (i.e. $75,000) secured by a mortgage. The mortgage instrument described the property interest as covering all 100-acres of Greenacre. After the Fawn-Scott transaction was completed, Oswald came forward and informed the parties that she was the true record owner of five acres of Greenacre. Upon Oswald's threat to sue to quiet title, Fawn negotiated a deal whereby she paid Oswald $5,000 for the five-acre tract. As part of their agreement, Oswald executed and delivered a quitclaim deed quitclaiming to Scott all of her interest in the five-acre parcel. Thereafter, Scott defaulted on the mortgage debt and Fawn properly consummated foreclosure proceedings, becoming the purchaser of Greenacre at her own foreclosure sale. The descrip¬tion of the deed in the foreclosure sale referred to Greenacre as consisting of all 100-acres. After the foreclosure sale was final¬ized, Scott brought suit claiming title in fee simple to the five-acre tract, formerly owned by Oswald. 27. In an appropriate action to determine the title to the said five-acre tract, Scott will (A) lose, because Fawn did not have good and marketable title at the time she purported to convey Greenacre to Scott (B) lose, because the doctrine of after-acquired title controls (C) win, because the deed from Oswald to Scott was a quitclaim deed (D) win, because the quitclaim deed from Oswald to Scott was subsequent to the deed from Fawn to Scott and to the mortgage

27. (B) The doctrine of estoppel by deed (sometimes referred to as the "after-acquired title" doc¬trine) applies when a person executes a deed purporting to convey an estate in land which he does not have or which is larger than he has, and such person at a later date acquires such estate in such land, then the subsequently acquired estate will, by estoppel, pass to the grantee. In this example, once Fawn purchased the five-acre tract from Oswald, title inured to the benefit of Scott as evidenced by Oswald's quitclaim deed to Scott. Most importantly, Fawn held a mort¬gage on Greenacre. The mortgage instrument described the property interest as covering all 100-acres of Greenacre. Thus, when Scott defaulted and Fawn foreclosed, she purchased back the entire tract of Greenacre (including the 5-acres previously owned by Oswald). Note that choice (A) is wrong because even though Fawn did not have good and marketable title to Greenacre initially, this title defect was subsequently cured by operation of the Fawn-Oswald transaction. Choice (C) is incorrect because at the moment Fawn acquired title to the 5-acres from Oswald, title automatically inured to the benefit of Scott by virtue of the estoppel by deed doctrine. It is really irrelevant whether Oswald quitclaimed the property to Scott. This is tricky because in most states estoppel by deed will not be applied where the conveyance is by a quit¬claim deed. In this question, however, students must remember that the Fawn-Scott conveyance was by warranty deed. That's why "after-acquired title" applies. In other words, even if Oswald didn't quitclaim the 5-acre parcel to Scott, title would nonetheless inure to Scott's benefit.

4. Regardless of which legal device is used in the preceding question, which of Duffy's objectives is LEAST likely to be upheld? (A) Restriction to use of the island for residential purposes. (B) Restriction prohibiting the hunting of animals on the island. (C) Restriction requiring the mansion to be painted only the color of pink. (D) Restriction on the sale, mortgage or other encumbrance on the land.

4. (D) Reasonable restrictions on the use of the land—such as residential use only, prohibition of hunting on the premises, or painting the premises only a certain color are generally upheld either as covenants at law or as equitable servitudes. Choices (A), (B) and (C) are reasonable types of restrictions. However, regarding restraints on alienation, the general rule is that any type of direct restraint upon alienation is null and void insofar as an estate in fee is concerned, based on strong public policy favoring free alienation. Burby, Real Property, p. 427. Therefore, choice (D) is the least likely restriction to be upheld since restricting the sale, mortgage, or other encumbrance of the land would amount to a direct restraint on alienation.

Wilson, being fee simple owner of Blackacre, devised it to his daughter, Jennifer, and her husband, Brad, as tenants by the entirety. Jennifer and Brad took immediate possession of Blackacre and lived there with their children, Beth and Jeffrey. Thereafter, Brad died in an automobile accident. In 1952, two years after her father's death, Beth moved to another state. Jeffrey lived with his mother on Blackacre until her death intestate in 1960. He continued in exclusive possession of Blackacre until his death in 1981. In his will, Jeffrey devised Blackacre to the Boy Scouts of America. Jeffrey was unaware that Beth was still alive and that title to Blackacre had descend¬ed to the two of them as their mother's sole surviving heirs. Since his mother's death in 1960, Jeffrey has held himself out as the owner of Blackacre, maintaining it and paying all of the taxes on the property. Beth has not communicated with either her moth¬er or Jeffrey since her redomiciling in 1952. The jurisdiction in which Blackacre is located has a 20 year limitation period for the acquisition of property by adverse possession. 28. What interest, if any, does Beth have in the property? (A) None, because of her own laches. (B) None, because Jeffrey acquired title to Blackacre by adverse possession. (C) An undivided one-half interest because Jeffrey's pos¬session was not adverse to Beth's title. (D) An undivided one-half interest because the 20 year limitation period did not run against her since she was unaware of Jeffrey's exclusive possession.

28. (C) Here, Jeffrey and Beth acquired title to Blackacre as tenants in common (by descent). In a tenancy in common each tenant owns an undivided fractional part of the property, none owns the whole as in joint tenancy. Note that choice (B) is wrong because Jeffrey's possession of Blackacre was not hostile and adverse since he in fact was the owner of the property (as a ten¬ant in common).

Questions 29-30 are based on the following fact situation. Landlord owns a three-story building which is located in the township of Scotch Plains. Landlord leased the building and the surrounding property to tenant for a term of six years commenc¬ing on December 1, 1975 and terminating on November 30,1981. Their leasehold agreement contained the following provisions: "Tenant covenants to pay the rent of $500 per month on the first day of each month and to keep the building situated upon leased premises in as good repair as it was at the time of said lease until the expiration thereof." On April 15, 1976 Neighbor, owner of a adjoining parcel of land, was burning leaves and underbrush on his land. There was a high wind blowing in the direction of the land which Landlord leased to Tenant and the wind carried the burning leaves onto Tenant's property. The building caught fire and caused $150,000 in fire damages. Because of the fire, Tenant has not occupied the leased premises since the date of the fire because the building was rendered uninhabitable. NOTE: Scotch Plains is a common law jurisdiction. In addi¬tion, the controlling Scotch Plains Statute of Limitations for initi¬ating an action for damages due to negligence is five years from the time the cause of damage occurs. 29. On May 1, 1976 Landlord brings suit against Tenant assert¬ing breach of contract. Judgment for (A) Landlord, since a Tenant remains liable to pay rent even though as a result of the fire, the property was rendered uninhabitable (B) Landlord, since the covenant to pay rent runs with the land (C) Tenant, as it would be inequitable to enforce the rental provision in the lease because the premises was ren¬dered uninhabitable (D) Tenant, since an increasing number of states have enacted statutes relieving a tenant of his obligation to pay rent where the premises are rendered uninhabit¬able by fire or other acts of God

29. (A) Tenant is still liable to Landlord for the $500 monthly rental despite the fact that he can no longer reside in the building. At common law, the general rule is that a tenant remains liable to pay rent even though because of fire, floods, or other unforeseen action, the property is ren¬dered totally uninhabitable. Please note that it is possible for the parties to provide in a lease for certain excuses for nonpayment of rental. But in our hypothetical, no such provision was made in the lease.

Questions 3-4 are based on the following fact situation. Duffy, a wealthy and devoted naturalist, owned a large and uninhabited island just a short distance off shore in Balboa Lake in the state of Dakota. Duffy permitted no pesticides or hunting on the island, and over the course of his life had bought several quite rare species of animals which were native to Dakota and turned them loose on the island. Among these was a magnificent albino ocelot. Duffy lived in a large Tudor-style mansion that his grandfather, Dudley Duffy, built in 1902. Since the time the mansion was built, it has passed from father to son. In 1993, Duffy has informed his attorney that he wishes to transfer ownership of the island to his daughter, Dawn. During the consultations with his attorney, Duffy indicated that he wanted the following restric¬tions incorporated in the instrument of conveyance: (1) all use of the island's land must be restricted to residential, non-commercial purposes; (2) no hunting of animal life on the island shall be permitted and reasonable efforts should be made to pre¬serve the albino ocelot species; (3) the Tudor-style mansion should be repaired as needed but must never be torn down or demolished; and (4) the mansion shall never be any color except pink, the color it has been since it was built at the turn of the century. Furthermore, Duffy told his attorney that he wanted to prevent Dawn from selling, mortgaging or otherwise encumbering the land within Dawn's lifetime. Lastly, Duffy remarked to his attor¬ney, "I want to keep things as simple and plain as possible; but I flat out don't want you setting up any type of trust arrangement." 3. Which of the following legal devices is most likely to be appropriate for carrying out Duffy's wishes? (A) Equitable servitude (B) Defeasible fee (C) Covenant (D) Tenancy at will

3. (B) A "fee" describes a freehold estate of inheritance which has potentially infinite duration, or which can be terminable upon the happening of an event (i.e., "defeasance"). As a posses¬sory real property interest a defeasible fee can be distinguished from a covenant, choice (C), or an equitable servitude, choice (A). The latter two interests are non-possessory interests which act as private land use restrictions. Since Duffy wants to transfer ownership of the island to Dawn, he must convey to her a possessory interest. Choice (B), a defeasible fee, is the correct answer, since it is the only possessory interest given. Choice (D) is incorrect because a tenancy at will is an estate in land terminable at the will of either the landlord or the tenant. No such relationship is intended under these facts.

30. In an appropriate action to construe Tenant's obligation under the covenant to keep the premises in repair, which of the following is the most accurate statement? (A) Tenant is liable for normal wear and tear under the covenant to repair. (B) Tenant is liable under such a covenant for all defects, including the acts of God. (C) Tenant's covenant to repair is void as against public policy, since the landlord is under an affirmative duty to make repairs on the demised premises. (D) Tenant's duty to keep the premises in good repair relieves the landlord of his obligation to disclose the existence of any latent defects.

30. (B) The general rule provides that a tenant who covenants to keep the leasehold premises in good repair is liable under such a covenant for all defects (except normal wear and tear) regard¬less of their cause. As a result, a tenant remains liable for any defects caused by himself, third persons and even acts of God. Choices (A), (C) and (D) are all incorrect statements of law.

Harry is the owner in fee simple of a tract of land on which is situated a large office building. Harry leases the land and build¬ing thereon to Tony for a term of seven years, commencing on August 1, 1974, and terminating at midnight on July 31, 1981. The lease contains the following provisions: "(1) Tony covenants not to assign the leased premises without the consent of Harry; (2) Tony covenants to pay the rent of $750 per month on the first day of each month; (3) Tony covenants to keep the leased premises in repair; (4) Tony covenants to keep the building on the leased premises insured against fire in the amount of $100,000; (5) Harry covenants to utilize any fire insurance proceeds for repair or replacement only." After three years of the lease had expired, Tony, without the consent of Harry, assigned the entire balance of the lease period to Rick, who took immediate possession of the leased property. Harry accepted rental payments from Rick. Then in 1978, Rick assigned his leasehold interest to Simon. Simon went into pos¬session, but failed to pay any rent for two months. Soon after¬wards, a fire spread from an adjoining building to the leased property, completely destroying the building thereon. It was established that the owner of the premises on which the fire started had failed for several years to comply with the municipal Fire Code measures applicable to his building. While Tony was in possession of the leased property, he carried a fire insurance policy on the premises in the amount of $100,000. However, Tony allowed the policy to lapse after his assignment to Rick. Rick did carry insurance on the leased building, but only in the amount of $50,000. When Simon took possession of the building, he did not obtain any fire insurance. 31. Harry learned that the building was not insured at the time of the fire. In an action by Harry against Tony to recover for the fire loss, Harry will most probably (A) recover, since in accordance with the Rule in Spencer's Case, the covenant to maintain fire insurance would "touch and concern" the land (B) recover, since Tony's obligation to maintain fire insur¬ance did not terminate after his assignment to Rick (C) not recover, since the covenant to provide fire insur¬ance did not run with the land (D) not recover, since Simon, as assignee, would be liable for the fire loss

31. (B) A covenant in a lease to pay (fire) insurance is held to "run with the land." In this regard, note that a covenant to pay insurance is capable of running if and only if the landlord is bound to use the proceeds for repair or replacement. It is important to understand that Tony's assignment of his leasehold interest to Rick did not relieve or extinguish Tony's contractual obligations under his lease with Harry. In short, an assignment does not release the tenant from his contract obligations to the landlord under the terms of this original leasehold agreement. To be sure, a lease is a contract as well as a conveyance.

34. Assume for the purposes of this question only that Harry, after learning of Rick's assignment to Simon, brings suit against Rick to have the assignment declared void. Harry will most likely (A) succeed, since the original leasehold agreement pro¬hibited assignments (B) succeed, since the covenant prohibiting assignments did not run with the land (C) not succeed, since in accordance with the Rule in Dumpor's Case, where the landlord consents to one transfer, he waives his right to avoid future transfers (D) not succeed, since the privity of estate between Rick and Harry terminated when he assigned the leasehold to Simon

34. (C) In accordance with the Rule in Dumpor's Case, if a landlord grants consent to one trans¬fer (e.g., Tony's assignment to Rick), he waives his right to avoid future transfers, assigns or sub¬leases in violation of a prohibition in the lease against such transfers. By the same token, if the lessee/assignee pays rent to the landlord and such rent is accepted by him, in spite of the fact that the transfer was in violation of the lease, the landlord will be deemed to have waived his right to avoid the transfer.

Donaldson, an owner in fee simple, leased a three-story house to McQuirk for a term of ten years. By the terms of the lease McQuirk expressly covenanted to pay a monthly rental of $300 and to pay the taxes on the premises during the term of the lease. The lease also stipulated that McQuirk, as lessee, may not assign or sub-lease the said premises. McQuirk and his family lived in the house for two years. Then McQuirk, still owing Donaldson three months back rent, assigned his leasehold interest in the property to Moore, who was unaware of the prohibition against assignments. This writ¬ten assignment expressly provided that Moore would pay Donaldson the monthly rental of $300 but was silent concern¬ing the payment of taxes. Donaldson never objected to this assignment. Six months after the assignment, Donaldson con¬veyed his property interest in the premises to Davis. After residing in the house for a year, Moore sub-leased the third floor to Burger for a term of two years; the agreement pro¬vided that Moore would receive a monthly rental payment of $100. 40. Moore failed to pay the taxes on the property. Davis, after paying the taxes, brought suit against Moore for the amount paid. Judgment would be for: (A) Davis (B) Moore, since h21. (C) Choice (C) is correct. Carr's interest in Blackacre would be a vested interest subject to com¬plete divestiture. At the expiration of Burkhart's life estate, Carr would immediately be entitled to take possession of Blackacre. However, Carr's vested remainder (i.e., life estate in futuro in Blackacre) would be subject to complete divestiture upon the contingency of his redomiciling. Choice (A) is incorrect since a contingent remainder is a remainder limited so as to depend on an event or condition which may not happen or be performed until after the termination of the preceding estate. Conversely, we are certain, here, that Burkhart's life estate will eventually ter¬minate. Thus, although Carr may predecease Burkhart thereby terminating his actual enjoyment, his right to such enjoyment (i.e., vested remainder in Blackacre) is not uncertain. Choice (B) is incorrect since a "shifting" executory interest is a contingent remainder whereby one party's expectant interest is transferred or "shifted" to another upon the happening or non-happening of a particular contingency.did not contract with McQuirk to pay the taxes on the property (C) Moore, since the covenant to pay taxes will not "run with the land" unless the intent is clearly expressed in the original lease (D) Moore, since the covenant to pay taxes is merely col¬lateral and does not "run with the land"

40. (A) By virtue of the assignment, the assignee (i.e., Moore) falls into privity of estate with the landlord and he is liable on all covenants in the original lease which "run with the land". Since the covenant to pay taxes, under the original leasehold agreement between landlord Donaldson and tenant-McGuirk, "runs with the land", Moore, as assignee, would be held liable. In light of the fact that Moore, as assignee, is liable on the covenant to pay taxes, all other choices are incorrect.

42. After Burger had made regular rental payments to Moore for the first six months of the sub-lease, he defaulted. The fol¬lowing month, Moore sent Davis $200 as payment for rent and notified him that he should collect the remaining $100 from Burger, who refused to vacate the premises. In an action by Davis against Burger to recover the additional $100 for rent due, he will most probably (A) recover, since the landlord and sublessee are in privity of estate (B) recover, even though he never objected to the sublease (C) not recover, since McQuirk, as assignor, would only be held liable (D) not recover, since Moore remains liable

42. (D) Since the sub-lessee Burger is not in privity of estate with the landlord, the sub-lessee is not subject to the covenants or terms of the leasehold agreement. Therefore, Moore will remain liable for the entire $300 rental. As a result, choices (A) and (B) are incorrect. Choice (C) is wrong because McGuirk, as assignor, would not be the only party liable.

Landley owned a large building in the city of Ames. On January 15,1979, Landley leased the building to Tennance for a period of 20 years at a rental of $10,000 per month. The lease¬hold agreement between Landley and Tennance provided that the latter was not permitted "to assign this lease to anyone except a corporation with an 'A' credit rating from the Delmarva Credit Rating Corporation." On February 1, 1980, Tennance leased the premises to Aruba Inc., a corporation which did not have the required credit rating. The Tennance-Aruba lease was for a period of 5 years, with a rental of $15,000 per month, payable by Aruba to Tennance. In addition, Aruba agreed to abide "by all of the terms and conditions of the lease between Landley and Tennance." One year later, Aruba Inc. leased the premises to Simon for the balance of the term of the Aruba-Tennance lease. Simon took possession of the said premises on February 1, 1981, the same day that Aruba Inc. vacated its occupancy. Pursuant to the Aruba-Simon leasehold agreement, the latter was obligated to pay a monthly rental of $17,500 directly to Aruba Inc. Simon has a 'B' credit rating with the Delmarva Credit Rating Corporation. For one year, Simon paid $17,500, each month directly to Aruba Inc. During that same period, Aruba Inc. con¬tinued to pay $15,000 each month to Tennance while the latter paid $10,000 (each month) to Landley. Landley knew about the leases to Aruba Inc. and Simon, and protested promptly, but took no further action, apparently satisfied as long as he received his $10,000 per month from Tennance. On February 1, 1982, Simon abandoned the premises and stopped paying rent to Aruba Inc. After Simon discontinued pay¬ing rent, Aruba Inc. stopped paying rent to Tennance. When Tennance failed to receive his rent, he, too, stopped paying rent to Landley. The building is now vacant, and Tennance refuses to pay rent until the air conditioning is fixed. Simon has returned his keys to Aruba Inc., and the latter has returned its keys to Tennance. However Tennance has not returned any keys to Landley yet. Simon's abandonment was caused by the destruction of the air conditioning unit in a fire apparently set by some vandals. The damaged unit was located in an area inside the store that was used for storing merchandise. Simon has flatly refused to repair or replace the equipment at his own expense. Despite Simon's repeated demands, Aruba, Tennance and Landley have all refused to replace the air conditioning system. 44. Which of the following accurately states the legal effect of the non-assignability clause contained in the Landley-Tennance leasehold contract? (A) The non-assignability provision had no legal effect. (B) The non-assignability provision made the assignment from Tennance to Aruba ineffective. (C) The Tennance-Aruba lease did not effectuate a breach of the Landley-Tennance contract. (D) Although the Tennance-Aruba lease constituted a breach of the Landley-Tennance contract, Landley would nevertheless be required to recognize the validity of the transfer (of the premises) to Aruba.

44. (A) Choice (A) is the most accurate statement. The question asks in so many words "What is the legal effect of the non-assignability clause in the Landley-Tennance contract?" Well, the non-assignability provision had no legal effect. Since Tennance's transfer (of the premises) to Aruba constituted a sublease, not an assignment, the said provision would have no legal effect. Choice (C) is less correct because, although factually true, it does not answer the question asked. Students must know the distinction between an assignment and a sublease. An assignment is a transfer of tenant's entire leasehold estate to another; whereas a sublease is a transfer of anything less than tenant's entire leasehold estate. In the example here, Tennance entered into his leasehold agreement with Aruba one year after his contract with Landley.

45. If Landley brings suit to recover for past rent due, which of the following is (are) correct? I. Landley may recover against Tennance for past rent due. II. Landley may recover against Aruba for past rent due. III. Landley may recover against Simon for past rent due. (A) I only (B) I and II (C) II only (D) I, II and III

45. (A) In the case of a sublease, privity of estate and privity of contract exist between the original landlord and tenant. Similarly, privity of estate and privity of contract exist between the tenant and subtenant. However, neither privity of contract nor privity of estate exists between the orig¬inal landlord and subtenant. Therefore, the landlord cannot sue the subtenant. As a conse¬quence, Landley may not recover against subtenants Aruba and Simon. He may only recover against tenant-Tennance under the terms of their original leasehold contract.

46. Which of the following accurately states the legal relation¬ship^) of the various parties? I. Privity of estate and privity of contract exist between Landley and Tennance. II. Privity of estate and privity of contract exist between Tennance and Aruba. III. Privity of estate but not privity of contract exists between Landley and Aruba. IV. Neither privity of estate nor privity of contract exists between Landley and Simon. (A) I and II, but not III and IV (B) II and III, but not I and IV (C) I and IV, but not II and III (D) I, II and IV, but not III

46. (D) As noted in the previous explanation, there is privity of estate and privity of contract between the landlord and the tenant. Likewise, privity of estate and privity of contract exist between the tenant and subtenant. However, neither privity of contract nor privity of estate exists between the landlord and the subtenant.

Question 47 is based on the following fact situation. Cornell was the owner of a vacant warehouse which was located in the township of Richmond. On May 1, Cornell leased the warehouse to Yale for a term of five years with a renewable option. The leasehold agreement provided in part: "Yale hereby acknowledges that certain areas of the leased premises are in a state of disrepair and unsafe for the conduct of business, Nonetheless, Yale agrees to assume full responsibility for the necessary repairs. Furthermore, Yale agrees to indemnify Cornell for any loss resulting from the condition of the said premises." Under the terms of the lease, Cornell delivered possession of the warehouse to Yale on May 2. On that date, Brown, who was an employee of Yale's, was working in the warehouse. He was moving some equipment into the warehouse when a section of the ceiling collapsed and fell on him. At the time Cornell and Yale entered into their lease, only Cornell was aware that the ceiling was defective and needed to be repaired. Although Cornell could have remedied the defect before delivering pos¬session to Yale, he (Cornell) didn't perform any repair work on the ceiling. Brown initiated suit against Cornell to recover dam¬ages resulting from his injury. There are no applicable statutes. 47. If Brown prevails it will be because a (A) landowner is strictly liable for injuries occurring on his property (B) landowner's duty of care to third parties cannot be shifted to a tenant by the terms of a lease (C) tenant cannot waive the implied warranty of commer¬cial habitability (D) covenant to indemnify by a tenant in favor of a land¬owner is against public policy

47. (B) At common law, a landlord generally does not impliedly warrant that the leased premises are suitable for any particular purpose, and he is not liable for a dangerous condition existing on the leased premises. Normally, the doctrine of caveat emptor prevails, but there are two notable exceptions. First, Smith and Boyer point out that "A landlord may be liable in tort to the tenant, his guests, licensees and invitees, if at the commencement of the lease there is a dangerous condition which the landlord knows or should know about and the discovery of which would not likely occur by the tenant exercising due care." Property, pg. 138. Based on this exception, choice (B) is correct because the dangerous condition existed at the commencement of leasehold period (note: the facts indicate that Brown was injured on the first day Yale took possession of the warehouse). The second exception is that a landlord in the lease of a completely furnished dwelling for a short period of time impliedly warrants the fitness of the premises and the furnishings.

53. Upon O's death in 1977, his sole heir, B, brings an ejectment suit to recover possession of the property from A. In his action, B will most probably: (A) lose, because A has acquired title by adverse possession (B) win, because the statutory period for adverse possession does not run against a remainderman until his interest becomes possessory (C) lose, since A was successful in the action by O, B would be estopped from instituting another action against A for ejectment (D) win, since his title was perfected at the time of conveyance

53. (A) Since A was in continuous possession of Greenacre from 1957 to 1977, he would acquire title by adverse possession. As a result, choice (A) is correct. Students should note that choice (B) is incorrect because the facts do not disclose that B was a remainderman or had a remainder interest in Greenacre. Rather, O simply devised the property to B in 1961. As a con¬sequence, B did not have a remainder interest but a present possessory interest as of 1961.

Woody, a famous environmentalist in the State of Sylvania, was the owner of Pocono Woods, a large tract of virgin forest. In 1960, Woody conveyed Pocono Woods "to the people of the State of Sylvania in fee simple; provided, however, that if any portion of said tract shall ever be used or developed for any commercial purpose or otherwise converted from its natural state (with exception for recreational, scientific or educational purposes), then the grantor or his successors in interest may re¬enter as of the grantor's former estate." Under Sylvania law, conveyances of real property to "the people" of the State are deemed to vest title in the State regard¬ed as a legal entity, and custody over such property resides in an appointed official known as the Director of Environmental Resources. In 1970, the Director granted Crockett Company the privilege to cut timber on a remote portion of Pocono Woods, together with incidental easements of way. The section that Crockett Co. was to clear would be used for the development of a state recreational area. After the privilege was granted, Crockett proceeded to clear the timber which he later sold for $10,000. When Woody learned that Crockett was cutting the timber and selling it, he notified the Director to desist further exploitation of the land. 48. In 1960, the interest of the State of Sylvania in Pocono Woods would be best described as a (A) fee simple determinable (B) fee simple subject to condition subsequent (C) easement appurtenant (D) determinable fee subject to an executory interest

48. (B) In 1960 the state of Sylvania's interest in Pocono Woods would be described as a fee simple subject to condition subsequent, which means fee simple subject to being terminated by exercise of the power of termination or right of re-entry for condition broken. The important characteristic which distinguishes this type of estate from a fee simple determinable is that the estate will continue in the grantee, or his successors, unless and until the power of termination is exercised. Although no particular words are essential to create an estate on condition subsequent, the use in the conveyance of the traditional words of condition, "upon condition that," "provided that," or "but if" - coupled with a provision for re-entry by the transferor or the occurrence of the stated event will normally be construed to manifest an intention to create an estate on condition subsequent.

49. The right of Crockett to cut and sell the timber which was to be cleared from Pocono Woods would be an example of a (an) (A) license (B) easement appurtenant (C) easement in gross (D) profit-a-prendre

49. (D) A profit-a-pendre is similar to an easement, in that it is a non-possessory interest. The holder of the profit is entitled to enter upon the servient tenement and take the substance of the land (e.g., minerals, trees, oil or game) subject to the privilege. In this regard, a profit, like an easement, may be appurtenant or in gross.

For many months, Drew had been negotiating with Rollins for the purchase of Treeacre. Finally, on August 18, 1994 Drew and Rollins entered into a real estate sales contract which provided in part: "I, Rollins, agree to convey good and marketable title to Drew sixty days from the date of this contract." The stated pur¬chase price for Treeacre was $175,000. On October 11, 1994. Drew telephoned Rollins and told him that his title search indicated that Wilkins, not Rollins, was the owner of record to the property. Rollins responded that, notwith¬standing the state of the record, he had been in adverse pos¬session for twenty-one years. The statutory period of adverse possession in this jurisdiction is twenty years. The next day Drew conducted an investigation which revealed that Rollins had in fact been in adverse possession of Treeacre for twenty-one years. At the time set for closing, Rollins tendered a deed in the form agreed in the sales contract. Drew, however, refused to pay the purchase price or take possession of Treeacre because of Rollins' inability to convey "good and mar¬ketable title." 5. In an appropriate action by Rollins against Drew for specific performance, the vendor will (A) prevail, because he has obtained "good and marketable title" by adverse possession (B) prevail, because Rollins' action for specific perfor¬mance is an action in rem to which Wilkins is not a necessary party (C) not prevail, because an adverse possessor takes title subject to an equitable lien from the dispossessed owner (D) not prevail, because Drew cannot be required to buy a lawsuit even if the probability is great that Drew would prevail against Rollins

5. (D) This is an extremely tricky Multistate example dealing with conveyancing. Students should be aware that title established through adverse possession is free from encumbrance and of a character to assure quiet and peaceful enjoyment of the property by the vendee, but it is not a "marketable" title of record until there has been a judicial determination of such title. To show a record title by adverse possession requires a suit and the recording of a decree. Even though a court may determine that the vendor had title by adverse possession, the vendee did not bar¬gain for that kind of title when the contract required a "marketable" title of record. Therefore, in accordance with the prevailing view, choice (D) is correct. This specific problem is referred to in Smith and Boyer, Law of Property, pg, 264.

50. In an ejectment action instituted by Woody in 1971 against the State of Sylvania to recover Pocono Woods, what result? (A) Woody would succeed, since Crockett's selling of the timber would entitle Woody to re-enter and terminate the grant to the State. (B) Woody would succeed, since Crockett's selling of the timber would constitute an automatic termination of the State's fee in Pocono Woods. (C) Woody would not succeed, since the Director approved the clearing of the timber in order to develop the area for recreational purposes. (D) Woody would not succeed, since the reservation of Woody's right to terminate would be violative of the Rule Against Perpetuities.

50. (C) Woody will not succeed since the Director's objective in the clearing of the timber was for the purpose of developing a recreational area. Although Crockett was not privileged (under the terms of Woody's deed over to Sylvania) to use the tract for his own commercial advantage, the court will look to the ultimate purpose for which the land was to be used (i.e., recreational park). Thus, the court will conclude that Woody's original intention and expectation in conveying Pocono Woods to the state was not materially or adversely affected.

51. In 1968, O wanted to move back to Greenacre because he desired to return to the country. Finding A in possession, O initiated a suit in ejectment against A. The most probable judicial determination would be: (A) O would lose, since A's possession was open, hostile and continuous (B) O would lose, because A had fulfilled the required statutory period to become an adverse possessor (C) O would win, since A's possession was not continuous (D) O would win, because he is the title-holder of record

51. (C) O would win since A's possession was not continuous. For one to hold property adversely, his possession must be: (1) actual and exclusive, (2) open and notorious, (3) continuous and peaceable (meaning without any interruption) and (4) hostile and adverse. In this case, since A's possession was interrupted he didn't fulfill the statutory requirement of continuous possession for 20 years.

52. If A's nephew agreed to live and farm the Greenacre prop¬erty during A's absence, and A then returned from his moth¬er's home in 1957 and thereafter, continued in possession until 1968 when O brought the ejectment action, the court would most likely hold that: (A) O was the rightful owner and thus entitled to possess Greenacre (B) O was entitled to possession since A's intermittent pos¬session was insufficient to make him an adverse possessor (C) A was entitled to possession because his possession was open, notorious, continuous and hostile as required for the statutory period (D) A was entitled to possession because of the privity between A and his nephew which aided him in fulfilling the required statutory period

52. (D) Choice (D) is correct since the period of adverse possession of one possessor can be tacked to the period of adverse possession of another possessor when there is privity (e.g. blood or con¬tract) between the two.

Question 6 is based on the following fact situation. Testator owns Pineview in fee simple. Testator executes and delivers a deed to Pineview with the following granting clause: "To Peter and Paula for their joint lives and upon the death of the first of them to die to the survivor, but if Patrick outlives both of them, then to Patrick." 6. As a result of the grant, which of the following is the most accurate statement regarding the ownership rights of Pineview? (A) Peter and Paula are joint tenants in fee simple defeasi¬ble, and Patrick has an executory interest. (B) Peter and Paula have a joint life estate for the life of the one who lives the longest, and Peter, Paula and Patrick have contingent remainders. (C) Peter and Paula are tenants in common for the life of the first of them to die, Peter and Paula have contingent remainders in fee simple defeasible, and Patrick has an executory interest. (D) There is no legal difference between the results reached in alternatives (A), (B) or (C).

6. (B) First of all, the conveyance created a life estate in Peter and Paula for their "joint lives" in Pineview. This life estate in the joint lives of Peter and Paula could only last until one of them died. In other words, at the death of either Peter or Paula, the life estate terminated. Secondly, Peter, Paula and Patrick each have a contingent remainder in fee simple in Pineview. Contingent remainders are future interests created in favor of a transferee at the same time and in the same instrument as the prior estate and they are subject to a condition precedent. The condition precedent is a happening or non-happening which must occur on or before the termination of the prior estate. In our factual situation, Peter, Paula or Patrick's taking of the estate is subject to a condition precedent: that one of the parties must survive in order to take. Simply stated, the contingency of survival is expressed as the condition precedent.

Bobby Saul Estes, a resident of El Paso, had been a Texas state senator for three years. During that period of time, it had been Bobby Saul's regular practice to stay at the Four Seasons Hotel in Austin while the legislature was in session (namely from September through March). As a general rule, Bobby Saul stayed at the hotel each and every night during that seven month peri¬od. When Bobby Saul checked into the hotel on September 1, 1997 for the 1997-1998 term, the hotel manager quoted Bobby Saul his usual senatorial discount rate of $50 per night. Although nothing was said regarding payment terms, Bobby Saul custom¬arily paid the bill in weekly installments. During the months of September and October, he paid the hotel at the end of each week $350 plus tax and other accumu¬lated expenses. On the last day of October, Bobby Saul made his usual weekly payment covering the rental period from October 25th to October 31st. Then on the morning of November 4th (before check-out time for that day), Bobby Saul decided to suddenly move out of the Four Seasons and register into the Ritz Carlton, a new hotel located two blocks away. He notified the manager at the Four Seasons of his intent to check out and tendered a check for $150 plus tax and other incidental charges incurred from November 1st through November 3rd. When the manager noticed that Bobby Saul was only paying for three days, she remarked, "You still owe us $200 plus tax for the rest of the week and $350 plus taxes for an additional week because you didn't give us a week's notice that you planned to vacate." Bobby Saul refused to pay the additional charges. 60. If the Four Seasons Hotel brings suit against Bobby Saul to recover for the unpaid balance, how many additional days lodging will he be required to pay for? (A) None (B) Four (C) Seven (D) Eleven

60. (A) Every leasehold includes the following elements: (a) an estate in the tenant, (b) a reversion in the landlord, (c) exclusive possession and control of the land in the tenant, and (d) generally, a contract between the parties. The first three elements constitute privity of estate, and the fourth is privity of contract. Whereas in a lease exclusive possession must reside in the tenant, in a license the possession remains in the licensor and the licensee has a mere privilege of being on the land without being treated as a trespasser. Smith and Boyer, Real Property, p. 48. Ordinarily a lodging contract does not create a landlord-tenant relationship but only that of licensor and licensee, whereby the occupant contracts for the use of a room and facilities without a posses¬sory interest in the land. Exclusive possession and control remains in the landowner. In this question Bobby Saul, as a hotel guest, does not have exclusive possession and control of his hotel room. He is a licensee rather than a tenant. Furthermore, the facts do not strongly support a periodic tenancy since his oral agreement with the Four Seasons Hotel was not referred to as a "lease", nor was the weekly payment referred to as "rent." Therefore, no seven-day prior notice will be required. Thus, in the absence of any leasehold Bobby will owe no additional balance beyond the day he checks out. Choice (A) is correct.

Jim is the fee simple owner of Blue Bayou, a one thousand acre tract of land in Louisiana. Half of Blue Bayou is swampland totally unfit for cultivation, but heavily covered with valuable hardwood timber. The other half of Blue Bayou is also covered with valuable timber, but is land which is fit for the raising of crops when cleared of the timber thereon. The latter section of land is more valuable for cultivation than for the growing of timber. In 1972, Jim conveyed his Blue Bayou tract to his brother Tim for life. At the time of the conveyance, the swampland had never been used for the production of timber. Tim took possession and cleared forty acres of the timber on the section that was suitable for cultivation. In addition, Tim cut sixty acres of timber in the swampland, thus becoming the first person to exploit this area. He then sold the timber from the swampland for $2,400; he sold the timber from the area adaptable for cultivation for $2,000. Tim then proceeded to clear some timber on the tract and used it as "estovers" for the purpose of utilization in repairing fences, buildings, equipment and the like on the Blue Bayou property. 61. In an action by Jim to permanently enjoin Tim from cutting any more timber on the swampland section of Blue Bayou and to account for profits received in the sale of the timber, Jim will most likely (A) succeed, since a life tenant must account for permissive waste to the reversioner or remainderman (B) succeed, since a life tenant may not exploit natural resources where no such prior use had been made (C) not succeed, since a life tenant is not liable for ameliorative waste (D) not succeed, since a life tenant has a right to make reasonable use of the land.

61. (B) The swampland is not now and cannot be made usable for the growing of crops. It seems to be capable of growing trees only. This swampland, as far as the facts disclose, has never been used and at the time of the taking effect of Tim's life estate was not being used for the produc¬tion of lumber. Tim is the first to cut timber thereon. At this point an analogy may assist in the analysis and understanding of the problem. It is well settled that a tenant for life or for years has a right to continue the operation of old mines on the leased premises for the reason that such is the use to which the land has been put or is being put at the time of the beginning of the ten¬ant's estate. The lease implies that such use may be continued. On the other hand, such tenants have no right to open new mines and appropriate the minerals therefrom. The reason is clear. It has taken eons of time to produce the minerals which underlay the land. It is the very sub¬stance of which the land is made and that corpus should not be available for consumption or destruction by a tenant of a limited interest in the premises, unless of course the lease is made for the very purpose of mining. Likewise, it has taken generations of time to produce the hard-wood forests of the swampland and the value of that forest is the very heart and substance of that portion of Blue Bayou. It should not be available for consumption and destruction by a ten¬ant of a limited estate as against the reversioner or remainderman who owns the inheritance. Therefore, Tim should be permanently enjoined from cutting any more trees on the swampland, and should account for the proceeds from the sale of the timber which was cut on the swampland.

62. Tim's cutting of the timber on the section of Blue Bayou suitable for cultivation would be an example of (A) permissive waste (B) ameliorative waste (C) equitable waste (D) unreasonable exploitation

62. (B) Ameliorating waste is a change in the physical characteristics of the occupied premises by an unauthorized act of the tenant which increases the value of the property. Herein, any change in the characteristics of land which makes it more productive and adds to the store of consumable goods should be given all the protection which sound legal principles permit. It takes much time, labor and power to convert timberland into tillable fields for the production of annual crops. The law takes account of such by awarding to him the timber which is removed for such purpose. Thus, the tenant for life or for years has a right to use land as good husbandry dictates. As such, this usually includes the right to transform timberland into arable land for the plow, provided of course the soil can be made suitable for such purpose.

Chet Walker died in 1940, owning a tract of land in New Mexico which was then being used for a gold mine, and leaving a will (executed in 1935) in which he specifically devised "my gold mine in New Mexico to my beloved brother, Luke, whatev¬er is left over at his death to go first to his widow if he leaves one and then to his surviving heirs." Chet left the rest and residue of his ample estate in trust for his wife, Wendy, for life, remainder to his son Hal. Chefs only surviving family were, Luke, Wendy, and Hal. Luke continued to mine the land for a number of years, ceas¬ing operations after the profit margin from mining had been destroyed by a combination of increasing costs, diminishing returns, and legal controls on gold prices. Luke has now acquired some adjacent land which he pro¬poses to use for a mountain resort-hotel. In conjunction with the hotel, he would like to use the tract devised to him by Chet for a riding stable and pasture. This would entail dismantling the mining structures, filling excavations, and constructing the sta¬ble buildings. Luke is married and has a 30-year-old son named Wilt, with whom he is not on friendly terms. Wendy and Hal are both still alive. Luke does not want to proceed with redevelop¬ment if there is any significant risk of his being enjoined from continuing. 63. Will anyone be able to obtain an injunction against Luke's proposed redevelopment of the gold mine? (A) Yes, because a life tenant or tenant for years is liable to the reversioner or the remainderman for voluntary waste. (B) Yes, because a life tenant or tenant for years is liable to the reversioner or the remainderman for permissive waste. (C) Yes, because a life tenant or tenant for years is liable to the reversioner or remainderman for equitable waste. (D) No, because a life tenant or tenant for years is ordinar¬ily not liable to the reversioner or the remainderman for ameliorating waste.

63. (C) It is of utmost importance to understand and recognize the difference between actions at law and in equity. Injunctive relief is an equitable remedy. Thus, choice (C) is correct because equitable waste is an injury to the reversionary interest in land which is inconsistent with good husbandry and is recognized only by the equity courts. Be aware that it (i.e., equitable waste) does not constitute legal waste as recognized by the courts of law. On the other hand, all other alternatives are incorrect because (a) voluntary waste, (b) permissive waste and (c) ameliorating waste are types of legal waste, wherein the life tenant or tenant for years will be liable for mon¬etary damages.

Widom is the owner of a large two-story office building in downtown Long Beach. Widom, an architect, designed and supervised the building's construction. He and his associate, Weed, occupy the first floor for their architectural business. On June, 1986, Widom entered into a valid and binding written lease with Coltrane to rent the second floor, office space for four years at a monthly rental of $5,000. The lease contained a pro-vision wherein Widom was required to repaint the second floor premises after two years of occupancy. On June 1, 1988, Coltrane sent Widom a written letter requesting the repainting of the second floor office space. Widom made no reply and failed to perform the repainting as the lease required. The cost of repainting Coltrane's office was estimated at $1,750. On July 1, 1989, Widom had still not repainted the premises. The following day, Coltrane moved out, mailed the keys to Widom, and refused to pay any more rent. The lease was silent as to the rights and remedies of Coltrane due to Widom's failure to perform the repainting. There is no applicable statute in the jurisdiction. 69. In an appropriate action by Widom against Coltrane for the rent due, Widom will (A) win, because there was no constructive eviction (B) win, because Coltrane had the remedy of self-help (C) lose, because he cannot maintain an action for rent while in breach of an express covenant (D) lose, because the obligation to pay rent is dependent on Widom's performance of his express covenant

69. (A) At common law, since a tenant was deemed to receive an estate in land, his rights and duties were treated as independent of the landlord's rights and duties. Thus, if the landlord promised to keep the property in repair, a breach of this promise did not relieve the tenant from the duty of paying rent. According to the common law rationale, the rent was owed as payment for the estate, and the promise to do repairs was merely a collateral promise which could be enforced only by a separate contractual suit brought by the tenant. This doctrine is generally referred to as the independence of covenants. As a result, choices (C) and (D) are incorrect. Choice (B) is wrong because the reason the landlord will prevail is because the premises were not uninhabitable (the basis for constructive eviction) rather than because the tenant failed to perform the re-painting himself. In other words, self-help is not an appropriate remedy inasmuch as the tenant had no duty to make such repairs.

Luzinski was the fee simple owner of Grassacre, a 20 acre tract located in northern Iowa. When Luzinski moved to Florida, Fisk took possession of Grassacre. Fisk's possession has at all times complied with the requirements of the applicable adverse possession statute in effect. Twelve years after Fisk took possession, Luzinski died intes¬tate, leaving Carlton, his six-year-old son, as his only surviving heir. Nine years after Luzinski's death when Carlton was fifteen, Law, who was Carlton's guardian, discovered that Fisk was in possession of Grassacre. In Iowa the statutory period of adverse possession is twenty years and the age of majority is eighteen. 7. Which of the following correctly describes the state of title to Grassacre? (A) Fisk has acquired title by adverse possession. (B) Fisk will not acquire title unless he continues in adverse possession for an additional three years, or until Carlton reaches the age of eighteen. (C) Fisk will not acquire title unless he continues in adverse possession for an additional eight years, making a total of twelve years after Luzinski's death. (D) Fisk will not acquire title unless he continues in adverse possession for an additional twelve years, or nine years after Carlton attains the age of eighteen.

7. (A) Here's a highly confusing Multistate Property example. Quite often, you may be confront¬ed with a perplexing problem requiring mathematical computation. In this situation, don't get flustered! Stay cool! Obviously, this is an adverse possession problem. First, the facts indicate that the statutory period of adverse possession in this jurisdiction is 20 years. Looking back, the facts tell you that 12 years after Fisk took possession (of Grassacre), Luzinski died. Next, the facts indicate that 9 years after his death, his son's guardian discovered Fisk in (adverse) possession of Grassacre. Now, adding the 12 years (before Luzinski's death) + the 9 years (following his death) = 21 years. Thus, since Fisk's adverse possession is beyond the statutory period, choice (A) is correct. This is an extremely popular Multistate example because the test maker knows that many students will incorrectly choose either choice (C) or (D) because they are familiar with the rule wherein one who is under a disability at the time of the accrual of the cause of action against the adverse possessor, is given by statute, time beyond the removal of his dis¬ability in which to bring his action. This defense, however, is not applicable here because the facts do not indicate that this Jurisdiction has enacted such a statute.

In a jurisdiction using the grantor/grantee indices, the follow¬ing events have occurred in the order listed below: (1) In 1942, Alpha conveyed to Beta a tract of land now com¬posing Greenacre, by a deed which was immediately recorded. (2) In 1962, Beta died a widower and devised Greenacre to his son, Gamma, by a will that was duly admitted to probate. (3) In 1973, Gamma mortgaged Greenacre to Epsilon Mortgage Company to secure a note for $25,000, due on December 31, 1983, with 9.2% interest per annum. (4) In 1976, Epsilon recorded the mortgage on Greenacre. (5) In 1983, in a signed writing which Gamma delivered to his daughter, Delta, Gamma promised to convey Greenacre to Delta by a quitclaim deed and pay the mort¬gage debt when it came due. 70. If Gamma refused to convey Greenacre to Delta and Delta brings suit against Gamma to compel a conveyance of the property, Delta will most likely (A) prevail, because there was an effective gift (B) prevail, because there was an effective assignment (C) not prevail, because there was no consideration to sup¬port Gamma's promise to convey the property (D) not prevail, because specific performance will not be granted where there is an adequate remedy at law

70. (C) The Statute of Frauds requires a writing (1) identifying the parties (2) describing the subject matter (3) stating the purchase price and (4) signed by the party to be charged. The writing Gamma delivered to Delta did not state any consideration — it was intended as a gift. Furthermore, it was not an effective gift because it was only a promise to convey. An effective gift requires a present transfer of an interest. If the transfer is to take effect in futuro, it is a mere promise to make a gift and unenforceable for lack of consideration. Smith and Boyer, Survey of the Law of Property, p. 469. Therefore, choice (A) is incorrect, and choice (C) is the correct answer. Since Gamma's writing did not create an assignment, choice (B) is incorrect. Choice (D) is incorrect. Delta does not have an adequate remedy at law for lack of considera¬tion, nor will specific performance be granted for lack of mutuality.

71. Suppose that Gamma did, in fact, deliver to Delta a quit¬claim deed which made no reference to the mortgage. If Gamma thereafter defaulted on the mortgage debt, which of the following is probably correct? (A) Only Delta has a cause of action against Gamma. (B) Only Epsilon has a cause of action against Gamma. (C) Both Delta and Epsilon have causes of action against Gamma. (D) Neither Delta nor Epsilon has a cause of action against Gamma.

71. (B) A mortgagee has both an in personam claim against the mortgagor on the debt or obligation and an in rem action against the security. As a result, choice (B) is correct because mortgagee-Epsilon has an in personam action against mortgagor-Gamma. Note that choices (A) and (C) are wrong because Gamma's conveyance to Delta was by a quitclaim deed. A quitclaim deed usually contains no assurances; it simply operates as a transfer to the grantee of whatever interest, if any, the grantor happens to own.

Hancock, the owner of Hancock House Apartments, orally entered into a five year lease with Jay, with the rental period to commence on February 1, 1978. The lease provided that Jay would pay Hancock rent at the rate of $3,500 per year for a one bedroom apartment. Jay made payments of $3,500 on the first day of February in 1978 and 1979. On March 31, 1979, Hancock notified Jay and the other tenants of the apartment building that he had sold the building that day to Madison. The following day, Madison, the new owner, notified Jay that the building would be converted to a condominium and that Jay's lease would be terminated as of December 31, 1979. However, Jay would be entitled to purchase the condominium unit (an interest in real estate subject to recording statutes) at a special resident's discount price of $32,500, if Jay signed an agreement of sale within six months. Jay decided not to purchase the unit, because he had already planned to move away to the city of Anatevka. However, Fred, who was Jay's friend, wanted to buy Jay's unit at the discounted price. Since Madison would sell to Fred only at the nonresident's price of $38,500, Fred consulted a lawyer and determined how he could purchase at the lower price by a series of transactions. Jay signed an agreement of sale on May 1, 1979 and pur¬chased the unit at the discounted price. Closing was held on September 15, 1979. On September 16, 1979, Jay conveyed his entire interest to Fred for $32,500. The two deeds were record¬ed immediately, in proper order. On July 3,1980, Fred signed an agreement to sell his interest in the unit to Miss Kitty for $47,500. Closing was scheduled for September 17,1980. 73. On April 2, 1979 considering all appropriate defenses, the termination date of Jay's lease was: (A) September 15,1979 (B) December 31,1979 (C) January 31,1980 (D) January 31,1983

73. (C) The termination date of Jay's lease was January 31, 1980. The facts state that on February 1, 1979, Jay had made the yearly rental payment of $3500 to Hancock for the year 1979. Therefore, Jay had complied with his obligation in accordance with the original leasehold agreement. It is important to note that Hancock and Jay entered into an oral leasehold agreement for five years. In the majority of states, such a parol lease for a period not to exceed one year from the making thereof is valid. However, parol leases which exceed the one year's time period are generally invalid to create such a tenancy for years because of the Statute of Frauds. But as a general rule, the Statute of Frauds only affects the duration of the lease. Thus any covenants and conditions agreed upon in the lease such as the rental obligations, are still valid as long as the landlord-tenant relationship exists.

74. If Miss Kitty properly checks the real estate records prior to her closing she would (A) look for Fred's name in the grantee index (B) look for Fred's name in the grantor index (C) look for Fred's name in the grantee and grantor index (D) not look for Fred's name in either the grantee or grantor

74. (C) The grantor and grantee indices are utilized in most jurisdictions today in the title search of real state records. In the grantor index, all conveyances are indexed alphabetically and chronologically under the initial letter of the grantor's surname. In the grantee index, a like index is made of conveyances under the initial letter of the grantee's surname. A title searcher must trace both indices in order to complete his search. Therefore, Miss Kitty should properly trace Fred's name in both the grantor index and the grantee index.

76. Assume the following facts for this question only. In 1965, Dooley constructed a driveway from Looney Lane over what is now Parcel 2. This driveway, which extended over Parcel 1 where it connected with Lincoln Boulevard, was used con¬tinuously by Dooley until 1997 when he conveyed his prop¬erty to Perez and Eagleson. When Eagleson took possession of Parcel 2, he immediately began to use the driveway across Parcel 1. Thereafter, Perez requested that Eagleson cease to use the driveway over Parcel 1. After Eagleson refused, Perez brought an appropriate action to enjoin Eagleson from using the driveway. In this action, Perez will (A) win, because his deed antedated Eagleson's deed (B) win, because no encumbrances were mentioned in his deed (C) lose, because Eagleson has an easement by necessity (D) lose, because Eagleson has a prescriptive easement

76. (C) Perez will not be able to enjoin Eagleson's use of the driveway across Parcel 1 because Eagleson has an easement by necessity. Choice (C) is correct. An easement by necessity requires the following; (1) existence of a quasi-easement at the time of conveyance; (2) apparent use; (3) continuous use; (4) reasonable necessity, where the grantee receives the dominant tenement. A quasi-easement existed when Dooley conveyed to Eagleson. At that time Parcel 2 had no other access to a public road, thereby allowing Dooley an easement by strict necessity over Perez' land, which passed to Eagleson upon conveyance of Parcel 2. Choice (D) is incorrect because Dooley's use was not adverse to anyone—he owned both parcels—until 1986, so no prescriptive easement is possible. Choices (A) and (B) reach the wrong conclusion and are incorrect because of the existence of the easement by necessity.

In 1954, Oswald, a devoted conservationist, was the owner of Greenacre, a 100 acre tract of undeveloped land. In that year, Oswald conveyed Greenacre "to Grant in fee simple, provided, however, that the grantee agrees that neither he nor his heirs or assigns shall ever use the property for any commercial purpose." The deed further stipulated that "if any portion of said tract is used for other than residential purposes, then the grantor or his successors in interest may re-enter as of the grantor's former estate." This deed was properly recorded. Grant died intestate in 1972, survived by Wilma, his wife. Oswald died in 1975, sur¬vived by his children, Carla and Carrie, his only heirs. During the period between 1954 and 1980, the spreading development from the nearby city of Greenville has begun to engulf Greenacre. Though still undeveloped, Greenacre has become surrounded by office buildings, shopping malls and other commercial edification. In 1982, Wilma executed and delivered to Miles a fee simple conveyance of Greenacre, which Miles immediately recorded. The deed did not contain any ref¬erence to the restriction noted above. After Miles acquired title to Greenacre, he planned to commence construction of a hotel complex on a portion of the tract that bordered an apartment building. The applicable recording statute in effect in this juris¬diction provides, in part "No deed or other instrument in writ¬ing, not recorded in accordance with this statute, shall affect the title or rights to, in any real estate, or any devisee or purchaser in good faith, without knowledge of the existence of such unrecorded instruments." If Carla brings suit to enjoin Miles from constructing the hotel, the plaintiff will most likely (A) win, because Miles is charged by the recording act with notice of the provision in the original deed (B) win, because either Carla or Carrie has the right of reentry for condition broken (C) lose, because the change in the character of the sur¬rounding area has rendered the restriction meaningless (D) lose, because a common development scheme had been established for the entire Greenacre tract

78. (B) In this example, Grant has a fee simple subject to a condition subsequent, which is an estate in fee simple that may be terminated by the conveyor, or those claiming under him upon the happening of a named event. Students should note that the important characteristic which distinguishes this type of estate from a fee simple determinable, is that the estate will continue in the grantee, or his successors, unless and until the power of termination is exercised. The basic difference, therefore, is that the fee simple determinable automatically expires by force of the special limitation contained in the instrument creating the estate, when the stated contingency occurs, whereas the fee simple on condition subsequent continues despite the breach of the specified condition until it is divested or cut short by the exercise by the grantor of his power to terminate. Although no particular words are essential to create an estate on condition subsequent the use in the conveyance of the traditional words of condition - "upon condition that," "provided that," "but if" - coupled with a provision for re-entry by the grantor will be construed to create an estate on condition subsequent.

79. Assume for the purposes of this question only that Grant developed Greenacre into a residential subdivision between 1960 and 1969. The 100 acre tract was divided into single acre lots all of which were sold during the same decade. Single family residential dwellings were construct¬ed on all of the lots except four which were sold to Home in 1968. The deed to the four lots contained the same restriction as stipulated in the Oswald-Grant deed. In 1980, the Federal Savings & Loan Co. acquired Home's four lots in a foreclosure proceeding, and in the same year sold the property to Holiday. If Holiday seeks to construct a service station on his parcel of land, which of the following arguments, if true, would offer him the best chance of implementation? (A) The restriction against residential use is void as violative of the Rule Against Perpetuities. (B) The commercial development surrounding the property has made the deed restriction unenforceable. (C) When Holiday purchased the property from Federal Savings & Loan, the deed did not contain any mention of the restriction. (D) The jurisdiction in which Greenacre is located approved a zoning ordinance permitting the commercial development of the area surrounding Holiday's property.

79. (B) An equitable servitude may be extinguished by the existence of conditions which make the purpose and object of the servitude impossible of achievement, such as change in the character of the neighborhood from a residential to a business area. Note choice (D) is incorrect because a zoning ordinance will not generally invalidate an equitable servitude.

Questions 8-9 are based on the following fact situation. 8. Wanda and Hugo own Blackacre as joint tenants. If Wanda transfers her interest, by quitclaim deed to Louis, without Hugo's knowledge, what interest, if any, does Louis have in Blackacre? (A) no interest (B) an undivided one-half interest with right of survivorship. (C) an undivided one-half interest without right of survivorship (D) a lien against the entire property

8. (C) If one joint tenant conveys his (or her) interest in the property to another, even if the con¬veyance is done secretly, severance occurs, whereby the right of survivorship is severed and a tenancy in common results. Based on the facts presented, Louis would take an undivided one-half interest in Blackacre without right of survivorship.

Question 80 is based on the following fact situation. Rose, owner of Roseacre, granted to Morgan, owner of Diamondacre, an easement of way. Rose then went to Japan to live. Morgan then moved into possession of Roseacre and used it openly and exclusively, paying the taxes, for 23 years. He did not use his easement during that period. Rose returned and tried to evict Morgan from Roseacre. The court held that Morgan had acquired Roseacre by adverse possession. Morgan then sold Roseacre back to Rose who then put a chain across the ease¬ment. Morgan has now brought an action to remove the chain. 80. In most jurisdictions, Morgan will (A) lose, because he abandoned his easement (B) lose, because he did not use his easement for the statu¬tory period of 20 years (C) lose, because, rather than using his easement, he used all of Roseacre (D) win, because mere non-use of an easement does not extinguish it

80. (C) This is an example of the extinguishment of an easement by merger. An easement appurtenant is terminated by merger if the dominant and servient tenements come into common own¬ership. Choice (A) is wrong because failure to use an easement or profit will not result in its extinguishment. Note, however, that non-use coupled with physical acts (clearly indicating the user's intent to abandon) may be sufficient to effectuate an extinguishment. As a general review, remember that easements (or profits) may be terminated in the following ways: (1) by merger, (2) by written release, (3) non-use + other acts indicating an intent to abandon, (4) by prescription, (5) destruction of servient tenement, (6) tax sale of servient tenement, or (7) by estoppel.

R.J. Longhorn was a cattle rancher who owned substantial oil and mineral interests in West Texas. In July, 1990, Longhorn acquired title to Alpine Flats, a 200 acre tract located just inside the city limits of El Paso. The property was purchased at a sheriff's sale after its former owner, N.E. Fectual, became delinquent on his property taxes. In 1997, Longhorn decided to relocate in California and sell his vast ranching and oil interests in the Southwest. In July, Longhorn conveyed Alpine Flats for a consideration of $1.00, receipt of which was acknowledged, "to Austin, his heirs and assigns, but if Austin's wife, Molly, dies without issue, to Rosalina and her heirs and assigns." After taking possession of Alpine Flats, Austin discovered considerable oil reserves on the property. He thus began oil drilling operations and removed large quantities of oil which he sold on the spot market. At no time did he give notice to Rosalina of his oil drilling activities. Months passed before Rosalina learned that Austin was selling oil from the drilling operations. 81. Rosalina has now brought an action in equity for an accounting of the value of the oil removed and for an injunction against any further oil removal. If the decision is in favor of Austin, it will be on account of the fact that (A) the right to remove oil is an incident of a defeasible fee simple (B) Rosalina has no interest in Alpine Flats (C) there was no showing that Austin acted in bad faith (D) the right to remove oil is an incident of the right to possession

81. (A/D) The owner of a possessory estate in land is under a duty to use his possessory rights in a reasonable manner. Physical damage to the land is waste if it causes a substantial diminution in the value of estates owned by others in the same land. Burby, Real Property, p. 33. However, the owner of an estate in fee simple has almost complete freedom of action with respect to the use and enjoyment of the land. Austin is a fee simple owner and as an incident of possession has "the right to the exclusive possession of the surface of the ground, the airspace above and the soil underneath." Smith and Boyer, Survey of the Law of Property, p. 71. Choice (D) is correct. Choice (A) is also correct, since the right to remove oil is an incident of possession of any fee simple interest, whether a fee simple absolute or a fee simple defeasible. Removal of oil turns on Austin's right of exclusive possession. Furthermore, students should remember in this case that Rosalina, as the holder of the shifting executory interest has no vested interest in Alpine Flats, but rather a mere expectancy, until such time that Molly might die without children. Until then, Austin can remove oil and his doing so as a fee simple owner does not constitute waste.

In 1850 the State of Baden conveyed to Evers a tract of land, which today composes the two farms of Breezyacre and Windyacre, by deed that was immediately recorded. Evers died a widower in 1900, devising all of his real property to his son Warner. The will was duly probated shortly thereafter. In 1915, Warner conveyed the Breezyacre-Windyacre tract to Tidwell. This deed was not recorded until after Warner's death in 1919. Warner's will, duly probated following his death, named Tidwell as sole legatee and devisee of Warner's entire estate. In 1918, Warner mortgaged the Breezyacre-Windyacre tract to the Baden National Bank. The mortgage instrument, which was recorded in 1918, recited that it was subordinate to a mort¬gage on the same land given by Warner to Miller in 1898 and recorded in 1898. In that instrument Warner purported to grant Miller a mortgage on the Breezyacre-Windyacre tract. In 1941, Tidwell conveyed the parcel known as Breezyacre to Rumson, retaining Windyacre. This deed was duly recorded, but did not mention any mortgage. In 1956, Rumson conveyed Breezyacre to Vincent, who promptly recorded the deed. Recorded at the same time was an instrument from the Baden National Bank discharging and releasing the 1918 mortgage, which recited that the underlying debt has been fully repaid. In 1970, Baggett entered into an agreement with Vincent, whereby Vincent would convey Breezyacre in fee simple to Baggett for the sum of $75,000. The closing date was set for January 15,1971. NOTE that all of the deeds mentioned in the aforementioned transactions are general warranty deeds. In addition, the State of Baden has a notice-type recording statute and follows a title the¬ory for mortgages. 84. In 1915, asserting that his title to the Breezyacre-Windyacre tract was held free of any claim by Miller, Tidwell instituted suit against Miller to quiet title to the property. Judgment for (A) Tidwell, since Warner did not own the property in 1898, and therefore, his mortgage to Miller would be ineffective (B) Tidwell, since a subsequent bona fide purchaser with¬out notice would prevail over a prior mortgagee (C) Miller, since under the after acquired title doctrine, where the mortgagor mortgages a larger estate than he possesses in the land and subsequently acquires such larger estate, it inures to the benefit of the mortgagee (D) Miller, since in the title theory state, a mortgagee would prevail over a bona fide purchaser

84. (B) Choice (B) is the preferred alternative since a subsequent bona fide purchaser without notice of a prior mortgage (e.g., the 1898 Warner-Miller mortgage) outside the chain of title would prevail over the prior mortgagee. It is important to note that Tidwell would not be charged with "constructive notice" of the 1898 Warner-Miller mortgage since Warner did not acquire title to the property until 1900. Therefore, Tidwell would be charged with constructive notice of the 1900 devise from Evers to Warner and the 1850 conveyance from the State of Baden to Evers. In sum, the 1898 Warner-Miller mortgage would fall outside Tidwell's chain of title.

86. Assume that on January 10, 1971, Baggett conducted a title search which revealed the existence of Miller's 1898 mort¬gage on the Breezyacre-Windyacre tract. Which of the following statements is most accurate with respect to that mortgage? (A) Baggett would be entitled to rescind the real estate contract with Vincent immediately. (B) The encumbrance renders title to Breezyacre unmarketable. (C) The mortgage would only encumber the Windyacre parcel, not the Breezyacre tract, since Tidwell's parti¬tion in 1941 created a joint tenancy. (D) The encumbrance would not entitle Baggett to rescind the real estate contract until closing on January 15th. 87. Assume for the purposes of this question only that on January

86. (D) There can be no rescission of an executory contract for the sale of land merely because of lack of title (e.g., existence of an encumbrance) in the vendor prior to the date when performance is due (on the "closing" date). Neither can a vendee place the vendor in default by tendering payment and demanding a deed in advance of the time and under circumstances not contemplated by the contract.

87. Assume for the purposes of this question only that on January 15,1971, the sale of Breezyacre is finalized, with Baggett paying Vincent $75,000 and Vincent executing a general warranty deed. The deed contains the following covenants of title: (a) Covenant for seisin; (b) Covenant of the right to convey; and (c) Covenant against encumbrances. After Baggett takes possession of Breezyacre, he learns of the Warner-Miller 1898 mortgage, which was not satisfied, and seeks monetary damages for breach of the covenant against encumbrances. Judgment for (A) Baggett, since the covenant against encumbrances is a guarantee to the grantee that the property is not subject to outstanding rights or interests. (B) Baggett, since the covenant against encumbrances would be breached at the time the deed was delivered, thereby entitling the covenantee to recover damages. (C) Vincent, since the covenant against encumbrances may only be breached, if at all, at the time of conveyance. (D) Vincent, unless the covenantee is disturbed in his actual enjoyment of the land thereby conveyed.

87. (B) Here, it is clear that Vincent's covenant against encumbrances was breached the very instance he conveyed Breezyacre to Baggett because the encumbrance of the 1898 mortgage burdened Breezyacre at that time. Students should note that, in this regard, even though the mortgagee never forecloses or threatens to foreclose and the subsequent purchaser is never called upon to pay off the encumbrance, there is, nevertheless, a breach of covenant and the covenantee may recover damages.

Question 88 is based on the following fact situation. Amos is the owner in fee simple of Blackacre, a 7-acre tract, on which he maintains a dwelling house for himself and his fam¬ily. Adjoining Blackacre is Whiteacre, a 10-acre tract, owned by Andy. In order to gain access to the highway, Amos has an ease¬ment to cross over Whiteacre. Amos has recently purchased Greenacre, a 12-acre tract, which abuts Whiteacre but is not appurtenant to Blackacre. Amos has begun constructing a farmhouse on Greenacre and is using the existing easement (across Whiteacre) to gain access to the 12-acre tract. Amos has never received permission from Andy to use the road across Whiteacre to gain access to Greenacre. 88. In an appropriate action by Andy to enjoin Amos from using the existing easement to gain access to Greenacre, the plaintiff will most likely (A) succeed, because Amos is making use of the servient tenement beyond the scope and extent of the easement as it was originally created (B) succeed, because Amos has no right to use the servient tenement in connection with a tract of land which is not part of the dominant tenement (C) not succeed, because Amos has an easement by necessity (D) not succeed, because Amos has a right to use the easement in a manner not inconsistent with the rights of the owner of the servient tenement

88. (B) According to Smith and Boyer, every easement appurtenant requires two pieces of land which are owned by two different persons. The two pieces of land involved are: (1) the dominant tenement, which is the land whose owner is benefited by the easement and (2) the servient tenement, which is the land whose owner is burdened by the easement. The owner of the dominant tenement is called the dominant tenant and the owner of the servient tenement is called the servient tenant. Second, when an easement appurtenant is created (either by conveyance or prescription) over a servient tenement the boundaries and extent of such easement become fixed and are binding on both the servient and the dominant tenants. It is important to remember that neither tenant has the right to change the location of such easement or change the boundaries of the servient or dominant tenement(s). In this example, many students will incorrectly choose choice (A). Here, choice (B) is the best answer because Amos has an easement appurtenant between Blackacre (the dominant tenement) and Whiteacre (the servient tene¬ment). He is attempting to use Whiteacre in connection with Greenacre, another tract of land that is not part of the dominant tenement. If, on the contrary, Amos attempted to make excessive use of Whiteacre in relationship to Blackacre, then choice (A) would be correct. For example, if Amos had a right of way for crossing Whiteacre (from Blackacre) on foot or by bicycle, then it would be beyond the scope of the easement for him to cross Whiteacre (from Blackacre) by automobile. This, however, is nor the situation. Therefore, choice (B) is the correct answer.

Mays owns Whiteacre, a 10-acre tract used for agricultural purposes, in fee simple. On Whiteacre Mays maintains a dwelling house for himself and his family. Mantle is the fee sim¬ple owner of Blackacre, a 5-acre tract, which abuts Whiteacre. In 1964 Mays began to erect a concrete wall along the bound¬ary line appurtenant to Blackacre. After Mays started to build the wall, Mantle informed him that he believed the wall was pro¬truding onto his property. Mays informed Mantle that he paid $500 to have the boundary line surveyed. Mays said that the sur¬veyor indicated that the wall did not encroach on Blackacre. Mantle accepted Mays' assurances and in 1980 devised Blackacre to Snider. After Snider entered into possession of Blackacre, he had the boundary line surveyed. The survey conclusively showed that the concrete wall extended two feet onto Blackacre. Although the encroachment does not interfere with Snider's use of Blackacre, he nevertheless demanded that Mays remove the wall. Upon Mays' refusal, Snider brought an appropriate action to have the wall removed. 89. The most likely result is that (A) Mays must remove the wall at his own expense (B) Mays must remove the wall but at Snider's expense (C) Mays may leave the wall without being liable to Snider for money damages (D) Mays may leave the wall but he will be liable to Snider for money damages

89. (C) This is a classic Multistate Property example where a substantial number of students will incorrectly choose choice (D). The test maker knows that many students will be led astray by viewing this as a trespass-type question. In trespass, of course, the plaintiff is entitled to relief if he (or she) shows an intentional unprivileged entry. If he (or she) does so, the plaintiff is then entitled at least to nominal damages and possibly an injunction prohibiting further trespass. This example, however, does not really deal with the issue of trespass. Rather, what we have here is a boundary line agreement between Mays and Mantle. As a general rule, judicial recognition is extended to boundary line agreements even if not executed with the formalities prescribed by the Statute of Frauds. Burby notes that application of the doctrine usually requires proof that the parties were not informed as to the true boundary line, that there was an express or implied agreement as to its locations and possession that conformed to the agreement. As such, Snider's (i.e., the grantee's) proper cause of action is not against Mays for trespass, but rather it is against Mantle (as grantor) for breach of covenant against encumbrances. An encumbrance that affects ownership but does not interfere with the use of land constitutes a breach of covenant against encumbrances. This is even true if the grantee knew of the encumbrance at the time he (or she) acquired ownership. Real Property, pg. 310.

93. Assume for the purposes of this question only that in 1954, after the conveyance from Babson, Callahan informed Groves Mining Co., that he would no longer honor the 1950 agreement permitting Groves to use the pathway. Groves brought an action for specific performance. Judgment for (A) Groves, since their property interest would "run with the land" (B) Groves, since the possessor of a servient interest would prevail against subsequent owners (C) Callahan, since Groves' interest was extinguished by the subsequent conveyance (D) Callahan, since there was no privity of estate between Callahan and Groves

93. (A) The provision in the 1950 agreement, whereby Groves was granted an easement appurtenant would "run with the land" and, thus, be enforceable against successors of the original parties. An easement appurtenant is, for purposes of succession, an incident of the possession of the dominant tenement. Even though an easement appurtenant is a property interest in the servient tenement rather than an interest in the dominant tenement, the easement privilege passes with the possession of the dominant tenement. As a consequence, the fact that Groves remained in possession of his adjacent dominant tenement is sufficient to establish his right to use the servient tenement in a manner consistent with the easement right.

In 1992, Johnson conveyed 100 acres of his Red Oak farm in Dallas County to Williams. The deed contained the following covenants: (1) seisin, (2) right to convey and (3) against encum¬brances. Subsequently, Williams conveyed the property to Allen by warranty deed. However, Allen is later evicted by Davidson because of paramount title. 94. Allen now brings suit against Johnson for breach of covenants in the deed. Judgment should be for: (A) Allen, because the covenants contained in the deed run with the land (B) Johnson, since no privity of estate exists between Allen and him (C) Allen, but only for the covenants of seisin and right to convey (D) Johnson, because the covenants are personal in nature and do not run with the land

94. (D) The covenants of seisin, right to convey and against encumbrances are personal covenants which, from their nature, must be broken instantaneously on the delivery of the deed or they are never broken. It follows that such personal covenants never run with the land and, there¬fore, the subsequent grantee may not maintain a cause of action against the original grantor.

95. For the purposes of this question only, suppose that Johnson conveyed the property to Williams and there was an out¬standing mortgage. The deed contained the above-mentioned covenants. Williams took possession, and shortly thereafter, threatened with foreclosure, he paid off the mortgage with interest. Williams now sues for breach of covenant against encumbrances. The court will most likely allow recovery for: (A) the amount in principal and in interest thereon from the time of the mortgage payment (B) only the principal which Williams paid on the mortgage (C) the measure of value between the value of the land with and without such an encumbrance (D) no recovery, since Williams should have brought suit against Johnson immediately upon notice of the out¬standing mortgage

95. (A) Since the covenant against encumbrances (i.e., existence of mortgage) was breached at the instant of the delivery of the deed, Williams will succeed in his action because the encumbrance of the mortgage burdened the property at the time of conveyance, entitling him to recover the amount he was compelled to pay in principal and in interest with interest accruing from time of payment.

97. In 1980 what is the present state of title to Scenicacre if the jurisdiction's recording act provides: "Every conveyance of real estate which is not recorded is void against a subse¬quent purchaser in good faith for valuable consideration, whose conveyance shall be first duly recorded?" (A) In a race-notice jurisdiction, Oscar, as a subsequent bona fide purchaser, would acquire record title to Scenicacre, since he was the last in time to record. (B) In a pure race jurisdiction, Buyer would acquire record title to Scenicacre, since he purchased the property for value and recorded first in time. (C) In a race-notice jurisdiction, Vendee, as a subsequent bona fide purchaser without notice of the prior Ohner-to-Son instrument, would acquire record title to Scenicacre, since he recorded his deed first in time. (D) In a pure race jurisdiction, Vendee, as a subsequent bona fide purchaser without notice of the prior Ohner-to-Son instrument, would acquire record title to Scenicacre, since he recorded his deed first in time.

97. (C) The recording act referred to in this question is a race-notice type statute. Under such a statute, an unrecorded conveyance or other instrument is invalid as against a subsequent bona fide purchaser for value without notice, who records "first." In effect, a race-notice statute combines the essential features of both the notice and race type recording statutes. In order for a subsequent party to prevail in a race-notice jurisdiction, he must be both a bona fide pur¬chaser for value without notice of the prior interest and record first. In our hypothetical, Vendee was a subsequent BFP for value and without notice of the prior Ohner-to-Son deed. Moreover, Vendee purchased Scenicacre in 1975 and immediately recorded the deed. Vendee recorded his deed first in time. (Note: Oscar did not purchase the property until 1979). Thus, Vendee is the record titleholder of Scenicacre in 1980 under this race-notice recording statute.

98. Assume for the purposes of this question only, that (in question 97), when Vendee purchased Scenicacre from Buyer in 1975, he was aware of the Ohner-to-Son instrument. In 1980, in an action to quiet title to Scenicacre, which of the following parties would have priority of title? (A) Oscar (B) Buyer (C) Vendee (D) Purchaser

98. (A) In 1980 Oscar would succeed in an action to quiet title to Scenicacre under a race-notice type recording statute. To reiterate, the recording act in the previous question is a race-notice type statute. In order to gain priority of title in a race-notice jurisdiction, a subsequent party must be (1) a BFP for value without notice of any prior interests and (2) record first. Since Buyer and Vendee were both aware of the existence of the prior Ohner-to-Son deed, the notice requirement could not be met as to them. Furthermore, Purchaser, who was unaware of any prior interests, failed to record first in time. Therefore, Oscar would prevail since he was the only party to satisfy both requirements; he was a subsequent BFP for value without notice (paying $25,000 for the property) and was the first to record (between Purchaser and himself).

On February 1, 1956, Williams conveys his farm known as "The Oaks" to Adams, and Adams duly records the conveyance. The following day, Adams conveys the property to Betty; she does not record her deed. Then on February 4th, Adams executes an identical conveyance of "The Oaks" to Clyde. Clyde gives Adams a check for $100,000 for the property and records the conveyance even though he has actual knowledge of Betty's prior conveyance. Betty however, records her deed on February 6th. Clyde then conveys his interest in the farm to Dave, who gives a purchase price of $115,000 to Clyde. On February 5th, Dave purchases the farm without notice of the conveyance to Betty and duly records the deed. 99. In conducting a title search, Dave should pursue his inves- tigation by looking in the: (A) Grantor Index under Clyde's name to ascertain if Clyde acquired title (B) Grantee Index under Clyde's name only (C) Grantee Index under Clyde's name, then Grantor Index under Clyde's name, and then look in Grantee Index again, this time under Adams' name to discover if he acquired title (D) Grantee Index under Betty's name, then to the Grantor Index also under Betty's name to find out if she made any prior conveyances

99. (C) Betty's recording is "out of the chain of title" and therefore it is not notice to Dave.

PACIFIC OCEAN House MALIBU PROPERTY VOGEL PROPERTY PACIFIC COAST HIGHWAY UNDEVELOPED TRACT WHERE THE LANDSLIDE ORIGINATED. PROPERTY OWNED BY SEABREEZE REALTY CO. Question 59 is based on the following fact situation. The following incident occurred during the heavy rains that battered Southern California in the winter and spring of 1995. On February 12, 1995, a violent storm struck the Los Angeles basin area. Triggered by heavy rains and wind gusts of over 50 mph, a mudslide caused Velma Vogel's house to be swept down¬hill into a home owned by Marilyn Malibu. After the Vogel home crashed into Malibu's, the mudslide then carried both of them into the ocean. Undisputed evidence revealed that the mudslide started when a large plot of land, situated on an uphill slope, owned by the Seabreeze Realty Co., slid downward across Pacific Coast Highway towards the ocean. The landslide then caused the Vogel dwelling to dislodge, rotate and press against the home of her neighbor, Marilyn Malibu, and both homes then slid into the sea. 59. Malibu asserts a claim against Vogel to recover damages to her home caused by the mudslide. Judgment for whom? (A) Malibu, because a landowner has the absolute right to have his land supported laterally by the neighboring land. (B) Malibu, because one who withdraws lateral support from his neighbor's land is liable for the injury done to such land in its natural condition, regardless of negligence. (C) Vogel, because although Malibu may recover for injury to the land in its natural condition, she cannot recover for injury to the artificial structures thereon. (D) Vogel, because Malibu's proper cause of action should be against Seabreeze Realty Co.

C) If there are artificial structures on the land and the land in its natural condition would be injured by the taking away of lateral support, then there are two distinct views as to the dam¬ages recoverable: (a) in some states the recovery will include both the damages to the land and the damage to the artificial structures thereon (called the English rule, (b) but in the majority of states the recovery is limited to damage to the land in its natural condition and may not include any damage to the artificial structures on the land (called the American rule). The the¬ory of the American rule is that to permit the wronged landowner to recover for damage to his buildings is in substance a requirement that the adjoining landowner's land furnish lateral sup¬port for both the land and the buildings of the plaintiff.

Assume for the purposes of this question only that Reek has agreed to sell the property to Westin. However, Farpo has notified Reek that she intends to exercise her right of first refusal. Which of the following will be a relevant consideration regarding Farpo's right of first refusal? I. Statute of Frauds II. Parol Evidence Rule III. The Type of Recording Statute in Effect IV. The Rule Against Perpetuities (A) I only (B) IV only (C) II and III (D) I, II and III

Multistate Nuance Chart: PROPERTY RULE AGAINST PERPETUITIES AS APPLIED TO OPTIONS TO PURCHASE LAND OPTIONS CONTAINED IN A DEED INSTRUMENT R/P Applies OPTIONS CONTAINED IN ALEASEHOLD CONTRACT R/P Does Not Apply 191. (B) Another "key" Multistate testing area is whether a right of first refusal (i.e., option to purchase land in the future) violates the Rule Against Perpetuities. First, it is important to point out that the rule applies to options to purchase land in the future. In this example, the option violates the rule because the grantor's heirs are given until the year 2222 to exercise their right of first refusal. Since this option may not vest within the perpetuity period, it constitutes a violation of the rule.


Conjuntos de estudio relacionados

CHAPTER 26 CNS-SEDATIVES AND HYPNOTICS

View Set

Compare and contrast systems theory, adaptation theory, and developmental theory.

View Set

401: Community In-Class Questions

View Set

103H [HEALTH] - Chapter 9 - Patients, Providers, and Treatments

View Set

Microeconomics Final Exam (Chpt. 11, 12, 13)

View Set